Você está na página 1de 115

ACERTIJOS Y JUEGOS DE LÓGICA PARA

DESARROLLAR LA INTELIGENCIA. CÓMO


RAZONAR EN MATEMÁTICAS
Miguel @ifilosofia (Matemático por la Universidad Autónoma de Madrid)

20 de abril de 2013

1
INDICE

TEMA 1: ATRÉVETE A PENSAR

TEMA 2: ANÁLISIS DE LAS SOLUCIONES

TEMA 3: ¿QUÉ SON LAS MATEMÁTICAS?

TEMA 4: CÓMO ESTUDIAR Y COMPRENDER LAS MATEMÁTICAS

2
TEMA 1: ATRÉVETE A PENSAR

1) Tenemos tres bolsas de caramelos. En una hay caramelos de fresa, en otra ca-
ramelos de limón y en la tercera caramelos de fresa y de limón. Cada bolsa tiene una
etiqueta indicando el tipo de caramelos que contiene. Hay una etiqueta que pone fresa,
otra que pone limón y otra que pone mezcla. La persona que puso las etiquetas las
colocó todas mal. El aspecto de los caramelos de fresa es idéntico al de los caramelos
de limón y es imposible diferenciarlos hasta que no se los prueba. ¿Cuántos caramelos
y de qué bolsas son necesarios probar para conocer el contenido de cada bolsa?

2) El juego del Nim consiste en que dos jugadores van retirando palillos una vez
cada uno. Se pueden retirar el número de palillos que se desee, pero sólo de una fila. El
que retire el último palillo pierde. Hay una estrategia con la que gana uno de los dos
jugadores: el que comienza retirando palillos o el otro ¿Para qué jugador es? ¿Cuál es
esa estrategia?

II

III

IIII

3) Un viajante sale de Madrid hacia Toledo a las 9 de la mañana. Al dı́a siguien-


te regresa a Madrid saliendo de Toledo también a las 9 de la mañana . Averigua si
existe algún punto entre Madrid y Toledo por el que pasa los dos dı́as a la misma hora

4) Un comerciante tiene una garrafa de 16 litros llena de aceite y dos garrafas de


10 litros y 6 litros vacı́as. Un cliente le pide 8 litros. Razona cómo puede conseguir que
haya 8 litros de aceite en la garrafa de 10 litros. Las garrafas no tienen marcas para
poder medir litros

3
5) Averigua el número que sigue en la siguiente sucesión
1, 1, 2, 3, 5, 8, 13, 21...

6) Disponemos de 11 montones de monedas. En cada montón hay 10 monedas. Las


monedas de un montón son falsas. Las monedas verdaderas pesan 2 gramos cada una,
las falsas 1 gramo. Tenemos un peso electrónico -de un solo platillo- que nos da la
cantidad numérica exacta del peso. Averigua el número de pesadas necesarias para
detectar el montón de las monedas falsas. Razona la respuesta

7) Se quiere alicatar una pared con azulejos rectangulares. Estos azulejos ocupan
justo el doble que los cuadrados que figuran en el dibujo adjunto. Loa azulejos se pue-
den colocar horizontalmente, verticalmente o una mezcla de ambas. ¿Es posible hacerlo
sin tener que partir ningún azulejo rectangular? Si es posible resuélvelo. Si no lo es,
demuestra por qué no se puede. (Fuente: hoja volante de la Universidad Autónoma de
Madrid)

8) Antonio y Enrique se encuentran descansando en la orilla de un rı́o aprovechando

4
que sus ovejas están bebiendo. Antonio le dice a Enrique: me he dado cuenta que si
me dieras una oveja tendrı́a el doble que tú. Enrique le responde: y si me las dieras
tú los dos tendrı́amos las mismas. Averigua cuántas ovejas tiene cada uno

9) Tres amigos A, B y C deciden jugar al lanzamiento de una moneda. Comienzan


jugando A y B; uno elige cara y el otro cruz. El perdedor deja su puesto para que entre
a jugar C. Siempre es ası́, el perdedor deja su puesto al que no está jugando. El juego
lo gana quien acierte dos veces consecutivas el resultado del lanzamiento ¿Tienen los
tres amigos la misma probabilidad de ganar el juego? Razona la respuesta

10) El acertijo de Einstein. Tenemos 5 casas de cinco colores diferentes y en cada


una de ellas vive una persona de una nacionalidad diferente. Cada uno de los propie-
tarios bebe una bebida diferente, fuma una marca de cigarrillos diferente y tiene una
mascota diferente.
Tenemos los siguientes datos:

El británico vive en la casa roja


El sueco tiene un perro
El danés toma té
La casa verde está a la izquierda de la blanca
El propietario de la casa verde toma café
La persona que fuma Pall Mall tiene un pájaro
El propietario de la casa amarilla fuma Dunhill
El que vive en la casa del centro toma leche
El noruego vive en la primera casa
La persona que fuma Brends vive junto a la que tiene un gato
La persona que tiene un caballo vive junto a la que fuma Dunhill
El que fuma Bluemasters bebe cerveza
El alemán fuma prince
El noruego vive junto a la casa azul
El que fuma Brends tiene un vecino que toma agua

Averigua de quién es el pececito

5
11) Con 6 palillos construye 4 triángulos exactamente iguales. No se pueden partir
los palillos.

12) Un profesor de Matemáticas llega a un acuerdo con sus alumnos. El profesor


pondrá en fila a los 40 alumnos con una camiseta blanca, donde pegará un cı́rculo en
la espalda que puede ser de color azul o rojo. Cada alumno podrá ver los cı́rculos de los
demás, el suyo obviamente no. Los alumnos irán diciendo de uno en uno -comenzando
por el último de la fila, seguido del penúltimo, etcétera- el color del que creen que es el
cı́rculo de su camiseta. Los alumnos, a la hora de decidir el color, pueden salirse de la
fila, observar todos los cı́rculos, volver a su sitio y decir el color en voz alta, para que lo
oigan todos. No pueden hacer nada más, el profesor vigila muy de cerca y no pueden
hacerse gestos. Si consiguen acertar el color más de 20, obtendrán un sobresaliente
los 40 alumnos sin necesidad de examinarse. Se reúnen los estudiantes para estudiar
la forma de obtener de forma segura más de 20 aciertos ¿Podrán lograr el objetivo?
Nota: los colores se colocan al azar, no tiene por qué haber 20 cı́rculos de cada color
(Fuente: hoja volante de la Universidad Autónoma de Madrid)

13) Un determinado programa de ordenador al introducirle como datos los núme-


ros 1 y 2 da como resultado el número 3. Al introducir 2 y 3 da como resultado el 10.
Al introducirr 3 y 4 da el 21. Al introducir el 4 y 5 da el 36. ¿Qué resultado dará si
introducimos el 5 y 6 ?

14) Averigua el número que sigue en la siguiente sucesión


3, 10, 21, 36, 55...

15) Se ha medido la velocidad media a la que ha ido un atleta A al recorrer un


trayecto -para calcular esta velocidad media simplemente se divide la longitud del tra-
yecto entre el tiempo efectuado en recorrerlo- A la mitad del recorrido, otro atleta B,
ha obtenido una velocidad media que es la mitad de la velocidad media del atleta A en

6
todo el trayecto. Al atleta B se le comunica este dato e imprime un ritmo muchı́simo
más fuerte para intentar igualar la velocidad media de A en todo el trayecto. ¿Es po-
sible que el atleta B pueda obtener la misma velocidad media que el atleta A en todo
el trayecto? Razona la respuesta (Fuente: hoja volante de la Universidad Autónoma
de Madrid)

16) Disponemos de tres pegatinas rojas y dos azules. Ponemos al azar una pegati-
na en la espalda de cada uno de los tres niños que forman una fila india. Las dos
pegatinas que sobran se esconden -para que los niños no vean las que son- Los niños
saben que las pegatinas que les han puesto se han elegido de un total de 3 rojas y 2
azules. Se pregunta al último de la fila -que puede ver la pegatina de los dos de delante-
si puede averiguar el color de la suya y responde que no puede. Las respuestas se dan
en voz alta de tal forma que las oyen todos los niños. Se pregunta después al penúltimo
de la fila, que sólo puede ver la pegatina del que está delante de el, y también responde
que no puede. Se pregunta al primero de la fila -que no puede ver ninguna pegatina-
y acierta el color ¿Qué lógica ha usado para acertar el color y cuál es el color?

17) Di a un amigo que escriba en un papel un número de las cifras que desee y
que no te diga cuál es. A continuación que escriba debajo un número cambiando las
cifras de orden del original -se pueden cambiar como se quiera- y que tampoco te lo
diga. Ejemplo: escribe el 9752211 y debajo el 1212759. Ahora le pides que reste los
dos números. En este caso quedarı́a 9752211 - 1212759 = 8539452. De este resultado
tiene que suprimir un número distinto de 0, por ejemplo el 4, y decirte cuáles son los
números restantes, en este caso 853952. Con estos números restantes, ¿cómo puedes
saber que el número que ha suprimido es un 4?
Nota: al cambiar las cifras de orden del primer número hay que procurar que el número
obtenido sea menor que el original -para que se puedan restar-

18) Demostrar que en un conjunto formado por diez números naturales cualesquiera -
los números naturales son: 1, 2, 3,4, 5, 6 ....- existen al menos dos, cuya diferencia es un
múltiplo de 9. Nota: para resolver este acertijo es conveniente haber leı́do la solución
del acertijo 17

7
19) Coloca los números del 1 al 9 en el siguiente cuadrado de tal forma que todas las
columnas, todas las filas y las dos diagonales sumen lo mismo. Hay que resolverlo, no
tanteando, sino razonando matemáticamente

20) Este juego es otra versión del Nim -vuelve a leer el acertijo 2 de este capı́tulo- En
este modelo gana el que retire el último palillo. Hay una estrategia ganadora para el
jugador que comienza retirando palillos. ¿Cuál es esta estrategia?

II

III

IIII

IIIII

IIIIII

21) Marta y Pilar mantienen la siguiente conversación en la calle

Marta: ¿qué edades tienen tus tres hijos?


Pilar: si multiplicas las tres edades obtienes 36 y si las sumas obtienes el dı́a del mes
al que estamos hoy
Marta: pero me faltan datos
Pilar: es verdad: la edad del mayor supera a la suma de las edades de los otros dos

8
¿Cuáles son las edades de los tres hijos de Pilar?

22) Otro modelo del juego del Nim. -leer antes los acertijos 2 y 18 de este capı́tulo-
En este modelo gana el que retire el último palillo y hay una estrategia ganadora para
el jugador que retira palillos en segundo lugar ¿Cúal es esta estrategia?

II

III

IIII

IIIII

IIIIII

IIIIIII

23) El 70 por ciento de los alumnos de una clase aprobó Inglés; el 75 por ciento
Matemáticas; el 80 por ciento Filosofı́a y el 85 por ciento Quı́mica ¿Como mı́nimo,
cuántos aprobaron las cuatro asignaturas?

24) En el patio de un colegio hay 40 chicos y 40 chicas jugando, situados en el arco


de una circunferencia. Ocupan toda la circunferencia y están ordenados totalmente al
azar -es decir no están todos los chicos juntos, ni tampoco van alternándose chico-
chica-chico-chica...- El profesor decide cambiar de juego y necesita hacer dos grupos
donde haya en cada uno 20 chicos y 20 chicas. Como quiere hacerlo de la forma más
silenciosa y pacı́fica posible -para no molestar al resto de alumnos que están en clase-
se le ocurre un método en el que no hace falta que se muevan del sitio donde están y
donde los dos grupos van a quedar totalmente separados ¿Cuál es este método?

9
25) Este acertijo es igual que el 11 de este capı́tulo -vuelve a leerlo-, salvo una dife-
rencia. Ahora el profesor de Matemáticas coloca cı́rculos de tres colores: rojos, azules
y verdes. ¿Es posible que más de 20 alumnos acierten el color de su cı́rculo de forma
segura?. Nota: no tiene por qué haber la tercera parte de cada color (Fuente: hoja
volante de la Universidad Autónoma de Madrid)

26) El director de una prisión concede a un preso la posibilidad de quedar libre si


de las dos puertas que hay, averigua cuál es la de salida a la calle. En cada puerta
hay un guardián. El preso sabe que un guardián siempre dice la verdad y que el otro
siempre miente, pero no sabe quien de los dos es el mentiroso. Tiene que acercarse a
uno de los dos y hacerle una pregunta ¿Qué pregunta tiene que hacer para saber cuál
es la puerta de salida a la calle?

27) Construye con 24 palillos los nueve cuadrados de la siguiente figura. Retiran-
do 4 palillos tienes que conseguir que los nueve cuadrados se queden en cinco y que no
queden palillos aislados -sin formar parte de ningún cuadrado-

28) En un cuadrado 3 x 3 (3 filas y 3 columnas) hay que colocar los pares A1, A2, A3,
B1, B2, B3, C1, C2, C3, de manera que en cada fila y cada columna no se repitan letras
ni números. Ejemplo: una fila o una columna no puede ser A2, C1, A3, por repetirse
la letra A. Tampoco puede ser B1, A1, C2, porque se repite el 1. ¿Es posible colocar
los pares ası́? En caso afirmativo, hazlo. Si no es posible demuestra por qué no lo es

10
29) Este pasatiempo es igual que el 26 -leer el 26- pero ahora trabajaremos en un
cuadrado 2 x 2. Hay que colocar los pares A1, A2, B1, B2. ¿Es posible? En caso afir-
mativo, hazlo. Si no es posible demuestra por qué no lo es

30) Este pasatiempo es igual que el 26 -leer el 26- pero ahora trabajaremos en un
cuadrado 4 x 4. Hay que colocar los pares A1, A2, A3, A4, B1, B2, B3, B4, C1, C2,
C3, C4, D1, D2, D3, D4. ¿Es posible? En caso afirmativo, hazlo. Si no es posible de-
muestra por qué no lo es

31) Este pasatiempo es igual que el 26 -leer el 26- pero ahora trabajaremos en un
cuadrado 5 x 5. Hay que colocar los pares A1, A2, A3, A4, A5, B1, B2, B3, B4, B5,

11
C1, C2, C3, C4, C5, D1, D2, D3, D4, D5, E1, E2, E3, E4, E5. ¿Es posible? En caso
afirmativo, hazlo. Si no es posible demuestra por qué no lo es

32) Este pasatiempo es igual que el 26 -leer el 26- pero ahora trabajamos en un
cuadrado 6 x 6. Hay que colocar los pares A1, A2, A3, A4, A5, A6, B1, B2, B3, B4,
B5, B6, C1, C2, C3, C4, C5, C6, D1, D2, D3, D4, D5, D6, E1, E2, E3, E4, E5, E6.
¿Es posible? En caso afirmativo, hazlo

33) En un cuadrado de 4 x 4 (4 filas y 4 columnas) hay que colocar los números


del 1 al 16 de tal manera que al sumar los tres números de cada fila, de cada columna
y de cada diagonal -hay dos diagonales- se obtenga siempre el mismo resultado. Deduce

12
primero cuál debe ser la cantidad que tienen que sumar los números de cada fila, de
cada columna y de cada diagonal

34) En un cuadrado de 3 x 3 (3 filas y 3 columnas ) hay que colocar nueve núme-


ros naturales distintos: 1, 2, 3, 4, 5,...etc...-elige los que quieras- de tal forma que al
multiplicar los tres números da cada fila, de cada columna y de cada diagonal -hay
dos diagonales- se obtenga siempre el mismo resultado. También se exige que en el
cuadrado central haya un 16 y que no haya números repetidos

?? ?? ??
?? 16 ??
?? ?? ??

35) Esta acertijo es igual que el 32 -vuelve a leerlo- pero ahora se exige que en la
casilla central haya un 10

13
?? ?? ??
?? 10 ??
?? ?? ??

36) En un cuadrado 3 x 3 (3 filas y 3 columnas) hay que colocar los nueve prime-
ros números impares: 1, 3, 5, 7, 9, 11, 13, 15, 17 de tal modo que al sumar los tres
números de cada fila, de cada columna y de cada diagonal -hay dos diagonales- se
obtenga siempre el mismo resultado. Aunque el problema se puede resolver tantean-
do, existe una demostración matemática de la solución. Encontrar esta demostración
obviamente tiene más mérito

37) En el siguiente pasatiempo hay que sustituir las letras por dı́gitos 0, 1, 2, 3,
4, 5, 6, 7, 8, 9 de tal modo que se verifique la suma que se indica. A letras distintas
debemos asociarles números distintos y al sustituir las letras por números, ninguno
puede comenzar con un cero. Ejemplo: el 0534 no vale, pero si vale el 5034. En con-
creto, en este caso, la C y la R no pueden sustituirse por el cero

CAMY + RELA = REMAN

38) Queremos sumar los primeros mil números naturales 1 + 2 + 3 + 4 +.......+

14
999 + 1000. Si no pensamos algo inteligente el trabajo se nos hará largo y tedioso pues
tendremos que hacer 999 sumas. ¿Cómo puede realizarse efectuando solamente una
suma, una división y una multiplicación?

39) Averigua qué número sigue en la siguiente sucesión


0, 1, 2, 4, 7, 12, 20....

40) Disponemos de una servilleta de papel -un cuadrado de aproximadamente 15


centı́metros de lado-. Queremos construir una figura de tal modo que podamos in-
troducir la cabeza y nos quede una especie de medalla colgando del cuello ¿Cómo se
puede conseguir?

41) Este juego es para dos personas. El primer jugador dice un número del 1 al
3, por ejemplo el 3. El segundo jugador tiene que sumar 1, 2 ó 3 y decir el 4, 5 ó 6
respectivamente. Supongamos que dice el 5. Ahora le toca el turno al primer jugador,
que por ejemplo sumando 1, dice el 6. Siempre hay que sumar 1, 2, ó 3. El que diga
31 o más de 31 pierde. Busca una estrategia para ganar el juego

42) Este acertijo es muy parecido al 4 de este capı́tulo. Disponemos de 12 cofres


contieniendo cada uno 240 monedas. Las monedas de dos cofres son falsas. Cada mo-
neda verdadera pesa 3 gramos y cada moneda falsa 4 gramos. Tenemos una báscula
de precisión (no de dos platillos) que nos da el peso exacto cuando colocamos algo
encima. ¿Cuántas pesadas son necesarias para saber qué cofres cotienen las monedas
falsas? Razona la respuesta

43) En este juego disponemos de un octógono regular pero sólo dibujamos los 8 pun-

15
tos de los vértices -los lados no- Existen 28 formas distintas de unir estos 8 puntos.
El primer jugador elige dos puntos y los une con un segmento rojo. A continuación el
segundo jugador elige otros dos puntos y los une con un segmento azul. Ası́ sucesiva-
mente se van alternando las jugadas. Pierde el primer jugador que se vea obligado a
formar un triángulo con los segmentos de su color. Sólo cuentan los triángulos cuyos
tres vertices sean también vértices del octógono. Ejemplos: el triangulo con vertices
A-2-4 no cuenta porque no todos sus vértices son vertices del octógono. El triángulo
con vertices 2-4-6 si cuenta. En este juego no se conoce una estrategia ganadora para
alguno de los dos jugadores. Ası́ pues, puedes jugar con un amigo sin temor de que
conozca la forma de ganarte siguiendo una estrategia.

44) Un matemático dispone de siete tarjetas. En cada una de estas tarjetas ha es-
crito una serie de números del 1 al 100. Dice a un amigo que piense un número del 1
al 100 y que le indique las tarjetas en las que figura dicho número. En menos de 15
segundos el matemático averigua el número que ha pensado su amigo. ¿Qué criterio
ha usado el matemático para escribir los números?

45) Este acertijo es igual que el 35 de este capı́tulo -vuelve a leerlo- pero ahora con la
suma

16
COPA + CAPE + REOGA = MESAEL

46) Pedro está en una de las 4 habitaciones de su casa: comedor, dormitorio, coci-
na o salón. En la puerta de cada habitación ha dejado una nota. Sólo una de estas
notas es verdadera. Hay que averiguar en qué habitación está Pedro. Las notas que ha
escrito son estas:

En el comedor: Pedro está aquı́


En el salón: Pedro está en la cocina
En el dormitorio: Pedro no está aquı́
En la cocina: Pedro no está en el comedor

47) En la siguiente figura las letras representan a ciudades y las aristas a carrete-
ras. Elegimos como ciudad de salida la letra Z. Hay que trazar un camino que pase
una vez por cada ciudad y regrese al punto de partida. No se exige pasar por todas las
carreteras. Sólo se exige pasar por todas las ciudades una y sólo una vez y regresar a
la ciudad de partida. Obviamente la letra de la ciudad de partida y la letra del final
del camino deben coincidir, pero es la unica que puede repetirse. ¿Es posible realizar
este recorrido? En caso afirmativo hazlo, en caso negativo demuestra que no se puede.

17
48) En la siguiente figura, partiendo de un vertice cualquiera, por ejemplo el mar-
cado con amarillo, hay que recorrer todas las lı́neas una y sólo una vez y regresar a
dicho vértice. Por cada vértice se puede pasar más de una vez. Si es posible resuélvelo.
Si no se puede demuestra por qué no es posible.

49) En un hexágono unimos cada dos vértices con una arista, como muestra el grafo.
Obtenemos de esta forma 15 aristas -9 interiores- Partiendo de un vértice cualquiera
hay que recorrer cada arista una y sólo una vez y regresar al vértice de partida. ¿Es
posible? En caso afirmativo resuélvelo. Si no se puede demuestra por qué no es posible.
Cada lı́nea del recorrido tiene que ser una arista completa, no vale coger desvı́os por
puntos de corte entre aristas interiores

18
50) Este pasatiempo es análogo al 48 -vuelve a leerlo- pero ahora hay que resolver
un pentágono no regular.

51) En este acertijo tenemos los once primeros números naturales: 1, 2, 3, 4, 5, 6,


7, 8, 9, 10, 11. Tenemos que construir una sucesión de doce números de modo que
cumpla las siguientes condiciones:
1) La sucesión tiene que comenzar y terminar con el mismo número

19
2) Los diez números del centro tienen que ser los que queden libres, una vez que haya-
mos elegido el número que será primero y último de la sucesión. En estos diez números
no puede haber repeticiones
3) Dos números pueden ser consecutivos en la sucesión si pertenecen a la siguiente lista:

/ 1 y 3 / 1 y 8 / 1 y 9 / 1 y 10 /
/ 2 y 3 / 2 y 10 /
/3y7/
/4y5/4y8/
/5y6/5y7/
/6y8/6y9/
/7y9/
/ 8 y 11 /
/ 10 y 11 /
En la anterior lista no importa el orden. Ejemplos: después de un 2 puede ir un 10,
pero también después de un 10 puede ir un 2. Después de un 1 puede ir un 3 pero
también después de un 3 puede ir un 1
El trozo de sucesión: 8, 11, 10, 1, ........8 serı́a válido porque el 8 y el 11 pueden ser
consecutivos. Lo mismo ocurre con el 11 y el 10 -porque en la anterior lista está el /10
y 11/- y con el /10 y 1/
El trozo de sucesión: 2, 4...... no serı́a válido, porque el / 2 y 4 / no está en la lista
anterior, y no pueden ser consecutivos
Si es posible construye la sucesión, y si no lo es, demuestra que no se puede construir

52) Haciendo uso de todos los números naturales del 1 al 9, coloca uno distinto en
cada casilla para que al operar en la primera, tercera y quinta fila se obtenga 14, 10 y
20 respectivamente; y al operar en la primera, tercera y quinta columna se obtenga 18,
30 y 20 respectivamente. Razonando matemáticamente, demuestra que el problema
admite dos y sólo dos soluciones. Observación: en las filas segunda y cuarta no hay
que colocar números. En las columnas segunda y cuarta tampoco

20
+ + 14
+ x x
+ x 10
+ + x
+ + 20
18 30 20

53) Siete universidades están investigando sobre un mismo tema. En cada univer-
sidad hay dos cientı́ficos investigando. Los cientı́ficos pueden intercambiar unos dı́as
sus trabajos, pero sólo si pertenecen a universidades diferentes. Un cientı́fico de la Uni-
versidad de Parı́s puede intercambiar su trabajo con cualquiera de los otros cientı́ficos
de las otras universidades, pero no con su compañero de la Universidad de Parı́s. Al fi-
nalizar el tiempo que tienen para investigar, el cientı́fico Einstein, de la Universidad de
Berlı́n, pregunta a los otros trece -incluido su compañero de la Universidad de Berlı́n-
con cuántos han intercambiado sus trabajos. Cada cientı́fico le da una respuesta dis-
tinta ¿Con cuántos cientı́ficos ha intercambiado Einstein su trabajo? ¿Y el compañero
de Einstein de la Universidad de Berlı́n?
Nota: cuando un cientı́fico A intercambia su trabajo con otro B, este cientı́fico B puede
intercambiar su trabajo con otro C, pero no el trabajo que le ha pasado A. Es decir,
tiene que esperar a que A le devuelva su trabajo (Fuente: hoja volante de la Universi-
dad Autónoma de Madrid)

54) Consideramos las ciudades: Madrid, Burgos, San Sebastián, Barcelona, Castellón
y Albacete. Las ciudades están conectadas mediante las siguientes carreteras directas
-sin pasar por ninguna ciudad intermedia-

Existen dos carreteras que conectan Madrid y Burgos


Una que conecta Burgos y San Sebastián
Una que conecta San Sebastián y Madrid
Una que conecta Burgos y Barcelona
Una que conecta San Sebastián y Barcelona
Una que conecta Barcelona con Castellón
Una que conecta Barcelona con Albacete
Una que conecta Castellón con Albacete

21
Una que conecta Albacete con Madrid

¿Se puede salir de alguna ciudad de las siete, recorrer todas las carreteras una y
sólo una vez, y volver a la ciudad de partida?. Por cualquier ciudad se puede pasar
las veces que se quieran. Si se puede conseguir, resuélvelo. Si no es posible demuestra
que no se puede y añade las carreteras que hagan falta para que sea posible. Nota: no
vale tomar desvı́os por los cruces que pueda haber entre las carreteras. Si por ejemplo
elegimos el tramo Madrid- San Sebastián, hay que recorrerlo entero y no desviarse al
llegar al cruce con la carretera Burgos-Barcelona

55) Me he inventado dos números enteros mayores que 1. He escrito su producto


en un papel y se lo he dado al matemático A. He escrito su suma en otro papel y se lo
he dado al matemático B. Entonces, sin mirar cada uno más que su papel han dicho:
A: no sé la suma. B: no sé el producto. A: ya sé la suma. B: ya sé el producto. ¿Cuáles
son los números que me he inventado? (Fuente: hoja volante del Departamento de
Matemáticas de la Universidad Autónoma de Madrid)

56) El siguiente cuadrado de 8 x 8 ( 8 filas y 8 columnas ) hay que dividirlo en 4


regiones de igual forma y superficie y de modo que cada región contenga exactamente

22
uno de los cuatro primeros números naturales: 1, 2, 3, 4

5 6 7 1 8 9 10 11
12 13 14 2 15 16 17 18
19 20 21 3 22 23 24 25
26 27 28 4 29 30 31 32
33 34 35 36 37 38 39 40
41 42 43 44 45 46 47 48
49 50 51 52 53 54 55 56
57 58 59 60 61 62 63 64

57) En este problema hay que colorear las provincias de un paı́s de modo que dos
provincias que tengan frontera común tienen que colorearse con colores distintos. Si
dos provincias tienen por frontera común un punto, a efectos prácticos no se consi-
derará frontera -la frontera debe ser una lı́nea-. Si, por ejemplo, suponemos un paı́s
circular dividido en 6 sectores circulares de 60 grados, el centro del circulo no se con-
sidera frontera comun entre provincias. Los radios que forman los sectores si serı́an
fronteras entre paises
a) ¿Podrı́as dibujar un mapa que no se pudiera colorear usando dos colores pero sı́ usan-
do tres?
b) ¿Podrı́as dibujar un mapa que no se pudiera colorear usando tres colores pero
sı́ usando cuatro?
c) Podrı́as dibujar un mapa que no se pudiera colorear usando cuatro colores pero
sı́ usando cinco?

58) En este juego tenemos un tablero de 8 x 8 - igual que el del ajedrez- donde
hamos anulado dos casillas, las dos esquinas de una diagonal. El juego consiste en
que dos jugadores tienen que ir colocando piezas rectangulares sobre el tablero, una
vez cada uno. Las piezas rectangulares ocupan el espacio de dos casillas del tablero.
El primero que no pueda continuar colocando piezas pierde el juego. ¿Eres capaz de

23
encontrar una estrategia ganadora para alguno de los dos jugadores?

59) El masyu es un pasatiempo en el que hay que trazar un recinto cerrado pasando
por todos los cı́rculos blancos y negros cumpliendo las siguientes reglas:
1) Cada vez que se toque a un cı́rculo negro hay que formar un ángulo de 90 grados,
es decir, cada cı́rculo negro tiene que ser el vértice de un ángulo de 90 grados
2) Cada vez que pase por un cı́rculo blanco tengo que atravesarlo totalmente y en el
cuadro inmediatamente anterior o posterior -o en ambos- debo formar un ángulo de
90 grados

Construye un masyu y dáselo a un amigo para que lo resuelva. Tu amigo tiene que
construir otro y dártelo para que lo resuelvas. -a mayor número de cı́rculos blancos y
negros mayor dificultad tiene- . Ası́ pues, se debe acordar el número de cı́rculos blancos
y negros de los dos tableros. El masyu también puede tener forma de rectángulo, es
decir, una tabla con más columnas que filas

60) Dos paı́ses se encuentran en guerra. Un Paı́s está enviando espı́as al otro. Di-
chos espı́as hacen su trabajo y regresan. Al regresar corren el peligro de ser agredidos
por los guardias de su propio paı́s al cruzar la frontera.
Se requiere de un mecanismo de contraseña. Se asume que los espı́as son gente de
confianza pero que los guardias que vigilan la frontera no, pues comúnmente van a
embriagarse a bares locales, por lo que se debe asumir que el enemigo llegará saber
cualquier cosa que ellos sepan.

24
¿Puedes idear un protocolo de seguridad con el que el espı́a sea capaz de regresar a
salvo pero que, al mismo tiempo, impida al enemigo ser capaz de introducir sus propios
espı́as usando la información conocida por los guardias?

61) Para este juego necesitamos un cuadrado de 9 x 9 (9 filas y 9 columnas) -pero


todas las casillas de un mismo color- donde hemos anulado 9 casillas, las marcadas con
un -. Los jugadores van colocando fichas en las 72 casillas marcadas con +, una vez
cada uno. Un jugador puede usar fichas blancas y el otro negras. Un jugador no puede
tener dos fichas en el tablero que se relacionen mediante el movimiento del caballo
de ajedrez -es decir que se pueda ir de una a otra mediante un salto de caballo-. El
primer jugador que tenga en el tablero dos fichas que se relacionen por medio de un
movimiento de caballo pierde.

- + + + + + + - +
- + + + + + + - +
+ + + + + + + + +
+ + - + + + + + +
+ + + + + + + + +
- + + + + - + - +
+ + + + + + + + +
+ + + + + + + + +
+ + + - + + + + +

62) Supongamos que tenemos un suelo de baldosas cuadradas. Elegimos una bal-
dosa para el inicio de un recorrido. Este recorrido consiste en moverse por las filas y
las columnas del suelo, pero en cada paso solo podemos movemos a la baldosa adya-

25
cente a la que estamos -y no vale caminar en las direcciones diagonales-. El recorrido
termina regresando a la baldosa de partida. Se puede regresar por cualquier sitio, no
es necesario hacerlo por el mismo camino que hemos ido. Si realizas estos recorridos
varias veces observarás que siempre hay que dar un número par de pasos. ¿A qué se
debe esto? En la figura adjunta se marca un recorrido posible, donde 0 es la baldosa
de salida y las X señalan las casillas por donde vamos pasando. En este ejemplo hemos
dado 22 pasos. Aunque haya 21 X son 22 pasos, pues el último paso tiene que ser llegar
al 0.

x x x x x x x x
x x
x x x x x
x x
0 x x x x

63) Un Nim especial es un juego inventado hace algunos años por Robert E. Gas-
kell, y consistente en ir retirando fichas de un montón. Los jugadores actúan por
turno. En la primera jugada no se puede retirar el montón completo, aunque sı́ en las
sucesivas, siempre que se respeten las siguientes reglas: en cada turno es forzoso retirar
al menos una ficha, y ningún jugador puede retirar más del doble del número de fichas
que haya tomado su oponente en la jugada anterior. Por tanto, si un jugador se lleva
tres, el siguiente no podrá retirar más de seis. Gana la partida quien retire la última
ficha. Encuentra una estrategia ganadora para alguno de los jugadores, dependiendo
del número inicial de fichas que hay en el montón

26
64) De un cuadrado hemos quitado la cuarta parte, quedando la figura que se mues-
tra. Hay que dividir la figura en cuatro partes exactamente iguales -de forma y de
superficie- Si es posible, resuélvelo. Si no se posible, demuestra por qué no lo es

65) En este acertijo vamos a realizar un paseo espacial. Una termita está comiéndose
el cubito marcado con una X de un cubo de rubik de 125 cubitos. Una vez que se ha
comido un cubito tiene que comerse un cubito adyacente pero no puede desplazarse en
las direcciones diagonales. Es decir puede comerse uno que sea adyacente a su derecha,
izquierda, delante, detrás, arriba o abajo. ¿Puede comerse los 125 cubitos? En caso
afirmativo, describe un camino. Si no es posible demuestra por qué no se puede. Ver
la figura adjunta

66) Consideremos los primeros 200 números naturales: 1, 2, 3, 4, 5, 6,.....199, 200.


Hay que demostrar que si de estos 200 números elegimos 101 números, independiente-
mente de la manera que se elijan, siempre hay dos números tales que uno es múltiplo

27
del otro

67) Un matemático propone a su amigo Luis un juego. El matemático numera las


6 caras de 4 dados con números que puede elegir entre 1 e infinito (1, 2, 3, 4,.....)
Los números pueden repetirse en un mismo dado las veces que se desee. Ejemplo: Dos
numeraciones pueden ser (4, 4, 8, 8, 10, 20) y (5, 5, 8, 9, 10, 10) El juego consiste en
que cada uno elige un dado y van haciendo tiradas. El que saque la puntuación más
baja tiene que dar un euro al otro. Una vez que el matemático termina de numerar los
dados deja a su amigo Luis que elija el dado que estime más oportuno, el matemático
elige uno de los 3 restantes Después de varias tiradas, el matemático gana claramente
dinero a su amigo Luis. Luis propone jugar con el dado con el que habı́a estado jugando
el matemático. El matemático acepta y elige uno de los 3 restantes. Después de varias
tiradas con los nuevos dados, el matemático vuelve a ganar claramente dinero a su
amigo Luis. Este propone otro cambio y el matemático vuelve a ganar con esta tercera
elección de dados. Ası́ hasta una cuarta elección de dados. El matemático siempre gana
y siempre deja elegir dado a su amigo Luis y él elige entre los tres restantes (Decir que
el matemático siempre gana, no significa que gane en todas las tiradas, sino que, por
ejemplo, de 30 tiradas con cada elección de dados, gana en 20 y Luis en 10) Pregunta:
¿Cómo numeró el matemático los 4 dados?

68) Tenemos una curva cerrada como la de la figura adjunta, donde la curva se corta
a sı́ misma 15 veces. A cada punto de corte, mi amigo Pedro ha asignado una letra
distinta. Obsérvese que hay que partir de una curva cerrada de tal modo que en un
punto de corte no pueden concurrir tres ramas de la curva - sólo dos- Mi amigo ha
elegido una letra y una dirección de salida y ha recorrido la curva completamente. De
esta forma se tiene que pasar dos veces por cada letra. Sin decirme nada de la forma
de la curva, me ha ido leyendo en voz alta todas las letras por donde iba pasando.
Es decir, una sucesión de 30 letras donde cada letra aparece dos veces. Como era la
primera vez que Pedro hacı́a esto, ha cometido un error. Ha leı́do correctamente la su-
cesión de 30 letras, salvo en dos lugares que permutó el orden de dos letras. Ejemplo:
en lugar de leer P, C, A, J, E ,L, Z, H, N, Y, H, W, X, A, J, S, K, B, W ,N, Y, Z, B,
K, L, P, C, X ,S, E, que es lo correcto, me dijo: P, C, A, J, E ,L, Z, H, N, Y, H, W, X,
A, J, S, K, B, N, W, Y, Z, B, K, L, P, C, X ,S, E.....Sólo cambió de orden la W y la
N en las posiciones 19 y 20. Rápidamente me di cuenta del error y se lo comuniqué a
mi amigo. ¿Qué hice para darme cuenta del error?

28
29
TEMA 2: ANÁLISIS DE LAS SOLUCIONES

Solución del 1) Sólo se necesita probar un caramelo de la bolsa etiquetada como


mezcla. El análisis lógico es el siguiente: supongamos que sale un caramelo de fresa.
Como la bolsa está mal etiquetada y dice que hay mezcla de caramelos, sólo contiene
caramelos de fresa. De esta forma, en la bolsa etiquetada como limón, no puede ha-
ber ni caramelos de fresa ni de limón -porque los caramelos de fresa ya los tenemos
localizados en la bolsa con la etiqueta mezcla; y porque al estar mal etiquetada no
puede contener caramelos de limón- La conclusion es que en la bolsa con la etiqueta
limón hay mezcla de caramelos. Por consiguiente, en la bolsa etiquetada como fresa,
hay caramelos de limón -pues es el único sabor que nos faltaba localizar-

Si el caramelo que pruebo de la bolsa etiquetada como mezcla es de limón, el análisis


es idéntico, pero ahora la bolsa con la etiqueta mezcla contiene caramelos de limón.
La etiquetada como fresa contiene mezcla de caramelos y la etiquetada como limón
contiene caramelos de fresa

Solución del 2) El que comienza el juego tiene que retirar 3 palillos de la fila que
tiene 4. De esta forma ganará seguro. Si el segundo jugador retira un palillo de la fila
que tiene 2, entonces el primer jugador, retira dos palillos de la fila que contiene 3
y gana. Si el segundo jugador retira los dos palillos de la fila que tiene 2, entonces
el primer jugador, retirando los 3 palillos de la fila que tiene 3, gana. Si el segundo
jugador retira un palillo de la fila que tiene 3, el primero retira el palillo que queda en
la última fila -la fila que comenzó con 4 palillos- y gana. Si el segundo jugador retira
2 palillos de la fila que tiene 3, el primero retira un palillo de la fila que tiene 2 y
gana. Si el segundo jugador retira los 3 palillos de la fila que tiene 3, el primer jugador
retira los 2 palillos de la fila que tiene 2 y gana. Último caso: si el segundo jugador
retira el palillo que queda en la ultima fila -la fila que comenzó con 4 palillos- el primer
jugador retira un palillo de la fila que tiene 3 palillos y gana. Un poco más adelante
veremos que el juego del Nim tiene una base matemática sólida y daremos estrategias
ganadoras cuando el juego conste de más filas y más palillos

30
Solución del 3) Existe un punto y sólo uno por donde pasó los dos dı́as a la mis-
ma hora. Se puede demostrar matemáticamente usando una propiedad de las funciones
continuas que se deriva del Teorema de Bolzano. Aquı́ lo demostraremos sin matemáti-
cas, de una forma más imaginativa. Si un coche sale de Toledo a Madrid a las 9 de la
mañana y el mismo dı́a sale otro coche de Madrid a Toledo a las 9 de la mañana, es
obvio que se cruzarán en un punto de la carretera. Imaginemos que el dı́a que el via-
jante hace el recorrido Toledo-Madrid, un segundo coche sale de Madrid hacia Toledo
de tal modo que en cada instante lleva la misma velocidad que llevó el viajante el dı́a
anterior. Es decir, lo que hace es repetir exactamente el movimiento del viajante del
primer dı́a. Estos dos coches se cruzarán en un punto por estar circulando el mismo
dı́a. ¿Qué ocurre en ese punto de cruce? Como está repitiendo de forma exacta el re-
corrido del viajante del primer dı́a, quiere decir que el viajante el dı́a anterior a esa
hora estaba en ese mismo punto. Por consiguiente por ese punto ha pasado el viajante
los dos dı́as a la misma hora

Solución del 4) Paso 1: con la de 16 litros llena la de 10 litros. Paso 2: con la


de 10 litros llena la de 6 litros. Paso 3: vacı́a la de 6 litros en la de 16 litros. Paso 4:
vacı́a la de 10 litros en la de 6 litros. Paso 5: llena la de 10 litros con la de 16 litros.
Paso 6: echa 2 litros de la de 10 litros a la de 6 litros.

Solución del 5) Es la famosa sucesión de Fibonacci. Podemos considerar que los


dos primeros términos: 1, 1, son arbitrarios, pero a partir de aquı́ cada término se ob-
tiene sumando los dos anteriores. 2 = 1 + 1; 3 = 1 + 2; 5 = 2 + 3 etc...Por consiguiente
el término que sigue en la sucesión es el 34

Solución del 6) Sólo se necesita una pesada. En esta pesada vamos a poner una mo-
neda del primer montón, dos del segundo montón, tres del tercer montón y ası́ sucesiva-
mente hasta poner10 monedas del décimo montón. Del undécimo montón no ponemos
ninguna. Si todas las monedas fueran verdaderas pesarı́an 110 gramos. ¿Qué ocurre si
por ejemplo pesan 107 gramos? Pues que hay 3 monedas falsas, porque cada moneda
falsa disminuye en un gramo el peso total. Por consiguiente las monedas falsas serı́an
las del tercer montón, pues de ese montón cogimos tres

31
Solución del 7) No se puede. La razón es la siguiente: pongo las letras A y B en
los cuadrados del dibujo, de tal modo que cada vez que me mueva en horizontal o en
vertical realizo un cambio de letra. La fila superir es A B A B A las siguientes son A
B A B A B A; B A B A B A B; A B A B A B A. La A aparece catorce veces, mientras
que la B aparece 12 veces. Cada vez que colocamos un azulejo rectangular tapamos
una A y una B. Si se hubiera podido alicatar tenı́amos que haber tapado 13 veces la
A y 13 veces la B. Esto es imposible porque la B sólo aparece 12 veces

Solución del 8) Antonio tiene 7 ovejas y Enrique 5. Si nos fijamos en el segundo


dato, Enrique dice: si me das tú una oveja los dos tenemos las mismas. Se deduce
que Antonio tiene dos ovejas más que Enrique. Vamos a suponer que elegimos dos
números altos, por ejemplo el 50 para Antonio y el 48 para Enrique. Antonio dice
que si Enrique le da una oveja tendrá el doble que él. Eligiendo estos números altos
es imposible que se cumpla eso, pues Antonio tendria 51 y Enrique 47. Observamos
que 51 está muy alejado de ser el doble. Si hubieramos elegido dos números más altos
la situación hubiera sido peor aún. Por ejemplo 100 y 98 se transforman en 101 y 97
ni mucho menos uno es el doble del otro. Esto sugiere que tenemos que probar con
números más pequeños. Probemos con 12 y 10. Se transforman en 13 y 9. Aún falta
mucho para que sea el doble. Probemos con números más pequeños. El 7 y el 5. Ahora
se transforman en el 8 y el 4. Ahora sı́ es el doble.

Este problema se puede resolver matemáticamente de una forma más directa que el
tanteo anterior: planteando un sistema de dos ecuaciones con dos incógnitas. LLamo x
al número de ovejas de Antonio e y al número de ovejas de Enrique. Si Antonio da una
oveja a Enrique Antonio quedarı́a con x − 1 y Enrique con y + 1. Como el enunciado
dice que quedarı́an con las mismas ovejas igualo estas dos cantidades y obtengo mi
primera ecuación

x−1=y+1
El enunciado tambien dice que si es Enrique el que da la oveja a Antonio, Antonio
tendrá el doble. Este dato lo paso a otra ecuación, buscando el igual. Si Enrique le da
una oveja a Antonio, Antonio tiene x + 1 y Enrique y − 1. Como dice que Antonio
tendrı́a el doble, el igual es: x + 1 = 2(y − 1) Antonio que es la parte izquierda del =,
es el doble de Enrique, que es la parte derecha del =. De esta manera he obtenido las
dos ecuaciones siguientes:

x−1=y+1

32
x + 1 = 2(y − 1)

Despejando la x de la primera ecuación queda: x = y + 1 + 1; Operando queda:


x = y + 2 ; Sustituyendo en la segunda ecuación queda: y + 2 + 1 = 2(y − 1); Operando
el paréntesis queda: y + 2 + 1 = 2y − 2; Juntando los terminos que tienen y a un lado
del =, y los que no tienen y a otro queda: 2 + 1 + 2 = 2y − y; Operando queda 5 = y;
Sustituyendo el valor de y en la igualdad x = y + 2 queda: x = 7; -coincidiendo con la
solución que habı́amos obtenido por tanteo-

Solución del 9) No tienen los tres jugadores la misma probabilidad de ganar el


juego. Los dos jugadores que comienzan tienen más probabilidad de ganar el juego que
el otro. Por otra parte, es obvio, que los dos que comienzan jugando tienen la misma
probabilidad -pues comienzan a jugar en el mismo instante y en idénticas condiciones-
Vamos a esbozar una demostración matemática

Supongamos que lanzamos un dado y nos piden la probablidad de que salga un


número par. Es obvio que la probabilidad es de un 50 por ciento, pues hay tres núme-
ros pares y tres impares. La probabilidad no trabaja en tantos por ciento sino que
trabaja con números comprendidos entre 0 y 1. Al anterior porcentaje hay que asig-
narle un número entre 0 y 1. Se asigna simplemente dividiendo entre 100 y obtenemos
el 0.5; este 0.5 es como medir un ”porcentaje”pero no sobre 100, sino sobre 1. Tene-
mos un 0.5 por 1 de que el resultado sea un número par. Este 0.5 tambien se puede
obtener aplicando la fórmula fundamental de la probabilidad. Esta fórmula dice que la
probabilidad de que ocurra un suceso es igual al número de casos favorables dividido
entre el número de casos posibles. En nuestro caso el suceso es: número par. Los casos
favorables son tres: el 2, el 4 y el 6; y los casos posibles son seis: el 1, 2, 3, 4, 5, 6.
Dividiendo 3 entre 6 obtenemos justo el 0.5. En nuestro acertijo vamos a medir la
probabilidad de que C gane el campeonato

Si analizamos las partidas en las que C puede proclamarse campeón y que termine
el juego, vemos que sólo puede ocurrir esto en las jugadas: 3, 6, 9, 12, 15....etc...Veamos
ahora las probabilidades de que C se proclame campeón en la jugada 3, en la jugada
6,... etc..Vamos a usar una notación de letras seguidas. Por ejemplo, las letras ACBB
significan que la primera partida la ha ganado A, la segunda la ha ganado C, la tercera
B y la cuarta B.

La probabilidad de AA es 1/4 pues es un caso favorable entre 4 posibles -AA, AC,


BB, BC- La probabilidad de una lista de 3 letras es 1/8, esto es facil de entender: hasta
2 letras la probabilidad es de 1/4 pero la tercera jugada puede tener 2 ganadores, luego

33
que se verifiquen 3 letras es el doble de difı́cil que se verifiquen 2- La probabilidad de
que se cumplan 4 letras es 1/16 por ser el doble de difı́cil que 3 letras..etc.. Los casos en
los que C puede proclamarse campeón en la tercera partida son: ACC y BCC. Como
cada una de estas 3 letras tiene una probabilidad de 1/8, si las sumamos obtenemos
1/8 + 1/8 = 2/8 = 1/4. Los casos en los que C puede proclamarse campeón en la
sexta partida son: ACBACC y BCABCC. La probabilidad de cada una de estas filas
de letras es 1/2 elevado a 6. Si las sumamos queda 1/2 elevado a 5. Siguiendo de este
modo, la probabilidad de que C se proclame campeón en la partida 9 serı́a 1/2 elevado
a 8. La probabilidad de que C se proclame campeón en la partida 12 serı́a 1/2 elevado
a 11. Ası́ seguirı́amos indefinidamente. Ahora hay que sumar todas las probabilidades
anteriores para calcular la probabilidad de que C sea campeón. Surge un problema:
tenemos que hacer una suma de infinitos sumandos. Sabemos sumar dos numeros, tres,
cuatro, cinco.. mil..etc.. pero no infinitos números.

¿Qué hacemos los matemáticos para sumar infinitos números? Sumamos los dos
primeros, despues los tres primeros, luego los 4 primeros,etc.. obteniendo sumas par-
ciales. Luego nos damos cuenta de que esas sumas parciales no pueden sobrepasar
una cantidad. Además a esa cantidad nos podemos aproximar todo lo que queramos.
¿Como nos aproximamos todo lo que queramos a esa cantidad? Simplemente sumando
más y más términos. Si existe una cantidad a la que nos podemos aproximar todo lo
que queramos sin sobrepasarla, decimos que esa cantidad es la suma de los infinitos
términos. Es como un techo que nunca sobrepasamos pero que podemos acercarnos a
él desde abajo todo lo que queramos. Matemáticamente se dice que este techo es a lo
que tienden las sumas parciales; o el lı́mite de las sumas parciales. Cuando decimos
que la suma de infinitos términos es 2 estamos diciendo que podemos encontrar sumas
parciales muy muy próximas a 2. No nos conformamos con encontrar sumas parciales
que valgan 1.9, vamos a encontrar sumas parciales mayores que 1.9 y menores que 2.
Tampoco nos conformamos con sumas parciales de 1.9999; vamos a encontrar sumas
parciales mayores que 1.9999 y menores que 2. No nos conformamos con nada, exigi-
mos acercarnos a 2 con una precisión infinita.

Si sumamos las probabilidades anteriores: 1/2 elevado a 2 + 1/2 elevado a 5 + 1/2


elevado a 8 + 1/2 elevado a 11 + ......La suma de los infinitos términos es 4/14 =
2/7. -Se ha aplicado la fórmula de la suma de los infinitos terminos de una progre-
sión geométrica, al final de la solucion viene explicado con detalle como se realiza la
suma-. Esta es la probabilidad que tiene C de proclamarse campeón. ¿Como se calcula
la probabilidad de A y de B de proclamarse campeon? Si lo miramos en términos de
tantos por ciento, la suma de las tres probabilidades tiene que ser el 100 por ciento.
Por ejemplo, uno puede ser el 35 por ciento, otro el 35 por ciento y otro el 30 por
ciento, pero entre los tres porcentajes tienen que sumar el 100 por ciento. Mirándolo
con números de probabilidad, la suma de las tres probabilidades tiene que ser igual a

34
1. Entonces si de 1 restamos la probabilidad de C de proclamarse campeón obtenemos
1 - 2/7 = 5/7 . Como la suma de las probabilidades de A y B tiene que ser 5/7 y los dos
tienen la misma probabilidad , dividiendo 5/7 entre 2 obtenemos la probabilidad de
A de proclamarse campeón -y también la de B- . Se obtiene 5/14. Luego el resultado
final es: la probabilidad de A de proclamarse campeón es 5/14; la probabilidad de B
de proclamarse campeon es 5/14; y la probabilidad de C de proclamarse campeón es
4/14. El objetivo de este acertijo es explicar lo que significa sumar infinitos términos
y ver un ejemplo de suma de infinitos términos de una progresión geométrica

¿Qué es una progresión geométrica? Es una sucesión de infinitos números tales que
para pasar de un término al siguiente se multiplica siempre por una cantidad fija deno-
minada razón. Ejemplo: comienzo con el 2 y voy multiplicando siempre por 3. Obtengo
la progresión geométrica: 2, 6, 18, 54, 162...y sigue indefinidamente. Para definir una
progresión geométrica sólo se necesitan dos datos: el primer término y la razón. Si la
razon es menor que 1 y mayor que -1, entonces existe la suma de sus infinitos términos.
La progresion geométrica anterior no se puede sumar porque la razon es 3 ( mayor que
1 y mayor que -1). Si calculamos sumas parciales 2+ 6, 2+6+18, 2+ 6 + 18 + 54,.....
los números que se obtienen son capaces de superar cualquier techo que pongamos. En
cambio la progresión geométrica que obtuvimos calculando la probabilidad de que C
se proclame campeón sı́ se puede sumar. Cuando una progresión geométrica se pue-
de sumar porque la razón es menor que 1, la suma se obtiene dividiendo el primer
término de la progresión entre (1 - r), donde r es la razón. Esto tiene su demostración
matemática, que no haremos aquı́. En el caso de las probabilidades de C tenı́amos la
sucesión: 1/2 elevado a 2, 1/2 elevado a 5, 1/2 elevado a 8, 1/2 elevado a 11.....Clara-
mente se ve que es una progresión geométrica pues para pasar de (1/2 elevado a 2),
a (1/2 elevado a 5), hay que multiplicar por (1/2 elevado a 3). Para pasar de (1/2
elevado a 5) a (1/2 elevado a 8) hay que multiplicar por (1/2 elevado a 3). Siempre
para pasar de un término al siguiente se pasa multiplicando por (1/2 elevado a 3).
Luego (1/2 elevado a 3) es la razón de la progresión geométrica. Aplicamos la fórmula
anterior de la suma y obtenemos (1/2 elevado a 2) dividido entre [1 - (1/2 elevado a
3)], que da como resultado 4/14 = 2/7

Solución del 10) El dueño del pececito es el alemán. Para llegar a este resultado
recorta cinco cuadrados de papel y sobre cada uno vas colocando los datos que se dan.
Es mejor con cuadrados que se puedan mover que con un dibujo de 5 cuadrados, por-
que el problema habla de posiciones de las casas. Los cuadrados recortados se pueden
mover de sitio segun vayamos leyendo los datos de posición. Con cuadrados dibujados
no se puede conseguir este cambio de posiciones

35
Solución del 11) Si construyes un tetraedro en tres dimensiones te darás cuenta que
tienes cuatro triángulos exactamente iguales. El tetraedro está formado por cuatro
triángulos equiláteros en sus caras -como muestra la figura adjunta– El mérito de este
problema es plantearse el problema, no en el plano, sino en el espacio. Este cambio
de contexto se emplea muchas veces en matemáticas para resolver problemas que no
tienen solución en un contexto dado

Solución del 12) 39 alumnos aciertan el color de forma segura y el primer alumno que
dice el color -el último de la fila- puede acertar con una probabilidad del 50 por ciento.
La estrategia que usan los estudiantes es la siguiente: el último de la fila -primero en
hablar- cuenta el número de cı́rculos rojos que hay por delante de él. Si el número es
impar dice rojo, si es par dice azul. El siguiente hace lo mismo, cuenta el número de
cı́rculos rojos que hay por delante de él. Si hay un cambio en la paridad respecto del
anterior -pasa de par a impar o de impar a par- es porque su cı́rculo es rojo. Si no
hay cambio es porque su cı́rculo es azul. De esta forma sabe el color de su cı́rculo y
acierta seguro. El siguiente alumno que dispone de la información que le han dado los
dos anteriores, hace lo mismo: contar el número de cı́rculos rojos que hay por delante
de él. Con todo esto deduce fácilmente el color de su cı́rculo. Observación importante:
El tercero en decir el color tiene que estar atento no sólo al color del cı́rculo que dijo
el anterior, sino a los dos anteriores. Esto es totalmente necesario para deducir si el
número de cı́culos rojos que contó el anterior a él es par o impar. Si pones un ejemplo
lo verás muy claro. El cuarto en decir el color tiene que estar atento a los colores que
han ido diciendo los tres anteriores -para deducir si el anterior a él ha contado un
número par o impar de cı́culos rojos- y contar los cı́culos rojos que hay por delante
de él. Ası́ sucesivamente, acertarán el color 39. El primero que dijo el color, que no
disponı́a de información previa, puede que acertara por casualidad, tiene un 50 por
ciento de probabilidad de haberlo hecho, al decir un color entre dos posibles

36
Solución del 13) El programa lo que hace es sumar los dos números y multiplicar
por el primero. Por consiguiente, al introducir el 5 y 6 tiene que dar como resultado
55

Solución del 14) Para pasar del primero al segundo se suman 7, del segundo al
tercero se suman 11, del tercero al cuarto se suman 15, del cuarto al quinto se suman
19.. -siempre se van sumando 4 más- De este modo, para pasar del quinto al sexto
habrá que sumar 23, y el sexto será el 78

Solución del 15) El atleta B ya no puede obtener la misma velocidad media que
A en todo el trayecto. La deducción es la siguiente: la velocidad media se calcula di-
vidiendo el espacio recorrido entre el tiempo empleado. En la mitad del recorrido el
atleta B ha obtenido una velocidad media que es la mitad de la velocidad media de A
en todo el trayecto. Como ha recorrido la mitad de espacio, para que la división sea la
mitad, el tiempo que ha tardado el atleta B en la mitad del trayecto tiene que ser el
mismo que el empleado por A en todo el trayecto. Por consiguiente, es imposible que
B emplee el mismo tiempo que A en recorrer todo el trayecto. Si pones un ejemplo con
números lo verás más claro

Solución del 16) Si el último de la fila no puede averiguar el color de su cı́rculo


es porque los cı́rculos que ve no son los dos azules, porque si fuesen los dos azules
sabrı́a que el suyo es rojo -ya que sólo hay dos cı́rculos azules-. Luego entre los dos
primeros, hay al menos un cı́rculo rojo. Si el penúltimo no puede averiguar su color
es porque el cı́rculo que ve es rojo, pues si fuera azul sabrı́a que su cı́rculo es rojo -ya
que habı́amos visto que entre los dos primeros habı́a al menos un rojo- Ası́ pues, el
primero de la fila sabe que su cı́rculo es rojo y acierta

Solución del 17) Si sumamos todos los dı́gitos de la resta obtenida 8539452, nos
queda: 8 + 5 + 3 + 9 + 4 + 5 + 2 = 36 que es múltiplo de 9. Al restar de un núme-
ro, otro número obtenido mediante una permutación de sus cifras, y sumar las cifras

37
obtenidas del resultado de la resta, siempre se obtiene un múltiplo de 9. Al quitar el
4 quedaron los dı́gitos: 853952 que al sumarlos queda: 8 + 5 + 3 + 9 + 5 + 2 = 32.
El siguiente múltiplo de 9 es 36, por consiguiente quitó un 4. Si al quitar un dı́gito
y sumar los que quedan se obtiene por ejemplo 44, quiere decir que ha quitado un 1,
pues el siguiente múltiplo de 9 es 45. Si al quitar un dı́gito y sumar los que quedan
se obtiene un múltiplo de 9, entonces es que ha quitado un 9. Vamos a ver por qué al
restar de un número uno obtenido mediante una permutación de sus dı́gitos y sumar
los dı́gitos obtenidos del resultado de la resta, siempre se obtiene un múltiplo de 9

Se dice que dos numeros son congruentes (módulo 9) Si se obtiene el mismo resto al
dividir los dos números entre 9. Ejemplo: 13 es congruente con 22 (módulo 9) Porque
al dividir 13 entre 9 se obtiene de resto 4 y al dividir 22 entre 9 tambien se obtiene de
resto 4. Esta definicion sugiere partir el conjunto de los numeros naturales en 9 trozos.
El primer trozo está formado por todos los números que al dividir entre 9 dan resto
igual a 0, es decir: 0, 9, 18, 27, 36, 45, 54,...El segundo trozo está formado por los
números que al dividir entre 9 dan resto 1, es decir: 1, 10, 19, 28, 37, 46, 55, 64,...El
tercero por los que dan resto 2, es decir: 2, 11, 20, 29, 38, 47, 56, 65,...El cuarto por los
que dan resto 3, es decir: 3, 1, 21, 30, 39, 48, 57, 65.. El quinto por los que dan resto
4, es decir: 4, 13, 22, 31, 40, 49, 58, 67......El sexto por los que dan resto 5, es decir: 5,
14, 23, 32, 41, 50, 59, 68, 77, 86... El septimo por los que dan resto 6. es decir: 6, 15,
24, 33, 42, 51, 60, 69, 78, 87.. El octavo por los que dan resto 7, es decir: 7, 16, 25, 34,
43, 52, 61, 70, 79, 88.... y el noveno por los que dan resto 8, es decir: 8, 17, 26, 35, 44,
53, 62, 71, 80, 89....

La siguiente propiedad es fundamental para la demostración: Un número es con-


gruente (módulo 9) con el número que se obtiene sumando sus cifras. Ejemplos: 14 es
congruente (módulo 9) con 5, 62 es congruente (módulo 9) con 8. Obsérvese que 5 y
14 están en el mismo trozo de los nueve anteriores. Lo mismo ocurre con 62 y 8. Esta
propiedad se puede demostrar, aunque no lo haremos aquı́

Otra propiedad: cuando a un número le permutamos sus dı́gitos se obtiene otro


número congruente (módulo 9) con el original. Esto es obvio por la propiedad anterior,
ya que la permutación no altera el valor de la suma de sus cifras. Ejemplo: 154 es
congruente (módulo 9) con el número obtenido sumando sus cifras, es decir, con 10.
Hago una permutación cualquiera de sus cifras y obtengo, por ejemplo, el 514. El 514
es congruente con el número obtenido sumando sus cifras, que vuelve a ser 10. Luego
si los dos números 154 y 514 son congruentes (módulo 9) con 10, ellos dos también
son congruentes (módulo 9) entre sı́. Divide 154 entre 3 y 514 entre 3 y verás que se
obtiene el mismo resto
Otra propiedad: Si restamos dos números que son congruentes (módulo 9) obte-
nemos un múltiplo de 9. Esto se puede comprobar eligiendo dos números del mismo

38
trozo, de los nueve trozos anteriores. Ejemplo: del octavo trozo elijo el 71 y el 44. Al
restar queda: 71 - 44 = 27, que es múltiplo de 9.

De las propiedades anteriores se sigue que cuando restamos de un número, otro


número obtenido mediante una permutación de sus cifras obtenemos un múltiplo de
9, es decir un número perteneciente al primer trozo de los nueve anteriores: 0, 9, 18,
27, 36, 45, 54, 63, etc...Como este número tiene que ser congruente (módulo 9) con el
que se obtiene sumando sus cifras, -primera propiedad de las tres anteriores- los dos
números tienen que pertenecer al mismo trozo de los nueve anteriores. Por consiguiente
el número obtenido al sumar sus cifras también tiene que pertenecer al primer trozo:
0, 9, 18, 27, 36, 45, 54.. es decir tiene que ser un múltiplo de 9

Solución del 18) Antes de leer esta solución, lee la solución del acertijo 16. Como
hemos visto en la solución del acertijo anterior, el conjunto de los números naturales
se puede descomponer en 9 trozos disjuntos dos a dos -no hay ningún número que
pertenezca a dos trozos- y de tal modo que al unir estos nueve trozos se obtiene el
conjunto de todos los numeros naturales. Un trozo está formado por los números que
al dividir entre 9 dan de resto 0, otro por los que dan resto 1, otro por los que dan
resto 2,... ası́ sucesivamente hasta el noveno trozo, formado por los números que al
dividir entre 9 dan de resto 8. Como el enunciado del acertijo dice que tenemos diez
números, es seguro que en alguno de estos trozos tiene que haber al menos dos nḿeros
de los diez que dice el enunciado del acertijo. Esto es fácil de razonar: si tenemos 10
pelotas en 9 cajas, es seguro que en alguna caja hay al menos dos pelotas. Supongamos
que en el trozo de los números que dan de resto 5 al dividir entre 9, hay dos números
del conjunto de 10 elementos. Por dar los dos números el mismo resto al dividir entre
9, la diferencia entre estos dos numeros es un múltiplo de 9. Ejemplo: el 23 y el 41
dan de resto 5 al dividir entre 9 y su diferencia es 18, múltiplo de 9. Esto es muy
fácil demostrarlo para cualesquiera dos números que pertenezcan al mismo trozo de
los nueve en los que quedó partido el conjunto de los números naturales. Supongamos
que m y n dan resto 5 al dividir por 9, entonces m= 9c + 5 donde c es el cociente; n=
9q + 5 donde q es el cociente. Si ahora hago la diferencia m - n los cincos desaparecen
y me queda 9c - 9q, que sacando factor común queda 9( c - q). Esta última expresión
me demuestra que la diferencia es un múltiplo de 9, pues es: 9 multiplicado por algo.
La demostración para otro resto es igual.

39
Solución del 19) Primero se deduce cuál tiene que ser la suma de cada columna,
cada fila y cada diagonal. La suma de los nueve primeros números naturales es 45 y
como esta suma está repartida en 3 filas que tienen que sumar lo mismo, divido 45
entre 3 y resulta que cada fila tiene que sumar 15 ( y cada columna y las dos dia-
gonales). La casilla del centro tiene que ser necesariamente un 5. Esto se demuestra
descartando todos los demás casos. El 4 no puede estar en la casilla del centro, porque
pongamos donde pongamos el 1, siempre estará en una misma fila, columna o diagonal
que el 4; y como suman 5, necesitamos un 10 para la otra casilla, pero sólo tenemos
del 1 al 9. Por la misma razón, en la casilla central no se pueden colocar ni el 3 ni el 2.
El 1 tampoco puede estar en la casilla del centro porque pongamos donde pongamos
el 2, siempre estará en una misma fila, columna o diagonal que el 1. Como suman 3,
necesitariamos un 12, que no tenemos. El 9 tampoco puede estar en la casilla del centro
porque coloquemos donde coloquemos el 8, siempre estará en una misma fila, columna
o diagonal que el 9; y sumarı́an 17. El 6 tampoco pueden ocupar la casilla central,
porque pongamos donde pongamos el 9 siempre estarı́a en una misma fila, columna o
diagonal que el 9 y como ya suman 15, no podrı́amos completarlo con ningún número
-nos pasarı́amos de 15-. Por la misma razón el 7 y el 8 tampoco pueden colocarse en
el cuadrado central. Por consiguiente en la casilla central tiene que estar el 5. El 9 no
puede estar en una esquina -de las 4 que hay- porque tendrı́a que formar parte de tres
sumas. Una suma tendrı́a que ser 9 + 5 + 1 -correspondiente a una diagonal- y para
las otras dos sumas tendrı́amos que tener dos formas de sumar 6, pero no conteniendo
al 5 y al 1. Sólo hay una manera: 2 + 4 porque 3 + 3 no es posible. Luego no es posible
colocar al 9 en una esquina. Por lo tanto el 9 tiene que estar en cualquier casilla de
las que quedan al descontar la casilla central y las 4 esquinas. Por simetrı́a cualquiera
de estas 4 casillas sirve para situar el 9. Una vez colocado el 9, en el extremo opuesto
tiene que ir el 1 para que tengamos 9 + 5 + 1 = 15. En la otra fila o columna afectada
por el 9 tienen que ir necesariamente el 4 y el 2 -por simetrı́a cualquier manera de
colocar el 4 y el 2 en esas dos casillas es válida- El resto del cuadrado está totalmente
determinado por lo que ya hemos puesto y solo faltarı́a completarlo. Es decir, la clave
es colocar los números de la siguiente figura

2
9 5 1
4

40
Solución del 20) Tenemos que escribir el número de palillos de cada fila en sistema
binario. ¿Cómo hacemos esto? Muy fácil. Cuando escribimos un número en sistema
decimal -el sistema habitual- por ejemplo el 523, observamos que 523 = 5 por(10 ele-
vado a 2) + 2 por(10 elevado a 1) + 3. Otro ejemplo: 4875= 4 por(10 elevado a 3)
+ 8 por(10 elevado a 2) + 7 por(10 elevado a 1) + 5. En binario sólo se pueden usar
los dı́gitos 0 y 1, y la base de las potencias anteriores tiene que ser un 2. Ejemplo: el
número 11011 es el número 1 por(2 elevado a 4) + 1 por (2 elevado a 3) + 0 (2 elevado
a 2) + 1 por(2 elevado a 1) + 1 = 27

Escribamos ahora el número de palillos de cada fila en binario. 1=1; 2= 10; 3= 11;
4 = 100; 5= 101; 6 = 110; y los ordenamos en la tabla adjunta, donde tambien he-
mos escrito en la primera fila las potencias de 2 (1 es una potencia de 2, es 2 elevado a 0)

Potencias de 2 4 2 1
Fila de 1 palillo - - 1
Fila de 2 palillos - 1 0
Fila de 3 palillos - 1 1
Fila de 4 palillos 1 0 0
Fila de 5 palillos 1 0 1
Fila de 6 palillos 1 1 0

Decimos que un jugador tiene una posición perdedora si al tocarle retirar palillos
se encuentra que en todas las columnas de la anterior tabla hay un número par de
unos. -sin contar el 1 de la primera fila, que es una potencia de 2- Se dice que un ju-
gador tiene una posición ganadora si al tocarle retirar palillos se encuentra con alguna
columna que tiene un número impar de unos- Si una columna no tiene unos diremos
que tiene un número par de unos. -pues tiene cero unos- Para ganar hay que retirar
palillos de tal manera que siempre le dejemos una posición perdedora al contrario.
En nuestro juego el que comienza retirando palillos tiene una posición ganadora, pues
tiene alguna columna con un número impar de unos -en este caso las tiene todas-
¿Cómo retirar palillos de tal modo que el contrario tenga una posición perdedora?
Nos fijamos en las columnas que tengan un número impar de unos y elegimos la que

41
más a la izquierda esté. En este caso es la columna del 4. Nos fijamos en un 1 de esa
columna, por ejemplo el 1 de la fila de 4 palillos. La fila 100 hay que transformarla
en 11, pues de esta forma le quedamos una posición perdedora al contrario. Como 100
es 4 y 11 es 3 el primer jugador tiene que retirar un palillo de esa fila. El segundo,
haga lo que haga después, llevará al primero a otra posición ganadora y éste tiene que
llevar al segundo a una posición perdedora. De esta forma el que comienza el juego
gana seguro. El primero gana seguro en este ejemplo porque la disposición inicial de
la tabla le daba una posición ganadora, pero en otros ejemplos con distintas filas y
distintos palillos puede ocurrir que gane el jugador que no comienza retirando palillos.
Esto ocurrirá precisamente cuando la disposición inicial de la tabla sea una posición
perdedora

Solución del 21) Analizamos las maneras de escribir 36 como producto de tres núme-
ros. 36 x 1 x 1; 18 x 2 x 1; 12 x 3 x 1; 9 x 4 x 1; 9 x 2 x 2; 6 x 6 x 1; 6 x 3 x 2; 4 x 3 x
3; Como Marta dice que le faltan datos es porque al realizar las sumas de los números
anteriores se encontró con más de una posible solución. Estas sumas son: 36 + 1 +1
= 38; 18 + 2 +1 = 21; 9 + 4 + 1 = 14; 9 + 2 +2 = 13; 6 + 6 +1 = 13; 6 + 3 + 2=
11; 4 + 3 + 3 = 10. El número 13 es el único que se repite en las sumas anteriores.
Se obtiene como 6 + 6 + 1 y como 9 + 2 + 2 luego la solución es una de estas dos
ternas de números. Como Pilar al final dice que la edad del mayor supera a la suma
de las edades de los otros dos, la solución es 9, 2, 2, porque en 6, 6, 1 no se cumple
este último dato

Solución del 22) El número de palillos de cada fila escrito en el sistema binario
es: 1=1; 2= 10; 3= 11; 4 = 100; 5= 101; 6 = 110; 7 = 111; Si los ordenamos en una
tabla observamos que el jugador que comienza retirando palillos parte de una posición
perdedora -leer la estrategia del acertijo 20-. Por consiguiente, el segundo jugador solo
tiene que seguir la estrategia ganadora descrita en el problema 20.

42
Potencias de 2 4 2 1
Fila de 1 palillo - - 1
Fila de 2 palillos - 1 0
Fila de 3 palillos - 1 1
Fila de 4 palillos 1 0 0
Fila de 5 palillos 1 0 1
Fila de 6 palillos 1 1 0
Fila de 7 palillos 1 1 1

Solución del 23) Dibuja el problema, verás como te quedará más claro. Puedes
partir de un cuadrado al que le vas aplicando porcentajes de su superficie. Primero
calculemos cuántos como mı́nimo han aprobado inglés y matemáticas. Supongamos
que el 30 por ciento de los que no han aprobado inglés, han aprobado todos matemáti-
cas. Aún me queda un 45 por ciento que han aprobado matemáticas e inglés. Es decir
como mı́nimo un 45 por ciento han aprobado inglés y matemáticas, porque si algunos
de los que no han aprobado ingles no han aprobado matemáticas, este 45 por ciento
de los que han aprobado las dos asignaturas serı́a mayor. Veamos esto con un ejemplo:
supongamos que el 5 por ciento del total no han aprobado ni inglés ni matemáticas.
Entonces me queda un 25 por ciento que no ha aprobado inglés pero si matemáticas.
Como las matemáticas las han aprobado un 75 por ciento tendrı́a que un 50 por ciento
habrı́a aprobado las dos asignaturas. Tenemos pues como mı́nimo un 45 por ciento
que han aprobado matemáticas e inglés. Al dibujar un 80 por ciento de los que han
aprobado filosofı́a, como mı́nimo cortará a este 45 por ciento anterior en un 25 por
ciento del total. Y al dibujar el 90 por ciento de los que han aprobado quı́mica, como
mı́nimo cortará al anterior 25 por ciento en un 15 por ciento del total. La solucion
final es esta, un 15 por ciento como mı́nimo han aprobado las cuatro asignaturas

43
Solución del 24) El profesor pensó en trazar un diámetro de tal modo que divi-
diera la circunferencia en dos arcos. En cada uno de estos arcos habrı́a 20 niños y 20
niñas. ¿Es esto posible? Sı́, veamos por qué.

Numeramos a los niños y niñas con los números 1, 2, 3,... hasta el 80, de forma
consecutiva y siguiendo una orientación en la circunferencia -por ejemplo la de las
agujas del reloj- Trazo un diámetro que los separe en dos grupos: uno que incluya
los números 1, 2, 3... hasta el 40 y otro que incluya los números 41, 42, 43, ... hasta
el 80. Supongamos que en el primer grupo hay 15 niños y 25 niñas. Entonces en el
segundo grupo hay 25 niños y 15 niñas. Ahora muevo el diametro inicial de tal modo
que divida la circunferencia en dos partes, la primera que comprenda desde el 2 al
41 y la segunda que comprenda desde el 42 hasta el 80 y termine en el 1. Con este
movimiento, el número de niños del primer trozo puede haberse quedado igual, puede
que haya aumentado en uno o puede que haya disminuido en uno. Vuelvo a mover
el diámetro de tal modo que divida la circunferencia en dos trozos: el primer trozo
que contenga a los números desde el 3 al 42 y el segundo trozo a los numeros desde
el 43 al 80 y termine con el 1 y el 2. Otra vez, el numero de niños del primer trozo,
como mucho varı́a en una unidad, es decir, puede haberse quedado igual, aumentado
en uno o disminuido en uno. Sigo haciendo estos movimientos del dı́ametro hasta que
el primer trozo coincida con el que va del 41 al 80, esto es, el segundo trozo del que
partı́ inicialmente. De esta forma he partido de 15 niños y he pasado a 25 niños y como
en cada movimiento del diámetro el número de niños como máximo ha variado una
unidad, necesariamente tiene que haber existido un instante en el que el número de
niños del primer trozo haya sido de 20 -y por tanto también 20 niñas-

Solución del 25) Siguiendo la estrategia que a continuación se explica, 39 alum-


nos aciertan el color de forma segura y el primero que habla tiene una probabilidad
del 33.333.. por ciento de acertar La estrategia es construir un conjunto formado por
tres elementos y asociar a cada color un elemento. Hay que definir una suma de tal
forma que al sumar dos elementos cualesquiera de este conjunto el resultado siga sien-
do un elemento del conjunto. Tambien hay que poder restar. Es decir, que al restar
dos elementos cualesquiera de este con junto el resultado siga estando en el conjunt.o
No es fácil pensar en un conjunto que verifique lo anterior. Si elijo números normales,
por ejemplo el 1, 2, 3, al sumar 1+ 3 = 4, el resultado se me sale del conjunto

DEFINICIÓN: Dos números son congruentes (módulo 3) si al dividir entre 3, los dos
números dan el mismo resto. Ejemplo: el 12 y el 15 son congruentes (módulo3) porque
al dividir12 entre 3 se obtiene el mismo resto que al dividir 15 entre 3 -en ambos casos
se obtiene resto 0-. Otro ejemplo: el 11 y el 16 no son congruentes (módulo 3) por-

44
que al dividir 11 entre 3 se obtiene de resto 2, y al dividir 16 entre 3 se obtiene de resto 1

Vamos a definir la clase del 3 como el conjunto formado por todos los números que
son congruentes con 3 (módulo 3). Es decir, los que al dividir entre 3 den el mismo
resto que cuando dividimos 3 entre 3 . Como al dividir 3 entre 3 se obtiene resto 0,
la clase del 3 está formada por los numeros que al dividir entre 3 se obtiene resto 0:
0, 3, 6, 9, 12, 15,18, 21, 24.....Definimos la clase del 4 como el conjunto formado por
todos los números que son congruentes con 4 (módulo 3). Es decir, los que al dividir
entre 3 den el mismo resto que cuando dividimos 4 entre 3. Como al dividir 4 entre
3 se obtiene resto 0, la clase del 4 está formada por los numeros que al dividir entre
3 se obtiene resto 1: 1, 4, 7, 10, 13, 16, 19, 22, 25, 28, 31..... Definimos la clase del 5
como el conjunto formado por todos los números que son congruentes con 5 (módu-
lo 3). Es decir, los que al dividir entre 3 den el mismo resto que cuando dividimos
5 entre 3 . Como al dividir 5 entre 3 se obtiene resto 2, la clase del 5 está formada
por los numeros que al dividir entre 3 se obtiene resto 2: 2, 5, 8, 11, 14, 17, 20, 23,
26..... Definimos la clase del 0 como el conjunto formado por todos los números que
son congruentes con 0 (módulo 3). Es decir, los que al dividir entre 3 den el mismo
resto que cuando dividimos 0 entre 3 . Como al dividir 0 entre 3 se obtiene resto 0, la
clase del 0 está formada por los números que al dividir entre 3 se obtiene resto 0: 0,
3, 6, 9, 12, 15, 18, 21, 24..... Observamos que la clase del 0 es idéntica a la clase del
3. Definimos la clase del 1 como el conjunto formado por todos los números que son
congruentes con 1 (módulo 3). Es decir, los que al dividir entre 3 den el mismo resto
que cuando dividimos 1 entre 3 . Como al dividir 1 entre 3 se obtiene resto 1, la clase
del 1 está formada por los números que al dividir entre 3 se obtiene resto 1: 1, 4, 7, 10,
13, 16, 19, 22, 25, 28, 31, 34.......Observamos que la clase del 1 es idéntica a la clase del
4. Definimos la clase del 2 como el conjunto formado por todos los números que son
congruentes con 2 (módulo 3). Es decir, los que al dividir entre 3 den el mismo resto
que cuando dividimos 2 entre 3 . Como al dividir 2 entre 3 se obtiene resto 2, la clase
del 2 está formada por los números que al dividir entre 3 se obtiene resto 2 : 2, 5, 8,
11, 14, 17, 20, 23, 26, 29, 32, 35...Observamos que la clase del 2 es idéntica a la clase
del 5

Notacion: la clase del 0 la vamos a denotar por [0], la clase del 1 por [1], la clase del
2 por [2], la clase del 3 por [3], etc...Hasta el momento tenemos que [0] = [3]; [1] = [4];
[2] = [5]. ¿Hay alguna clase más? En principio podemos creer que sı́, que estarııa la
clase del 6, la clase del 7, la clase del 8 etc... pero si nos paramos a pensar nos damos
cuenta de que sólo existen 3 clases distintas. Veamos esto: la clase del 6 está formada
por todos los números que son congruentes con 6 (módulo 3). Es decir, los que al
dividir entre 3 den el mismo resto que cuando dividimos 6 entre 3 . Como al dividir 6
entre 3 se obtiene resto 0, la clase del 6 está formada por los números que al dividir
entre 3 se obtiene resto 0: 0, 3, 6, 9, 12, 15,18, 21, 24.... Coincide con la clase del 3 y

45
con la clase del 0. Si analizo qué ocurre con la clase del 9 me doy cuenta que vuelve a
coincidir con la clase del 0 y con la clase del 3. Ası́ sucesivamente la clase del 12, la del
15, la del 18..etc.. coinciden todas con la clase del 0 y con la clase del 3. La clase del
7 es el conjunto formado por todos los números que son congruentes con 7 (módulo
3). Es decir, los que al dividir entre 3 den el mismo resto que cuando dividimos 7
entre 3 . Como al dividir 7 entre 3 se obtiene resto 1, la clase del 7 está formada por
los números que al dividir entre 3 se obtiene resto 1: 1, 4, 7, 10, 13, 16, 19, 22, 25,
28, 31..... Observamos que coincide con la clase del 1 y con la clase del 4. Si analizo
qué ocurre con la clase del 10 me doy cuenta que vuelve a coincidir con la clase del 1 y
con la clase del 4. Ası́ sucesivamente la clase del 13, la del 16, la del 19..etc.. coinciden
todas con la clase del 1 y con la clase del 4. La clase del 8 es el conjunto formado
por todos los números que son congruentes con 8 (módulo 3). Es decir, los que al
dividir entre 3 den el mismo resto que cuando dividimos 8 entre 3 . Como al dividir 8
entre 3 se obtiene resto 2, la clase del 8 está formada por los números que al dividir
entre 3 se obtiene resto 2: 2, 5, 8, 11, 14, 17, 20, 23, 26, 29, 32, 35...Observamos que
coincide con la clase del 2 y con la clase del 5. Si analizo qué ocurre con la clase del
11 me doy cuenta que vuelve a coincidir con la clase del 2 y con la clase del 5. Ası́ su-
cesivamente la clase del 14, la del 17, la del 20..etc.. coinciden todas con la clase del
2 y con la clase del 5. La conclusión de todo esto es que solo existen tres clases distintas.

[0] = [3] = [6] = [9] = [12] = ....etc...

[1] = [4] = [7] = [10] = [13] = ....etc...

[2] = [5] = [8] = [11] = [14] = ....etc...

Por lo tanto ya hemos construido un conjunto con sólo tres elementos: [0], [1] y [2]
Ahora hay que ver si se pueden sumar y restar. La forma lógica de sumar tiene que
ser esta: [3] + [5] = [8]; [6] + [4]=[10]; Es decir, sumar los números encerrados entre
corchetes. Pero hay que demostrar que la suma de clases no depende de los números
que elijamos encerrados entre paréntesis. Esto es totalmente necesario para que la su-
ma tenga sentido. ¿Qué quiere decir que la suma no dependa de los números elegidos
encerrados entre corchetes? Supongamos que en vez de elegir [3] y [5] elijo [6] y [8].
Como [3] = [6] y [5] = [8] al sumar [3] + [5] se debe obtenr lo mismo que al sumar [6] y [8]

[3] + [5] = [8] y [6] + [8] = [14]. Hemos obtenido el mismo resultado pues la clase
del 8 es idéntica a la clase del 14. Se puede demostrar que funciona siempre y no sólo
con este ejemplo. En Matemáticas siempre que se define una operación entre clases,
hay que demostrar que los resultados de las operaciones no dependen de los números
elegidos encerrados entre paréntesis. Matemáticamente se dice: la operación es inde-
pendiente de los representantes elegidos para realizarla. Si no se hubiese cumplido lo

46
anterior habrı́amos llegado a algo absurdo

Supongamos que [14] no hubiera sido igual a [8]. Si llamo A a [0] = [3] = [6] = [9]
= [12] = ....etc...B a [1] = [4] = [7] = [10] = [13] = ....etc...y C a [2] = [5] = [8] = [11]
= [14] = ....etc...Al sumar A + C eligiendo [3] y [5] se obtiene [8] = C; y eligiendo [6]
y [8] obtengo [14], que serı́a distinto de C. Por un lado tendrı́a que A + C = C y por
otro lado A + C serı́a distinto de C Las nueve sumas posibles son estas: A + A = A;
A + B= B; A + C = C; B + A = B; B+ B = C; B + C = A; C + A = C; C + B = A;
C + C =B. Para comprobar las sumas anteriores puedes elegir un representante de A,
otro de B y otro de C y sumar los números encerrados entre parentesis. Por ejemplo:
eligiendo [0] para A, [1] para B y [2] para C, al sumar B + C se obtiene [1] + [2] =
[3] y como [3] = [0] = A, se obtiene que B+ C = A. Para quienes tengan pensado
seguir estudiando matemáticas superiores la definición totalmente rigurosa de la suma
de clases es la siguiente: defino A + B como la única clase - llamemosla Z- tal que
para todo x perteneciente a A y para todo y perteneciente a B, [x+y] = Z . Si la suma
hubiese dependido de los representantes - números encerrados entre corchetes- esta
definición rigurosa no hubiera tenido sentido

¿Cómo se restan estas clases? A - B consiste en buscar una letra que sumada a B
me de A -esto no es otra cosa que la definición de resta de toda la vida- Esta letra que
busco es C, porque C+ B = A. De esta forma podrı́a seguir construyendo las nueve
restas posibles. Para quienes hayan visto la estructura de Grupo, el conjunto de clases:
A, B, C, junto a la suma de clases es un Grupo Conmutativo

Ya disponemos de todas las herramientas matemáticas necesarias para abordar la


estrategia del acertijo de los cı́rculos de tres colores. Asignamos a cada color una clase,
por ejemplo al rojo [0], al azul [1] y al verde [2]. El último de la fila -primero en hablar-
suma todas las clases que hay delante de él. Esta suma tendrá, por ejemplo, la forma:
[2] + [1] + [ 0] + [2] + [2] + [0]+... Supongamos que el resultado es [43]; como al dividir
43 entre 3 se obtiene de resto 1, la clase del 43 es igual que la clase del 1; [43] =[1] y
por tanto ha salido el color azul. Asi pues dice en voz alta: azul. El penúltimo de la
fila -siguiente en hablar- al sumar los que hay delante de él, por ejemplo se encuentra
con: [1] + [0] + [2] + [2] + [0]+..y da como resultado la clase del 41 que es igual a
la clase del 2; [41] = [2] ¿Cómo averigua su color? Muy fácil, restando [1] - [2] = [2]
porque [2] +[2] = [4] =[1] Por consiguiente el penúltimo dice el color verde y acierta
seguro. El siguiente en hablar que dispone de la información que le han dado los dos
anteriores -y no sólo el anterior- suma los colores que hay delante de él. Supongamos
que se encuentra con [ 0] + [2] + [2] + [0]+...y obtiene como suma la clase del 40,
que es igual a la clase del 1; [40] = [1]; ¿Cómo averigua su color? Sabe que el primero
en hablar obtuvo [1], con esta información y con el color que dijo el siguiente, puede
deducir facilmente que el anterior a él, al sumar, obtuvo [2]. Como hemos supuesto

47
que el tercero en hablar ha obtenido [1], para saber su color tiene que restar de lo
que obtuvo el segundo al sumar, lo que ha obtenido él al sumar, es decir [2] - [1] =
[1], porque [1] + [1] = 2 y sabe que su color es el que hemos asignado a [1], es decir,
el azul. Ası́ sucesivamente averiguan el color 39 alumnos. El primero en hablar que
no disponı́a de ninguna información previa puede acertar su color de casualidad. La
probabilidad que tiene de hacerlo es de un 33.333.. por ciento, pues dijo un color entre
tres posibles.

Solución del 26) Me acercarı́a a cualquiera de los dos guardianes y le preguntarı́a: Si


preguntase a tu compañero por la puerta de salida a la calle, ¿qué puerta me dirı́a que
es? Y luego eligirı́a la contraria de la que me ha dicho. De esta forma sabrı́a seguro cuál
es la puerta de salida a la calle. Vamos a analizarlo. Supongamos que he preguntado al
guardián que siempre dice la verdad. Como su compañero siempre dice la mentira, el
guardián que dice la verdad, me dirı́a la puerta falsa. Y eligiendo la contraria, saldrı́a
en libertad. Supongamos que he preguntado al guardián que dice siempre la mentira.
Como su compañero dice siempre la verdad, pero él dice siempre la mentira, también
me indicarı́a la puerta falsa. Por lo tanto, eligiendo la contraria de la que me dice,
también saldrı́a en libertad

Solución del 27) Los cinco cuadrados que tienen que quedar son los indicados con
números en la figura adjunta

1 2
3
4 5
Por lo tanto hay que quitar un palillo de cada uno de los cuatro cuadrados pequeños
que no se han numerado. Estos palillos son los que están formando parte de los lados
del cuadrado grande

Solución del 28) Sı́ tiene solución. Observad la figura adjunta

48
A1 B2 C3
B3 C1 A2
C2 A3 B1

Solución del 29) No es posible. La razón es muy sencilla. Supongamos que el par A1
lo hemos colocado en el cuadrado que ocupa la primera fila y primera columna. A la
derecha de este cuadrado necesariamente tiene que ir el par B2 -Para no repetir letra
ni número-. Pero debajo de este cuadrado también tiene que ir el par B2 -otra vez
para no repetir letra ni número- Pero entonces hemos repetido el par B2. Si supongo
el par A1 en cualquier otra posición el razonamiento es análogo

Solución del 30) Sı́, es posible. Obsérvese la figura adjunta

A1 B2 C3 D4
B3 A4 D1 C2
D2 C1 B4 A3
C4 D3 A2 B1

Solución del 31) Sı́, es posible. Obsérvese la figura adjunta

49
A1 B2 C3 D4 E5
B3 C4 D5 E1 A2
C5 D1 E2 A3 B4
D2 E3 A4 B5 C1
E4 A5 B1 C2 D3
Obsérvese que existe un orden matemático. La primera fila es libre. La segunda fila
se obtiene sumando una unidad a los números de cada letra. En la primera fila están:
A1, B2, C3, D4, E5. En la segunda B3, C4, D5, E1, A2 . Al sumar 1 al 5 hay que irse
al 1, por eso se pasa de E5 a E1. Para pasar de la segunda fila a la tercera se hace lo
mismo. Ası́ sucesivamente hasta completar la quinta fila

Solución del 32) No es posible. Curiosamente, en el cuadrado de 2 x 2 -que ya


se ha visto- y en el cuadrado de 6 x 6 no es posible encontrar una solución. En todos
los demás cuadrados si existe solución. La demostración se sale de los objetivos de este
libro- Lo que se puede intentar es resolver los cuadrados impares 7 x 7, 9 x 9, 11 x 11,
etc...con el método del acertijo anterior

Solución del 33) Como la suma de los 16 primeros números: 1 + 2 + 3 + ....+


16 = 136 y estos números están repartidos en cuatro filas. Cada fila debe de sumar 136
dividido entre 4, igual a 34 -y también cada columna y cada una de las dos diagonales-
Para encontrar la solución se parte de un cuadrado donde se han ido colocando conse-
cutivamente los números 1, 2, 3,... 16, completando la primera fila, después la segunda
fila, etc...como se indica en la figura de la izquierda. Ahora intercambiamos de posición
sólo cuatro parejas y obtenemos la solución. Intercambiamos el 2 con el 15, el 3 con el
14, el 5 con el 12, y el 9 con el 8. Obteniéndose la figura de la derecha

50
1 2 3 4 1 15 14 4
5 6 7 8 12 6 7 9
9 10 11 12 8 10 11 5
13 14 15 16 13 3 2 16

Solución del 34) Una solución es ésta: se comienza colocando un 1 encima del 16,
para que los números no se hagan demasiado grandes. Esto me obliga a colocar un 2 y
un 8 en la tercera fila, porque el cuadrado que queda libre en la tercera fila, es común
con la segunda columna y la única manera de conseguir 16 con una multiplicación de
números distintos es con el 2 y el 8. Esto se muestra en la figura de la izquierda. A
la izquierda del 1 obligatoriamente tiene que ir el resultado de multiplicar 2 x 16 =
32, porque el cuadrado de la primera fila y tercera columna es común a la diagonal
secundaria y a la primera fila. Esto se muestra en la figura del centro. Hacemos ahora
la multiplicación de la diagonal, que está completa en la figura de la derecha 32 x 16
x 8 = 4096. Y con esta cantidad es muy fácil deducir qué números hay que colocar en
las casillas que falta. La solución final se muestra en la figura de la derecha. Nota: este
problema puede admitir más soluciones con el dato inicial 16 en la casilla central

?? 1 ?? 32 1 ?? 32 1 128
?? 16 ?? ?? 16 ?? 64 16 4
2 ?? 8 2 ?? 8 2 256 8

Solución del 35) El razonamiento es análogo al acertijo anterior -vuelve a leerlo-


La solución se muestra en la figura adjunta

51
20 1 50
25 10 4
2 100 5

Solución del 36) Al sumar los números impares 1+ 3 + 5+ 7+......+15 + 17 se


obtiene 81. Como esta suma se reparte en tres filas que tienen que sumar lo mismo,
cada fila debe de sumar 81 didivido entre 3, es decir, 27. -y cada columna y cada dia-
gonal, de las dos que hay- En la casilla del centro tiene que ir necesariamente el 9. Lo
demostraremos por descarte de los otros ocho números. El 7 no puede ir en la casilla
del centro porque sitúe donde sit́ue al 1, estará ligado en una suma con el 7. Como 7
+ 1 = 8, necesito sumar 19 para llegar a 27, pero sólo hay números hasta el 17. Por la
misma razón tampoco pueden ir en el centro el 3 y el 5. El 1 tampoco puede ir en el
centro, porque sitúe donde sitúe el 3, estará ligado en una suma con el 1. Como 1 + 3
= 4, necesito sumar 23 para llegar a 27, pero el 23 no est-́sólo hay números hasta el 17-.
El 11 no puede ir en el centro porque en cualquier lugar que coloque el 17, estará ligado
en una suma con el 11. Como 11 + 17 = 28, me paso de 27. Por la misma razón el
13 y el 15 tampoco pueden ocupar la casilla del centro. El 17 tampoco puede ir en la
casilla central porque en cualquier sitio que coloque el 15, estará ligado al 17 en una
suma y como 15 + 17 = 32, me paso de 27. Despues de este razonamiento, sólo queda
el 9 para el cuadrado del centro. El 17 no puede ocupar una esquina de las cuatro que
hay, porque solo hay dos formas de sumar 10: 7 + 3 y 9 + 1 pero si el 17 ocupase una
esquina, 9 + 1 + 17 tendria que usarse en la diagonal y solo me quedaria una de las
dos formas anteriores (7 + 3) para ocupar una columna y una fila. Por lo tanto el 17
tiene que ocupar una de las casillas que no sea ni la central ni una de las cuatro es-
quinas. Con esto ya es muy fácil terminar de completar el cuadrado. Ver figura adjunta

7 5 15
17 9 1
3 13 11

52
Solución del 37) La solución es: 9567 + 1085 = 10652. Se comienza razonando que
el dı́gito de la izquierda de la suma resultante no puede ser un 2, porque al sumar dos
números de cuatro cifras no se puede obtener uno mayor que 19998. Por consiguiente
tiene que ser un 1. A partir de aquı́ se siguen analizando todas las posibilidades. Como
el segundo sumando comienza con un 1, para que al sumar se obtenga un número de
cinco cifras, el digito de la izquierda del primer sumando tiene que ser 8 ó 9... etc....

Solución del 38) Hay que fijarse que los cien sumandos se pueden agrupar de dos
en dos de la forma: (1 + 100) + (2 + 99) + ( 3 + 98) + ( 4 + 97) +.....Cada una de
estas sumas vale lo mismo, 101. Como tengo 50 grupos, la solución es 101 multiplicado
por 50 = 5050. He realizado una división para saber que salen 50 grupos, dividiendo
100 entre 2. Una suma 100 + 1 para saber lo que suma cada grupo. Finalmente una
multiplicación, 101 por 50

Solución del 39) Los dos primeros términos son libres, pero el tercero y los siguientes
se forman sumando los dos anteriores y añadiendo una unidad. El tercer término es 2
= 0 + 1+ 1; el cuarto término es 4 = 1 + 2 + 1; el quinto término es 7 = 2 + 4 + 1;
Por consiguiente el término que sigue en la sucesión es 12 + 20 + 1 = 33

Solución del 40) Doblamos el papel por la mitad. Desde el lado por donde se abre
la servilleta hacemos cortes perpendiculares de aproximadamente 1 cm de ancho, pero
sin llegar al otro extremo. En la figura adjunta son las lı́neas que comienzan en el lado
inferior-. Desde el otro extremo -lı́nea por donde doblamos la servilleta al principio-
hacemos cortes perpendiculares que dividan a las anteriores tiras en dos tiras iguales
-tambien sin llegar al extremo, tal y como se muestra en la figura-. Abrimos la servi-
lleta y ya lo tenemos

53
Solución del 41) Es claro que el jugador que diga el 30 gana, pues el otro está forzado
a decir 31 o pasarse de 31. Para decir el 30 tiene que haber dicho antes el 26, pues al
decir 26, el otro jugador no puede decir 30. Para decir 26, tiene que haber dicho antes
el 22, pues al decir el 22, el otro jugador no puede decir el 26. Ası́ sucesivamente, los
numeros para ganar son 18, 14, 10, 6 y 2

Solución del 42) Una solución es coger de cada cofre las siguientes monedas: 0,
1, 2, 4, 7, 12, 20, 33, 54, 88, 142, 231 Observese que cada número es la suma de los
dos anteriores más una unidad. De esta forma si hacemos las sumas de dos en dos
números de la anterior sucesión, siempre se obtienen resultados distintos. Si todos los
cofres contuvieran monedas verdaderas, al coger las monedas anteriores pesarı́an 1782
gramos. Supongamos que al pesar las monedas se obtienen 19 gramos más. Como el 19
solo se puede obtener como suma de elementos de la anterior sucesión sumando 7 y 12,
sabremos que las monedas falsas estarán en los cofres que contienen 7 y 12 monedas.

Solución del 43) En este juego no hay que escribir una solución porque hemos di-
cho que no se conoce una estrategia ganadora. Serı́a un excelente ejercicio intentar
encontrar alguna. El hecho de que no se conozca no quiere decir que no exista. Lo
que sı́ es cierto es, que si existe, es difı́cil encontrarla. Otra forma de dejar zanjada
la cuestión es demostrar que no existe. Demostrar la no existencia en este problema,
también es bastante difı́cil. Cuando se presenta un problema de este tipo - que no se
sabe qué ocurre- se dice que estamos ante un problema abierto

54
Solución del 44) Todos los núneros se pueden expresar como suma única de las
potencias de 2: 1, 2, 4, 8, 16, 32, 64,..etc...(1 también es potencia de 2, es 2 elevado
a 0). Ejemplos: 17 = 16 + 1; 22 = 16 + 4 + 2; 32= 32; El matemático escribe como
primer número de estas tarjetas las potencias de 2: 1, 2, 4, 8, 16, 32 y 64 . En la
tarjeta que comienza con el 1 incluye los números que al expresarse como suma de
las potencias de 2 contienen un 1 entre los sumandos. Ejemplos: 1 = 1; 3= 2 + 1;
5 = 4 + 1; 7 = 4 + 2 + 1; 9 = 8 + 1; 11 = 8 + 2 +1; etc...Es decir en la tarjeta
que comienza con el 1 ha incluido: 1, 3, 5, 7, 9, 11, etc...En la que comienza con 2
incluye los numeros que al expresarse como potencias de 2 contienen un 2 entre los
sumandos. Ejemplos: 2 = 2; 3 = 2 + 1; 6 = 4 + 2; 7 = 4 + 2 +1; 10 = 8 + 2; 11 =
8 + 2 + 1..etc..Es decir, en esta tarjeta incluye: 2, 3, 6, 7, 10, 11.. etc.. Siguiendo con
este método completa las siete tarjetas. ¿Cómo averigua el número que se ha pensado?
muy fácil, simplemente sumando los primeros números de las tarjetas donde figura el
número que se ha pensado

Solución del 45) La solución es: 7486 + 7680 + 90436 = 105602: Lo primero que
hay que deducir es que la M es un 1, pues dos números de cuatro cifras y un número
de cinco cifras no pueden sumar un número de seis cifras que comience con un 2, ni
con un 3, ni con un 4, etc...A continuación se deduce que la R tiene que ser un 8 ó un
9, pues si el tercer sumando comenzase con 7, como los dos primeros sumandos son
menores que 10.000, la suma serı́a menor que 100.000. De todo lo anterior se deduce
que la E sólo puede ser un 0 ó un 1, pues los dos primeros sumandos son menores que
10.000 y el tercero menor que 100.000. Se sigue razonando hasta llegar al resultado
final

Solución del 46) Pedro no está en el comedor, porque si lo estuviera habrı́a dos
notas verdaderas: la primera y la tercera. En el salón tampoco está, pues si lo estu-
viera habrı́a dos notas verdaderas: la tercera y la cuarta. Tampoco puede estar en la
cocina, pues si lo estuviera habria tres notas verdaderas: la segunda, tercera y cuarta.
Pedro está en el dormitorio. De esta forma sólo es verdadera la cuarta nota: Pedro no
está en el comedor

55
Solución del 47) Sı́ se puede. El recorrido es: Z–B–C–A–X–M–P–U–W–O–K–D–
N–V–T–E–R–S–Y–H–Z

En matemáticas y ciencias de la computación, un grafo es un conjunto de objetos


llamados vértices o nodos unidos por enlaces llamados aristas o arcos. Este acertijo es
un grafo. Otro ejemplo de grafo más sencillo es el de la figura adjunta

En un grafo no es obligatorio que todos los vertices esten unidos dos a dos mediante
aristas. En el grafo de la imagen obsérvese que los vértices 2 y 4 no están unidos por
una arista

Cuando un camino pasa por todos los vértices una y sólo una vez y regresa al vértice
de partida decimos que tenemos un ciclo Hamiltoniano. Obviamente el vértice inicial
y el final del ciclo deben coincidir, pero es el único vértice que puede repetirse. La
solución que se ha dado, es pues, un ciclo Hamiltoniano

Solución del 48) Sı́ se puede. En este caso los vértices son los puntos marcados
en negro y el punto amarillo. Existe un resultado en la teorı́a de grafos que afirma
que cuando cada vértice forma parte de un número par de aristas, se puede efectuar
un recorrido pasando por cada arista una sólo vez y volver al vértice de inicio. Por
los vértices se puede pasar más de una vez. Los 25 vértices del cuadrado grande for-
man parte de cuatro aristas. Los vértices exteriores al cuadrado forman parte de seis
aristas. Para encontrar el recorrido puedes comenzar sustituyendo el cuadrado grande
de cinco filas y cinco columnas por otro de tres filas y tres columnas -y dejando los
vértices exteriores al cuadrado- Nota: si lo intentas con un cuadrado de dos filas y dos
columnas junto con los cuatro vértices exteriores te darás cuenta que es imposible,
porque los vértices exteriores forman parte de 3 aristas -número impar- La razón de
esto es simple: cada vez que llegamos a un vértice por una arista tiene que haber otra
arista de salida del vértice. Con un número impar esto es imposible. Nota: una vez
que se sabe que hay un camino en el problema 47, se deja la búsqueda al cuidado del

56
lector

Solución del 49) No hay un recorrido, pues cada vertice del hexágono pertenece
a cinco aristas -número impar-. Para que no exista el camino es suficiente que un
vértice pertenezca a un número impar de aristas. Observación importante: el hecho
de que algunas aristas interiores se corten , no quiere decir que estos puntos de corte
sean vértices. Un grafo no es una cuestion de un dibujo sino que es cuestion de definir
de entrada los objetos que queremos que sean vértices y aristas. En este problema,
de entrada, se dice lo que son vertices y lo que son aristas. Podrı́amos haber dicho
de entrada que todos los cortes entre lineas interiores son vértices, pero entonces el
problema serı́a otro distinto. Al igual que los vértices, las aristas serı́an otras La defi-
nición matemática de grafo no tiene su fundamento en un dibujo. Es la siguiente: Un
grafo es un conjunto G -normalmente finito- y un conjunto formado por subconjuntos
de G de dos elementos. Estos subconjuntos de dos elementos van a ser las aristas y los
elementos del conjunto G, los vértices.

Ejemplo: el conjunto {1,2, 3, 4, 5, 6} lo tomamos como conjunto G o conjunto de


los vértices y como aristas tomamos los siguientes subconjuntos de G: {1, 4} {2, 4} {3,
4} {3, 5} {3, 6} {4, 5} {4, 6} Representamos el grafo en el siguiente dibujo -pero el
grafo es lo anterior no el dibujo- Obviamente podemos dibujar el grafo de más formas,
por ejemplo que no se corten las aristas. En el dibujo la arista {3, 5} se corta con
la arista {4, 6} Las aristas no tienen por qué ser rectas, también pueden ser curvas.
Intenta dibujar el anterior grafo sin que se cortan las aristas y utilizando sólo rectas
-se puede-

57
Solución del 50) Ahora sı́ es posible porque todos los vértices forman parte de un
número par de aristas. Cada vértice del pentágono forma parte de cuatro aristas. Un
camino es este: A–B–C–D–E–B–D–A–C–E–A -existen más caminos posibles-

Solución del 51) Este problema no es otra cosa que un grafo, donde los vertices
son los numeros del conjunto {1, 2, 3, 4, 5, 6, 7, 8, 9, 10, 11} y los datos de los pares de
números que pueden ser consecutivos en la sucesión son las aristas. Si representamos
todo esto en una figura se verá más claro.

No se puede construir la sucesión. La demostración es la siguiente: podemos asignar


a los vértices las letras A y B de tal modo que dos vértices que estén unidos mediante
una arista tengan letras distintas. Una asignación puede ser: al 2 la A, al 10 la B, al
3 la B, al 1 la A, al 11 la A, al 8 la B, al 6 la A, al 4 la A, al 5 la B, al 7 la A, y al
9 la B. Una sucesión como la que pide el problema es equivalente a un recorrido por
el grafo donde se pase una y solo una vez por cada vértice y se regrese al vértice de
partida. No es necesario pasar por todas las aristas. Supongamos que se comienza en el

58
2, que es una A. El siguiente vértice tiene que ser una B, puesto que dos vértices que se
comunican mediante una arista tienen letras distintas. El siguiente una A, el siguiente
una B, y asi sucesivamente. Es decir las letras A y B, van apareciendo una vez cada
una. Si existiese el recorrido tendrı́a la forma A-B-A-B-A-B-A-B-A-B-A. Pero hay que
regresar al 2 que es una A, y no se puede porque la última letra de las 11 anteriores
es una A y la siguiente a esta A tendrı́a que ser una B, pues en cada movimiento de
vértice a vértice hay que cambiar de letra. Por lo tanto no puedo regresar al 2. Hemos
puesto de ejemplo que se comienza en el 2, pero el anterior razonamiento es válido si
se comienza en cualquier otro vértice

Solución del 52) Si me fijo en la quinta columna observo que la multiplicación de


tres números comprendidos entre 1 y 9 solo puede ser 20 si estos números son el 5, el
4 y el 1. Observo ahora que en la casilla que ocupa la quinta columna y tercera fila
tiene que ir el 1. Esto es porque el 4 y el 5 no pueden ir ahı́. El 4, porque ningún
número multiplicado por 4 da 10. El 5 porque la suma que hay en la cuarta fila tendrı́a
que valer 2, pero esto sólo se puede conseguir con 1 + 1 y no vale porque se repite
el 1. Ahora demostramos que el 4 no puede ir en la quinta columna y quinta fila. Si
ponemos el 4 en esa posición, los dos primeros sumandos de la quinta fila tienen que
sumar 16. Esto sólo se consigue con 9 + 7 ó con 7 + 9 . 9+ 7 no puede ser porque la
multiplicación que hay en la tercera columna tendrı́a que valer 23, pero 23 es primo.
Si pongo 7 + 9, la multiplicación que hay en la tercera columna tiene que valer 21.
Este 21 sólo se puede obtener como 3 x 7 ó como 7 x 3. Si pongo 3 x 7, para que se
verifique la tercera fila, el primer elemento de dicha fila tiene que ser un 3, pero no
puedo ponerlo porque el 3 ya ha sido utilizado en otra casilla. Si pongo 7 x 3 , para
que se verifique la tercera fila, el primer elemento de dicha fila tiene que ser un 7,
pero no puedo ponerlo porque el 7 ya ha sido utilizado en otra casilla. La conclusión
de este razonamiento es que en la quinta fila y quinta columna no puede ir un 4. Por
consiguiente tiene que ir el 5 y de aquı́ se sigue que en la quinta columna y primera fila
tiene que ir el 4. Vamos a resolver ahora la quinta fila. Para los dos primeros sumandos
de la quinta tenemos cuatro posibilidades: 8 + 7, 7 + 8, 9 + 6, 6 + 9. En la quinta
fila y tercera columna no puede ir el 7, porque implicarı́a que la multiplicación que
hay en la tercera columna tendrı́a que valer 23, pero el 23 es primo. En la quinta fila
y tercera columna tampoco puede ir el 8, porque implicarı́a que la multiplicación que
hay en la tercera fila tendrı́a que valer 22, pero esto sólo se consigue con 11 x 2, y el
11 no se puede usar. El 9 tampoco puede ocupar la quinta fila y tercera columna -esto
ya lo habiamos analizado antes, vuelve a leerlo-. La conclusión para la qunta fila es
que tiene que estar ocupada ası́: 9 + 6 + 5. Ahora razonamos en la primera y tercera
columna. Sólo quedan colocar cuatro números: 2, 3, 7 y 8. Para que los dos sumandos

59
de la primera columna sumen 9, el primer sumando solo puede ser 2 ó 7, el segundo
sumando también tiene que ser 2 ó 7. Para que la multiplicación que hay en la tercera
columna valga 24, el primer factor sólo puede ser 3 ó 8, el segundo también tiene que
ser 3 ó 8. Si ponemos un 2 en el primer sumando de la primera fila, obliga a que el
segundo sumando de esta fila sea un 8 . También obliga que el segundo sumando de
la primera columna sea un 7 y sólo nos quedarı́a la casilla de la tercera fila y tercera
columna para el 3. Poniendo esto se ve que hemos encontrado una solución. La otra
posibilidad es poner un siete en el primer sumando de la primera fila. Esto obliga a
que el segundo sumando de la primera fila sea un 3. También obliga que el segundo
sumando de la primera columna sea un 2 y sólo nos quedarı́a la casilla de la tercera fila
y tercera columna para el 8. Poniendo esto se ve que hemos encontrado otra solució.n
Queda demostrado que hay dos y solo dos soluciones. También se han calculado las
dos soluciones

Solución del 53) Como cada cientı́fico puede intercambiar su trabajo con todos
excepto consigo mismo y con su compañero, al preguntar, Einstein escuchó las res-
puestas: 0, 1, 2, 3, 4, 5, 6, 7, 8, 9, 10, 11, 12. El que intercambió su trabajo con
12 es compañero del que lo intercambió con 0. Esto es fácil de deducir. Si el com-
pañero del que intercambió con 12 fuera por ejemplo el que intercambió con 1, el que
intercambió con 12 no habrı́a intercambiado el trabajo con el mismo, ni con el que
intercambió con 1, por ser su compañero, ni con el que intecambió con 0. Entonces no
habrı́a intercambiado su trabajo con 12. El que intercambió su trabajo con 11, es com-
pañero del que intercambió con 1. Esto también es fácil de deducir. Si el compañero del
que intercambió con 11 fuera por ejemplo el que intercambió con 2, el de 11 no habrı́a
intercambiado el trabajo consigo mismo, ni con el que intercambió con 2, por ser su
compañero. Tampoco con el que intercambió con 1, porque éste intercambió con el de
12 y tampoco con el que intercambió con 0. De esta forma no habrı́a intercambiado su
trabajo con 11. Ası́ sucesivamente se sigue deduciendo que el que intercambio con 10
es compañero del que intercambió con 2. El que intercambió con 9 es compañero del
que intercambió con 3. El que intercambió con 8 es compañero del que intercambió con
4. El que intercambió con 7 es compańero del que intercambió con 6. Por último el que
intercambió con 6 es compańero de otro que intercambió con 6. Esta última pareja es
Einstein y su compañero

Solución del 54) No se puede efectuar el recorrido porque San Sebastián y Alba-

60
cete forman parte de 3 aristas -número impar- . Como se vio en el acertijo 47, para
que se pueda efetuar el recorrido cada vértice tiene que formar parte de un número par
de aristas. Como se observa en la figura adjunta hay dos vertices - Madrid y Burgos-
unidos por más de una arista. Cuando esto ocurre se dice que tenemos un supergrafo.
Al igual que en los grafos, en los supergrafos, para poder efectuar un recorrido pasando
por todas las aristas sólo una vez y regresar al vértice de salida, cada vértice tiene que
formar parte de un número par de aristas. Si añadimos una carretera de San Sebastián
a Albacete, todos los vértices forman parte de un número par de aristas y entonces si
se puede efectuar el recorrido pasando sólo una vez por cada carretera y regresar a la
ciudad de partida. Compruébese trazando un recorrido

Solución del 55) Llamamos S a la suma y P al producto de los números. a) Si P fuera


producto de dos primos distintos, P = pq, entonces A sabrı́a la suma (S = p + q). b) Si
P fuera un primo al cuadrado, P = p2 , entonces A también sabrı́a la suma (S = 2p). c)
Si P fuera un primo al cubo, P = p3 , entonces A también sabrı́a la suma (S = p2 + p).
d) Si P fuera una potencia de un primo donde el exponente n es mayor que 3, P = pn ,
entonces habrı́a al menos dos sumas distintas, pn−1 + p es siempre estrictamente mayor
que pn−2 + p2 (porque pn−1 + p es mayor o igual que 2pn−2 + p y esto es estrictamente
mayor que pn−2 + p2 ). e) Si P fuera el producto de dos primos distintos por un número
mayor que 1, P = pqm donde p y q son primos y m es mayor que 1, entonces siempre
habrı́a al menos dos sumas distintas, p + qm y q + pm (de ser iguales p tendrı́a que
ser igual a q). Luego si A no sabe la suma es porque P no es pq, P no es p2 y P no
es p3 . Escribamos las primeras posiblidades, lo que descarta los casos S = 4, S = 5 y
S = 6: S = 4: 2 + 2 (P = 4) S = 5: 2 + 3 (P = 6) S = 6: 2 + 4 (P = 8) 3 + 3 (P = 9)

61
S = 7: 2 + 5 (P = 10) 3 + 4 (P = 12) S = 8: 2 + 6 (P = 12) 3 + 5 (P = 15) 4 + 4
(P = 16) S = 9: 2 + 7 (P = 14) 3 + 6 (P = 18) 4 + 5 (P = 20). Si la suma fuera 7, B
sabrı́a el producto con tener la suma porque la descomposición 2 + 5 no es posible por
lo que ha dicho A, y no podrı́a decir su primera frase. En todos los casos siguientes
B no sabe el producto aunque sepa la suma porque siempre hay al menos dos formas
que dan esa suma y que no son (p, q), (p, p) ó (p, p2 ). Ahora bien, si después de que B
diga que no sabe el producto, A dice que ya sabe la suma, es que el hecho de eliminar
el caso del 7 le ha servido para algo. El producto tenı́a que ser 10 ó 12. Como 10 es
un producto de dos primos distintos, tiene que ser 12. 12 sólo sale en las sumas 7 y 8.
Como acabamos de eliminar el 7, los números tienen que ser 2 y 6.

Solución del 56) La primera región está formada por los números {5, 6, 7, 1, 8,
9, 10, 17, 24, 31, 39, 47, 46, 45, 44, 36}. La segunda región por {11, 18, 25, 32, 40, 48,
56, 55, 54, 53, 52, 51, 43, 35, 28, 4} La tercera por {64, 63, 62, 61, 60, 59, 58, 50, 42,
34, 27, 20, 21, 3, 22, 29} y la cuarta región por los números restantes

Solución del 57) El dibujo de la izquierda es un mapa que no se puede colorear


usando dos colores pero sı́ usando tres. El de la derecha es un mapa que no se puede
colorear usando tres colores pero sı́ usando cuatro. La solución de la tercera pregun-
ta es: todos los mapas se pueden colorear usando cuatro colores, ası́ pues no hay un
mapa que no se pueda colorear usando cuatro colores pero sı́ usando cinco. Por muy
complicado que sea el mapa bastan cuatro colores. Este resultado es un teorema de
matemáticas que tuvo mucha polémica cuando se demostró por primera vez, pues la
demostración la realizó un ordenador. Muchos matemáticos se quejaron de que el or-
denador no les habı́a dado la razón por la que eran suficientes cuatro colores para
colorear cualquier mapa y no la consideraron una demostración matemática

62
Por más provincias que se añadan a cualquiera de los dos mapas, con cuatro colores
podréis colorearlos consiguiendo que dos provincias limı́trofes tengan distinto color

Solución del 58) La estrategia ganadora es para el jugador que coloca pieza en
segundo lugar. Tiene que colocar sus piezas en las posiciones simétricas, respecto del
centro del tablero, de las que vaya ocupando el primer jugador. La razón es simple.
Siempre que el primer jugador pueda colocar una pieza, seguro que el segundo también
puede hacerlo y cuando el primer jugador ya no pueda colocar pieza entonces perderá.
No se puede empatar por la siguiente razón: si pintamos las casillas del tablero en
blanco y negro -justo como un ajedrez- al quitar las dos esquinas de una diagonal,
tendremos dos casillas más de un color que del otro. Por ejemplo 32 blancas y 30
negras. Cada vez que se coloca una pieza, se tapa una casilla blanca y otra negra.
Si se pudiesen colocar 31 fichas para llenar todo el tablero, no habrı́a un perdedor y
habrı́amos tapado 31 blancas y 31 negras, pero esto no se puede porque sólo hay 30
casillas negras. En la siguiente figura se muestra cuáles son las posiciones simétricas
respecto del centro del tablero. En las posiciones simétricas se ha escrito el mismo
número

1 2 3 4 5 6 7 8
9 10 11 12 13 14 15 16
17 18 19 20 21 22 23 24
25 26 27 28 29 30 31 32
32 31 30 29 28 27 26 25
24 23 22 21 20 19 18 17
16 15 14 13 12 11 10 9
8 7 6 5 4 3 2 1

Solución del 59) En el masyu no hay que dar una solución pues no se pregunta

63
nada

Solución del 60) Al guardia hay que darle una forma de verificar una contraseña
pero no de averiguarla. De esta manera puede dejar pasar al espı́a sin saber cuál es la
contraseña. Al espı́a, su paı́s le da un número de 100 cifras y una máquina para que
efectue el siguiente cálculo: eleva su número de 100 cifras al cuadrado y se queda con
las 100 cifras que ocupan la posición central del resultado. El paı́s en cuestión ha dado
al guardia estas 100 cifras centrales. De esta forma el espı́a efectúa el cálculo sin que
el guardia le vea, y le muestra el resultado. Al coincidir con las 100 cifras que tiene el
guardia, puede pasar la frontera. Es prácticamente imposible que el guardia averigue
cuál es el número de 100 cifras que el paı́s ha dado al espı́a. Aunque el guardia sepa que
el cálculo consiste en elevar al cuadrado un número de 100 cifras y luego quedarse con
las 100 cifras centrales del resultado, es prácticamente imposible averiguar el número
del espı́a. Si el guardia quisiera averiguarlo tendrı́a que ir prácticamente número por
número hasta comprobarlos todos. El tiempo que emplearı́a serı́a gigantesco, pues hay
una cantidad enorme de números con 100 cifras. Actualmente los matemáticos cree-
mos que existen problemas que se pueden verificar en tiempo razonable pero no se
pueden resolver en tiempo razonable. Aquı́ la palabra verificar significa que se puede
comprobar si un n—’umero es o no solución. La palabra resolver significa que se pue-
de encontrar la solución directamente a través de algún algoritmo -programa- en un
tiempo razonable. Este problema del espı́a se sabe que se puede verificar en tiempo
razonable, simplemente haciendo el cuadrado y quedándose con las cifras centrales del
resultado, se puede saber si coinciden o no con las 100 cifras del guardia. Pero se cree
que no se puede resolver en tiempo razonable. Escribo ”se cree”porque es un problema
que se investiga en la actualidad. Es uno de los problemas del milenio: saber si existe
algún problema que se puede verificar en tiempo razonable pero no se puede resolver en
tiempo razonable. Si todos los problemas que se pueden verificar en tiempo razonable,
se pudiesen resolver en tiempo razonable, la criptografı́a - encargada de los protocolos
de seguridad- quedarı́a seriamente afectada

Solución del 61) En este problema no hay que contestar nada, pues no se pide
una estrategia ganadora

Solución del 62) Podemos colocar las letras A y B en las baldosas del suelo de

64
tal modo que siempre que nos movamos en horizontal o en vertical un paso cambie-
mos de letra. Es decir la primera fila seria: ABABABAB; la segunda BABABABA; la
tercera ABABABAB y asi sucesivamente. Supongamos que comenzamos el recorrido
en una A. Los pasos tienen la forma: BABABABA....pues cada vez que doy un paso
cambia la letra. El penúltimo paso tiene que ser llegar a una casilla adyacente a la de
salida, por lo tanto tiene que ser llegar a una B. El último paso tiene que ser una A.
Luego si comienzo con B y termino con A, hay un número par de pasos. En la figura
adjunta se muestran las letras sobre las baldosas.

A B A B A B A B
B A B A B A B A
A B A B A B A B
B A B A B A B A
A B A B A B A B
B A B A B A B A
A B A B A B A B
B A B A B A B A

Solución del 63) En el acertijo 5 vimos la sucesión de Fibonacci: 1, 1, 2, 3, 5, 8,


13, 21, 34, 55....cada término se obtiene sumando los dos anteriores. Si el número ini-
cial de fichas es un número de la sucesión de Fibonacci, el segundo jugador puede ganar
siempre. Si no es un número de Fibonacci el jugador que comienza retirando fichas pue-
de ganar siempre. Veámoslo con dos ejemplos. Supongamos que se parte de 20 fichas.
Como 20 no es número de Fibonacci, si el primer jugador actúa con inteligencia, es
seguro que ganará la partida. Para determinar la estrategia vencedora, se descompone
el número 20 en suma de números de Fibonacci, comenzando por el mayor posible, 13,
sumando después el máximo posible, 5, y después el siguiente, 2. Ası́ que 20 = 13 + 5
+ 2 es la descomposición buscada. Todo entero positivo puede ser expresado de forma
única como suma de nḿeros de Fibonacci; tal descomposición no podrá nunca contener
números de Fibonacci consecutivos. Los números de Fibonacci quedan expresados por
un solo número: ellos mismos. El último número, 2, es el número de piezas que debe

65
retirar el primer jugador para ganar. El segundo jugador queda imposibilitado, por las
reglas del juego, para tomar más del doble de 2; por consiguiente, no puede reducir
el monton (que tiene ahora 18 fichas) al nḿero de Fibonacci más cercano, que es 13.
Supongamos que decide retirar cuatro piezas; la pila contendrá entonces 14 piezas,
nḿero que se expresa como 13 + 1 en suma de dos números de Fibonacci, y el primer
jugador deberá entonces tomar 1 pieza. Prosiguiendo con esta estrategia, es seguro que
logrará tomar la última ficha, y con ella, ganar la partida. Si el número de inicial de
piezas fuera número de Fibonacci, por ejemplo, 144, es seguro que el segundo jugador
podrá ganar. Es verdad que el primer jugador puede retirar 55 piezas, dejando 89, que
es el siguiente número de Fibonacci, pero entonces el segundo jugador puede ganar
inmediatamente, retirando lı́citamente las 89 piezas restantes, pues 89 es menor que el
doble de 55. El primer jugador se ve entonces obligado a dejar un número de piezas no
perteneciente a la sucesión de Fibonacci, y el segundo jugador consigue ganar aplican-
do la estrategia que expliqué antes -descomponer en suma de números de Fibonacci y
tomar el más pequeño

Solución del 64) Si se puede, Obsérvese la figura adjunta. Además cada trozo tiene
la misma forma que el original

Solución del 65) No es posible recorrer los 125 cubitos. Se puede poner una A o

66
una B a cada cubito de tal forma que dos cubitos adyacentes tengan letras distintas.
Si dividimos el cubo grande en cinco pisos de altura. En el primero, tercero y quinto
piso tenemos 13 veces la A y 12 veces la B. En el segundo y cuarto piso 12 veces la A
y 13 veces la B. En total tenemos 63 veces la A y 62 veces la B. Al cubito de inicio
le corresponde una B. Si se pudiese efectuar el recorrido serı́a de la forma B-A-B-A-
B-A-B-A-B-A......B-A-B. Tiene que terminar en B, porque hay un número impar de
cubitos. Pero entonces, en la anterior sucesión tendrı́amos 123 veces la B y 122 veces
la A y esto es imposible porque la distribución de las letras en el cubo grande es: 123
veces la A y 122 veces la B

Solución del 66) Para el impar 1 construyo el conjunto {1, 2, 4, 8, 16, 32, 64, 128}.
Para el impar 3 construyo {3, 6, 12, 24, 48, 96, 192} Para el impar 5 construyo {5,
10, 20, 40, 80, 160}. Hago esto con los 100 números impares que hay en los primeros
200 números naturales. La forma de construirlos es comenzar con el número impar e
ir multiplicando siempre por 2. De esta forma es claro que dos números cualesquiera
que pertenezcan al mismo conjunto son uno múltiplo del otro. Cualquier número entre
1 y 200 pertenece a un conjunto de estos y sólo a uno. Como tengo 100 conjuntos,
y los 101 números que he elegido al azar están distribuidos en estos 100 conjuntos,
necesariamente en alguno de estos conjuntos tiene que haber al menos dos números.
Por consiguiente, uno múltiplo del otro. ¿Por qué todos los números pares están en
alguno de estos conjuntos? Muy fácil, partiendo por ejemplo del 100, divido por 2 y
obtengo 50. Vuelvo a dividir por 2 y obtengo 25. Ya no puedo seguir dividiendo por 2
porque 25 es impar. Esto quiere decir que el 100 pertenece al conjunto {25, 50, 100,
200} Es decir partiendo de cualquier número par si hago divisiones sucesivas entre 2,
en algún momento obtengo un número impar y ya no puedo seguir dividiendo entre 2.
Entonces ese número par pertenece al conjunto que comienza con el impar obtenido.
Para demostrar la unicidad, es decir, que un número par solo pertenece a un conjunto
de los anteriores, basta considerar lo siguiente. Si tuviese un numero par p = m mul-
tiplicado por( 2 elevado a r ) = n multiplicado por (2 elevado a s) con m y n impares
distintos, entonces los exponentes r y s también serı́an distintos. Supongamos que r
es mayor que s. Pasando dividiendo (2 elevado a s) al primer término y m al segundo
término, queda: (2 elevado a r) / (2 elevado a s) = n/m El número de la izquierda es
un número par y el de la derecha no, porque al dividir dos impares nunca se obtiene
par. Se obtiene o un entero impar o una fracción que no es un número entero. Por lo
tanto es imposible que un número par pertenezca a dos conjuntos distintos

Solución del 67) Esta es la numeración de los dados: A(4 4 4 5 5 5), B(1 1 6 6
6 6), C(2 2 2 7 7 7) y D(3 3 3 3 8 8). Tenemos que si juegan A y B, B va a ganar 2/3

67
de las veces (cuando B muestre un 6). Si juegan B y C, C va a ganar los 2/3 de las
veces (cuando B muestre un 1 y la mitad de las veces que muestre un 6). Si juegan C
y D, D va a ganar los 2/3 de las veces (cuando D muestre un 8 y la mitad de las veces
que muestre un 3). Y si juegan D y A, A va a ganar los 2/3 de las veces (cuando D
muestre un 3). De esta forma se cierra el cı́rculo y el amigo perderá siempre 2 de cada
3 tiradas. Si Luis elige A, el matemático elige B. Si Luis elige B, el matemático elige
C. Si Luis elige C, el matemático elige D. Y si Luis elige D, el matemático elige A

Solución del 68) Si no se hubiese equivocado, cada letra habrı́a parecido en la suce-
sión dos veces, una vez en un lugar par y la otra en uno impar. En la lectura errónea
la W aparece en los lugares 12 y 20 de la sucesión - los dos pares-. La N aparece en
los lugares 9 y 19 -los dos impares-. Por lo tanto el error lo tuvo en estas dos letras.
Esto, se fundamenta en un teorema de matemáticas para curvas

68
TEMA 3: ¿QUÉ SON LAS MATEMÁTICAS?

El objetivo de este capı́tulo es describir las matemáticas, respondiendo preguntas


como: ¿Qué son los axiomas y los teoremas en matemáticas? ¿En Geometrı́a, es ne-
cesario dar una definición de punto, recta y plano? ¿En Álgebra es necesario dar una
definición de conjunto? ¿Cómo se construye el edificio de las matemáticas? ¿De qué ele-
mentos se compone una demostración en matemáticas? ¿Qué papel juega la lógica en
las matemáticas?

3.1 AXIOMAS Y TEOREMAS

El corazón de la actividad matemática es la demostración. A partir de unas proposi-


ciones iniciales denominadas axiomas, se demuestran, mediante reglas de deducción,
proposiciones más complejas, denominadas teoremas. El término proposición hay que
entenderlo como una frase de caracter matemático. Ejemplos: el 5 es un número primo;
el 8 no es un número primo. A estas frases matemáticas o proposiciones también se
les denomina fórmulas, es decir, una fórmula en lógica matemática tiene un sentido
más amplio que las fórmulas matemáticas que usamos a diario. Aunque estas fórmulas
matemáticas que usamos a diario también están incluidas en el concepto de fórmula en
lógica matemática. Los axiomas constituyen los cimientos del edificio matemático, no
se pueden demostrar, y son aceptados por todos los matemáticos. Un teorema es un
enunciado matemático que admite una demostración y debe demostrarse, es decir, que
a diferencia de los axiomas, se puede demostrar. Veremos un ejemplo de sistema de
axiomas en matemáticas: la axiomática de la teorı́a de conjuntos. Después veremos co-
mo se pueden generar teoremas. Para quienes estén interesados en lógica matemática,
al final de este capitulo hay un apartado que explica con rigor los elementos que pue-
den usarse en una demostración matemática. El apartado se titula: Sistemas Formales,
y puede servir de guı́a a quienes quieran ampliar este libro con la lógica matemáti-
ca. Naturalmente, esta ampliación debe realizarse con algún texto riguroso de lógica
matemática. La teorı́a de conjuntos que vamos a considerar se debe a Zermelo. Ante-
riormente a Zermelo, se produjeron contradicciones en la teorı́a de conjuntos por no
haber tenido cuidado a la hora de enunciar los axiomas. La contradicción más famosa
fue la paradoja de Bertrand Russell. En la teorı́a de Zermelo no se define explı́cita-
mente lo que es un conjunto. Intuitivamente hay que entender un conjunto como una
colección de objetos, pero sólo existen los conjuntos que se pueden obtener mediante los
axiomas o posteriormente mediante los teoremas. Lo importante de este apartado no
es comprender los axiomas a la perfección, sino ver como se van generando teoremas.
En definitiva, ver como se va construyendo poco a poco el edificio de las matemáticas.
Posteriormente, a la teoria de conjuntos de Zermelo se le añadieron nuevos axiomas:
el axioma de reemplazo o esquema de reemplazo, el axioma de elección y el axioma

69
de regularidad, pero los axiomas iniciales de Zermelo son suficientes para los objetivos
de este capı́tulo. Los axiomas se pueden enunciar con más rigor del que se va a hacer
aquı́, pero eso llevarı́a probablemente a no entenderlos. Básicamente los axiomas son
los que siguen. Los conjuntos se pueden representar con un sı́mbolo a la izquierda de
los elementos y otro a la derecha. Estos sı́mbolos se denominan llaves de conjunto.
Para representar el conjunto cuyos elementos son el 1 y el 2 lo hacemos asi: {1, 2}

1) El axioma de extensionalidad: Dados dos conjuntos X e Y, si todo elemento


de X es un elemento de Y y todo elemento de Y es un elemento de X, entonces el
conjunto X es igual al conjunto Y y lo denotamos por X = Y. Es decir, si dos conjun-
tos X e Y tienen los mismos elementos entonces son iguales, X = Y. Este axioma nos
está diciendo que un conjunto está determinado por sus elementos. Nota: el axioma
solo dice que si dos conjuntos X e Y tienen los mismos elementos, entonces son iguales.
Pero si los conjuntos son iguales, X= Y ¿tendrán los mismos elementos? La respuesta
es afirmativa, y se basa en un principio lógico: si X = Y entonces todo lo que vale para
X, vale para Y; y todo lo que vale para Y vale para X. En concreto, si X contiene a un
elemento c, Y tambien contiene a c , y si Y contiene a un elemento c, X también contie-
ne a c. Por consiguiente tienen los mismos elementos. Esta nota que acabo de escribir
no es superflua, aunque pueda parecerlo. Supongamos que digo: si n es un múltiplo de
4 entonces n es par. Esto es cierto, pero no he dicho nada sobre: si n es par entonces
n es múltiplo de 4. Esta última implicación no es cierta porque hay números que son
pares y no son múltiplos de 4, por ejemplo el 10. Si n es múltiplo de 4, entonces n es
par; esto es lo que afirma el axioma -si X e Y tienen los mismos elementos entonces
son iguales- pero no dice nada de la implicación al revés, es decir, si dos conjuntos son
iguales entonces tienen los mismos elementos -si n es par entonces n es múltiplo de 4 -.
Como el axioma no dice nada de esto último, he tenido que demostrarlo, basándome
en un principio lógico. En el ejemplo de los números, la segunda implicación no es
cierta, pero en el caso de los conjuntos sı́ lo es. Lo es porque lo he demostrado usando
un principio lógico, no porque tenga obligatoriamente que serlo. Por esto, la nota que
he añadido no es superflua

El conjunto {1, 2} no es igual al conjunto {1, 2, 3} pues el 3 es un elemento del


segundo conjunto que no pertenece al primero. Esta desigualdad de los dos conjuntos
no se deduce del axioma del extensionalidad, sino de la implicación que he demostrado
usando el principio lógico -si dos conjuntos son iguales entonces tienen los mismos
elementos- decir esto, es equivalente a decir: si dos conjuntos no tienen los mismos
elementos entonces no son iguales. Esto último es exactamente lo que he aplicado. En
cambio, si llamo A al conjunto {1, 2} y B al conjunto {1, 2}, ahora sı́ puedo aplicar
el axioma de extensionalidad y lo aplico de la siguiente manera: como cada elemen-
to de A es un elemento de B y cada elemento de B es un elemento de A, entonces A = B

70
2) El axioma del conjunto vacio: Existe un conjunto que no tiene elementos.
Este axioma habla de la existencia de un conjunto, pero no de que sea único . No hace
falta que lo haga, pues no pueden existir dos conjuntos distintos Y, Z que no tengan
elementos. Supongamos que existen dos conjuntos distintos Y y Z, que no tienen ele-
mentos. Entonces, por ser distintos, tiene que ocurrir que exista un elemento de Y que
no pertenezca a Z o que exista un elemento de Z que no pertenezca a Y. Pero esto es
absurdo porque tanto Y como Z no tienen elementos. Por consiguiente existe un único
conjunto que no tiene elementos. A este único conjunto le denominamos el conjunto
vacı́o y su notación es: ∅

3) El axioma del par: Dados dos conjuntos X e Y existe un conjunto cuyos úni-
cos elementos son X e Y. Nuevamente el conjuto al que se refiere el axioma es único.
pues no pueden existir dos conjuntos Z y W cuyos únicos elementos sean X e Y, y ser
distintos. A este conjunto se le representa por: {X, Y}

4) El axioma de la unión: Para cada conjunto X, existe un conjunto formado


por los elementos de los elementos de X . Este axioma habla de la existencia, pero
no de unicidad. Para ver que el conjunto al que se refiere el axioma es único basta
comprender que no pueden existir dos conjuntos Y y Z formado por los elementos de
los elementos de X y ser distintos. Este conjunto único se denota por: ∪X. El sı́mbolo
∪ es el sı́mbolo de la unión. En el siguiente dibujo he considerado que X consta de tres
conjuntos, pero el axioma permite que X conste de todos los conjuntos que se quieran:
1, 2, 3, 4...incluso infinitos...El primer dibujo es el conjunto X al que voy a calcular la
unión y el segundo dibujo es el conjunto unión

71
Nota: Antes de enunciar el siguiente axioma veamos lo que es un subconjunto de un
conjunto. Un subconjunto Y del conjunto X, es un conjunto Y tal que cada elemento
de Y es un elemento de X. Ejemplo: el conjunto Y: {1, 2, 3} es un subconjunto del
conjunto X: {1, 2, 3, 4, 5} porque cada elemento de Y es un elemento de X

5) El axioma de las partes: Para cada conjunto X existe un conjunto formado


por los subconjuntos de X. Este conjunto es único porque no pueden existir dos con-
juntos formados por los subconjuntos de X y que estos conjuntos sean distintos. Al
único conjunto formado por los subconjuntos de X se le representa por: P(X).

Ejemplo: Sea X el conjunto: {1, 2, 3}, entonces el conjunto P(X) es: {{1, 2, 3},
{1,2}, {1, 3}, {2, 3}, {1}, {2}, {3}, ∅ } Nota: el conjunto vacı́o ∅ es un subconjunto
de cualquier conjunto X, porque no existe un elemento del conjunto vacı́o que no per-
tenezca a X

6) El axioma de especificación o esquema de especificación: Para cada


conjunto X y cada propiedad P, existe un conjunto Y formado por los elementos de
X que cumplen la propiedad P. Se llama de especificación porque especifica los ele-
mentos de X que van a pertenecer al nuevo conjunto Y. Ejemplo: Consideremos el
conjunto X: {1, 2, 3, 4, 5, 6} y la propiedad P: ser número par. Existe un conjunto Y
formado por los elementos de X que son pares. Nuevamente el conjunto Y al que se
refiere el axioma es único, pues no pueden existir dos conjuntos Y y Z formados por
los elementos de X que cumplan P y que estos conjuntos sean distintos. En el ejemplo
anterior, el unico conjunto formado por los elementos de X que son pares es Y: {2, 4, 6}

Ahora vamos a demostrar un teorema a partir de los axiomas que ya conocemos.


Es el teorema de la unión de dos conjuntos X e Y

Teorema: Dados dos conjuntos X e Y existe un único conjunto Z formado por los
elementos de X y los elementos de Y. En un lenguaje más matemático el teorema afir-
ma: Para cada dos conjuntos X e Y existe un único conjunto Z tal que z es un elemento
de Z si y solo si z es un elemento de X o z es un elemento de Y. Nota: poniendo la
disyunción o, lo que queremos es que el conjunto Z conste de todos los elementos de
X y todos los elementos de Y. A este único conjunto lo denotaremos por X ∪ Y

Demostración:
Sean X e Y dos conjuntos, aplicando el axioma del par, existe un conjunto W cuyos
únicos elementos son X e Y. Es decir, W: {X, Y} Lo que hemos hecho es meter a los
conjuntos X e Y en un conjunto W. Esto es legitimo por el axioma del par - axioma 3-.
Ahora aplico al conjunto W, el axioma de la unión -axioma4-. Aplicando este axioma
obtengo que existe un conjunto Z formado por los elementos de los elementos de W, es

72
decir, existe un conjunto Z formado por los elementos de X y por los elementos de Y.
Esto era precisamente lo que queriamos demostrar. Para demostrar la unicidad basta
ver que no pueden existir dos conjuntos R y S formados por los elementos de X y los
elementos de Y y que estos conjuntos R y S sean distintos. Nota: era fundamental
meter a los conjuntos X e Y en un conjunto W para poder aplicar el axioma de la
unión a W -axioma 4- En el siguiente dibujo se ve muy bien los dos pasos que he dado
en la demostración.

7) El axioma del infinito: Existe un conjunto W que cumple lo siguiente:

1) El conjunto vacı́o es un elemento de W

2) Si X es un elemento de W, entonces, X unión con el conjunto que tiene como


único elemento el X, también es un elemento de W.

Para expresar que a es un elemento del conjunto A, se utiliza el sı́mbolo de per-


tenencia: a ∈ A, y se lee a pertenece a A. En un lenguaje más matemático las dos
propiedades del axioma se expresan como sigue:

1) ∅ ∈ W

2) Si X ∈ W, entonces, X ∪ {X} ∈ W

73
Nota: En el axioma del par -axioma 3 - los conjuntos X e Y no tienen por qué ser
distintos, también pueden ser iguales. Si son iguales, obtenemos qe el conjunto {X, Y}
es igual al {X}. Este último conjunto es el que se ha usado en la segunda propiedad
del axioma del infinito. Veamos esto con un ejemplo: en el axioma del par, se puede
tomar como conjunto X: {1, 2} y como conjunto Y: {1, 2}, entonces, el conjunto cuyos
únicos elementos son el X y el Y es igual al conjunto cuyo unico elemento es el X. Es
decir el conjunto {{1, 2}, {1, 2}} = {{1, 2}} Por consiguiente, del axioma del par se
deduce, que dado un conjunto X, existe un conjunto cuyo único elemento es el X, y lo
escribimos como: {X}

Del axioma del infinito se desprende que existe un conjunto W con infinitos ele-
mentos. Para ver esto se comienza con el conjunto vacı́o ∅. El ∅ es un elemento de W
-eso dice la propiedad 1 del axioma del infinito-. Aplicamos ahora la propiedad 2 al
conjunto vacı́o, haciendo la unión ∅ ∪ {∅} y obtenemos {∅}, que es un elemento de W.
Volvemos a aplicar la propiedad 2 a {∅}, haciendo la unión {∅} ∪ {{∅}} y obtenemos
{∅, {∅}}, que es un elemento de W. Asi sucesivamente vamos obteniendo una susesión
infinita de elementos de W.

Hasta aquı́ los 7 axiomas de Zermelo. Quien haya estudiado alguna vez los con-
juntos habrá observado que la intersección de conjuntos no figura en los axiomas de
Zermelo. No figura porque se puede deducir como un teorema de la teorı́a de Zermelo,
es decir, se va a poder demostrar. Para quien no haya estudiado nunca los conjuntos,
la intersección de dos conjuntos X e Y simplemente es el conjunto formado por los
elementos que pertenecen a X y pertenecen a Y, y se escribe como: X ∩ Y Ejemplo:
Sea X el conjunto {1, 2, 3, 4} e Y el conjunto {2 ,4, 6, 8}, X ∩ Y = {2, 4}

El teorema de la intersección de dos conjuntos es el que sigue

Teorema: Dados dos conjuntos X e Y existe un único conjunto Z, tal que, u es un


elemento de Z, si y solo si, u es un elemento de X y u es un elemento de Y. A este
conjunto se le denomina X intersección Y y se denota por X ∩ Y

Demostración:
Aplicamos el axioma de especificacion al conjunto X y a la propiedad P: ser un elemen-
to de Y. Existe un conjunto Z formado por los elementos de X que cumplen P. Es decir,
un conjunto formado por los elementos de X que pertenecen a Y. Este conjunto Z no es
otra cosa que la intersección de X e Y, pues sus elementos pertenecen a X y pertenecen
a Y. Para ver la unicidad basta entender que no puede haber dos conjuntos Z y W
formados por los elementos comunes de X y de Y, y estos conjuntos Z y W ser distintos.

NOTA IMPORTANTE: En el enunciado del teorema he escrito ”si y sólo si”,

74
esto significa que si u es un elemento de X y de Y, entonces pertenece al conjunto Z,
y si u no es un elemento de X y de Y, entonces u no pertenece a Z. También se puede
interpretar de esta forma equivalente: si u es un elemento de X y de Y, entonces u
pertenece al conjunto Z, y si u pertenece al conjunto Z, entonces u pertenece al con-
junto X y al conjunto Y. Si no se pone el: si y sólo si, y solo se pone: Existe un único
conjunto Z, tal que, si u es un elemento de X y de Y entonces pertenece a Z, se puede
construir un conjunto Z, tal que, si u es un elemento de X y de Y entonces pertenece
a Z, pero al conjunto Z se le puede añadir algún elemento más que no perteneciera a
X y a Y, pues el enunciado solo me dice que si u es un elemento de X y de Y lo tengo
que meter en el conjunto Z, pero no me dice que tengo que hacer si u no pertenece a X
y a Y. Al añadir: sólo si, el conjunto Z no puede tener elementos que no pertenezcan
a X y a Y. En los siguiente dibujso se analiza el: si y sólo si

- Si u es un elemento de X y un elemento de Y entonces u es un elemento de Z.¿Es


verdadero? sı́, porque los elementos comunes de X y de Y son el 2 y el 3 y pertenecen
los dos a Z
- Si u es un elemento de Z entonces u es un elemento de X y un elemento de Y. ¿Es
verdadero? no, porque el 7 es un elemento de Z y no es un elemento común de X y de
Y
- u es un elemento de Z, si y sólo si, u es un elemento común de X y de Z. ¿Es verda-
dero? no, porque el enunciado dice: sólo si u es un elemento común de X y de Y, u es
un elemento de Z, y el 7 no es un elemento comun de X e Y, y pertenece a Z.
- Si u es un elemento de X y de Y entonces u es un elemento de W. ¿Es verdadero? sı́,
porque los elementos comunes de X y de Y son el 2 y el 3 y pertenecen los dos a W
- Si u es un elemento de W, entonces u es un elemento de X y un elemento de Y.

75
¿Es verdadero? sı́, porque los dos elementos de W, el 2 y el 3, son elementos de X y
elementos de Y
- u es un elemento de W, si y sólo si, u es un elemento común de X y de Y. ¿Es
verdadero? sı́, porque los elementos comunes de X y de Y pertenecen a W, y los que
no son comunes no pertenecen .

Casi todo lo que hemos dicho en este capı́tulo deberı́a haber llevado el: si y solo
si. No se ha puesto para facilitar su comprensión. Por ejemplo, el axioma 5 de las
partes, poniendo:si y solo si, tendrı́a la siguiente forma: para cada conjunto X existe
un conjunto Y, tal que y es un elemento de Y, si y solo si, y es un subconjunto de X.
En el teorema que demostramos anteriormente a éste, sobre la unión de dos conjuntos,
también se escribió ”si y solo si”. Volveremos a analizar el: si y solo si, cuando veamos
la base de la lógica matemática
Hemos demostrado la existencia del conjunto intersección de dos conjuntos X e
Y. ¿Existirá también el conjunto intersección cuando haga la intersección de muchos
conjuntos? Es decir, ¿sucederá algo análogo al axioma de la unión? -vuelve a leer el
axioma 4- En este axioma, si me daban un conjunto con muchos conjuntos: 1, 2, 3, 4,
5, ...incluso infinitos conjuntos, existı́a la unión de todos esos conjuntos. Lo único que
se requerı́a es que esos conjuntos fueran los elementos de un conjunto X, es decir, que
no fueran conjuntos aislados. La intersección de muchos conjuntos también existe, lo
que ocurre es que no va a ser un axioma de la teorı́a de conjuntos de Zermelo, sino un
teorema. Es decir, algo que se puede demostrar. Por eso no hace falta incluirlo como
un axioma. El teorema es el que sigue

Teorema: Para cada conjunto X existe un único conjunto Y, tal que, y es un ele-
mento de Y, si y solo si, y es un elemento de cada elemento de X. A este conjunto
único lo representaremos mediante ∩X

Demostración
Por el axioma 4, sabemos que existe un conjunto unión de todos los elementos de X.
También vimos que este conjunto era único y lo representabamos mediante ∪X. Aplico
a este conjunto el axioma de especificación, con la propiedad: ser un elemento de todos
los elementos de X. De esta forma, existe un conjunto Y formado por los elementos de
∪X que cumplen P. Es decir, que pertenecen a todos los elementos de X. Este conjunto
es el que buscamos, pues quiero que sus elementos pertenezcan a todos los elementos de
X. Para ver la unicidad del conjunto intersección basta entender que no pueden existir
dos conjuntos Y y Z, formados por los elementos comunes de todos los elementos de
X y estos conjuntos Y y Z ser distintos. En el siguiente dibujo se ve lo que significa la
intersección de muchos conjuntos,con la condición de que estos conjuntos sean
los elementos de otro conjunto X, es decir, que no sean conjuntos aislados

76
Ası́ es como se va avanzando en la construcción del edificio matemático. A través
de unas proposiciones iniciales denominadas axiomas se van generando otras propo-
siciones denominadas teoremas. Todas las proposiciones que no figuren en la lista de
axiomas hay que intentar demostrarlas o refutarlas. Refutarlas significa demostrar
justo lo contrario de la proposición, o lo que es lo mismo demostrar la proposición
contraria -por lo tanto no deja de ser otra demostración- y si se consigue demostrar
será también un teorema. Veamos un ejemplo. Si digo: existe una fracción m/n tal que
al elevarla al cuadrado se obtiene una fracción cuyo numerador es el doble que el de-
nominador. La proposicion contraria es: no existe una fracción m/n tal que al elevarla
al cuadrado se obtenga una fracción cuyo numerador es el doble que el denominador.
En este ejemplo, el teorema, es la proposición contraria: no existe una fracción m/n
tal que al elevarla al cuadrado se obtenga una fracción cuyo numerador es el doble que
el denominador. Esto último se puede demostrar. Un teorema, es pues, una frase de
carácter matemático que se puede demostrar.

En matemáticas no sólo se avanza demostrando teoremas, también se avanza in-


tuyendo nuevos conceptos e ideas. Esta creatividad hay que plasmarla en definiciones
precisas. Estas definiciones generarán nuevos teoremas. Un ejemplo de lo anterior es la
definición de par ordenado. Hasta ahora dados dos conjuntos X, Y podiamos meterlos
como elementos de un conjunto {X, Y}, creando asi un nuevo conjunto. El orden en el
que aparezcan X e Y no importa. El conjunto {Y, X} es igual al {X, Y} porque los dos
contienen los mismos elementos. Pero ahora quiero crear un objeto en el que sı́ importe
el orden. Este objeto va a ser el par ordenado. Antes de definir el par ordenado quiero
hacer una observación sobre el tercer axioma. Cuando en el tercer axioma se dice:

77
Para cada dos conjuntos X e Y existe un conjunto cuyos elementos son X e Y, estos
conjuntos no tienen por qué ser iguales. De este modo si considero los dos conjuntos
iguales: X, X, el axioma me dice que existe un conjunto formado por X y X, es decir,
{X, X} pero esto se puede simplificar en {X}. De este modo dados dos conjuntos X, Y,
existen los conjuntos {X} y {X, Y}. Si volvemos a aplicar el axioma a estos conjuntos,
existirá tambien el conjunto que los tiene por elementos, es decir, {{X}, {X, Y}}. Ya
tenemos todos los ingredientes para la nueva definición

Definicion: Dados dos conjuntos X e Y, defino el par ordenado X, Y como: {{X},


{X, Y}}. El par ordenado X, Y se representa mediante: (X, Y)

Con esta definición he conseguido que el par (X, Y ) y el par (Y, X) no sean iguales
cuando X e Y no son iguales, pues el par (Y, X) es: {{Y}, {Y, X}}. Las definiciones
nuevas deben tener alguna utilidad. Deben ir dirigidas a algún objetivo. La definición
de par ordenado me lleva inmediatamente a la construcción del conjunto: producto
cartesiano de dos conjuntos A y B, denotado por A × B. Intuitivamente, el producto
cartesiano A x B es el conjunto formado por los pares (a, b) tal que a pertenece al con-
junto A y b al conjunto B. El rigor matemático no permite definir alegremente A × B
como acabo de hacerlo. Se necesita demostrar la existencia de tal conjunto -y después
la unicidad-. Aunque las matemáticas tengan mucho de creatividad y de inspiración,
nunca se debe olvidar el rigor matemático y es necesario demostrar todo lo nuevo de
una forma inapelable, que no quepa lugar a la duda. Lo que sigue es una demostración
de la existencia y unicidad del conjunto producto cartesiano de dos conjuntos X e Y

Teorema: Dados dos conjuntos A y B existe un único conjunto formado por los
pares (a,b) tal que a es un elemento de A y b es un elemento de B.

Demostración
Primero se considera el conjunto unión A ∪ B. Ahora considero el conjunto formado
por los subconjuntos o partes de A ∪ B. Se escribe ası́: P(A ∪ B). Ahora nuevamen-
te consideramos el conjunto formado por todos los subconjuntos de P(A ∪ B) con
lo que tengo el conjunto P(P(A ∪ B)). Ahora aplico el axioma de especificación al
anterior conjunto P(P(A ∪ B)), con la propiedad P(X): Existen un elemento a de A
y un elemento b de B tal que X ={ {a}, {a,b}} . Es decir tomo los elementos X de
P(P(A ∪ B)) para los que existen un elemento a de A y un elemento b de B tal que X
={ {a}, {a,b}}. Con esto lo que he hecho es seleccionar los pares (a,b), con a elemento
de A y con b elemento de B, pues { {a}, {a,b}} =(a, b) por definición. La unicidad
del producto cartesiano es clara, pues no pueden existir dos conjuntos X, Y, formados
por los pares (a, b) con a elemento de A y con b elemento de B, y estos dos conjuntos
X e Y ser distintos. Para visualizar mejor la demostración puedes ir dando los pasos
anteriores con un ejemplo sencillo: A = {1, 3}, B = {2, 4}

78
El matemático Adolf Fraenkel añadió a los siete axiomas de Ernst Zermelo, el axioma
de reemplazo o esquema de reemplazo, porque la teorı́a de Zemelo presentaba lagunas
a la hora de trabajar con ordinales y cardinales infinitos. Como el axioma de especifi-
cación de Zermelo se pudo demostrar usando el axioma de reemplazo de Fraenkel, la
teorı́a de Zermelo-Fraenkel -teorı́a ZF- quita el axioma de especificación e incorpora el
axioma de reemplazo. También se añadieron otros dos axiomas: el axioma de regula-
ridad y el axioma de elección.
La teorı́a de conjuntos es muy potente. Se puede construir el conjunto de los números
naturales {0,1, 2, 3, 4....} que se denota por N. Considerando ahora el producto carte-
siano N × N y definiendo una relacion de equivalencia, se puede construir el conjunto
de los numeros enteros {....-4, -3, -2, -1, 0, 1, 2, 3, 4,...}, denotado por Z. Considerando
el conjunto formado por los pares (a,b) tal que a es un numero entero y b es un nume-
ro entero distinto de 0; y definiendo una relación de equivalencia se llega al conjunto
de los números racionales - fracciones- denotado por Q. Con la definición de sucesión
de Cauchy y definiendo una relación de equivalencia en el conjunto de sucesiones de
Cauchy de números racionales se llega a los números reales, denotado por R. -también
se puede llegar a R, mediante cortaduras de Dedekind- Con el producto cartesiano
R × R se construyen los números complejos. En definitiva, la teorı́a de conjuntos sirve
para formalizar la mayor parte de las matemáticas. No solo se construyen conjuntos,
sirve también para definir conceptos muy importantes en matemáticas como los de re-
lación de equivalencia, relación de orden y el de función, fundamentales para el análisis
matemático

Es importante resaltar que en la la teorı́a de conjuntos no se ha definido expli-


citamente lo que es un conjunto. No es necesario hacerlo, es suficiente saber como
funcionan los conjuntos. Este funcionamiento nos viene dado por los axiomas anterio-
res

La teorı́a de conjuntos es relativamente moderna, se inició a finales del siglo XIX y


su máxima actividad se produjo durante la primera mitad del siglo XX. Hasta entonces
la matemática habı́a sobrevivido perfectamente sin esta teorı́a, basta recordar a los dos
grandes genios y creadores, en el siglo XVII, del cálculo diferencial e integral: Newton
y Leibniz. En gran medida, la teorı́a de conjuntos, junto a la lógica matemática, lo
que hacen es formalizar la matemática que ya se conocı́a, aunque también añaden de-
finiciones y teoremas nuevos. Lo más importante de la lógica matemática y la teorı́a
de conjuntos es crear el marco adecuado para el posterior desarrollo de las matemáticas

Otra teorı́a axiomática muy importante en Matemáticas es la elaborada por David


Hilbert en 1903 sobre la Geometrı́a. Hilbert tampoco define lo que es un punto, ni una
recta, ni un plano. Son los axiomas los que explican el funcionamiento y la relación
entre estos objetos matemáticos. La labor de Hilbert consistió en formalizar de un

79
modo totalmente riguroso y transparente la geometrı́a del griego Euclides -Geometrı́a
Euclidiana- de hace más de 2000 años.

3.2 TABLAS DE VERDAD DE LAS PROPOSICIONES ELEMENTA-


LES

A partir de frases matemáticas -llamadas proposiciones o fórmulas- se pueden ge-


nerar otras proposiciones usando los conectores lógicos: no, y, o, etc...Veremos las seis
proposiciones más simples que se pueden generar. Las tablas de verdad son muy im-
portantes en lógica matemática, pues a partir de ellas se puede reconocer lo que es una
tautologı́a y lo que no lo es. Las tautologı́as son fundamentales en las demostraciones
matemáticas, pues son elementos que se pueden usar en una demostración. En las ta-
blas de verdad hay que escribir todas las posibilidades de verdadero y falso que pueden
tomar las proposiciones A, B, C..-es decir los valores de verdad y falsedad de todas las
letras que aparecen en la proposicion- Si estamos con una proposicion que consta de
las letras A y B, hay que poner todas las posibilidades de verdadero y falso de estas
dos letras. Por ejemplo, supongamos que estamos construyendo la tabla de verdad de
la proposición A y B. Ver la sección 3.2.2. Comenzamos poniendo el caso: A verdadera,
B Verdadera. En este caso la proposicion A y B es vedadera. En la tabla de verdad
esto se recoge en la segunda fila y hay que interpretarlo ası́: Si A es verdadera y B es
verdadera entonces la proposición A y B es verdadera. La siguiente fila corresponde
al caso: A verdadera y B falsa. En este caso la proposicion A y B es falsa por no ser
verdaderas las dos proposiciones A, B. La segunda fila se lee asi: si A es verdadera y B
es falsa, entonces la proposicion A y B es falsa. En la siguiente fila ponemos el caso: A
falsa y B verdadera. En este caso la proposicion A y B es falsa, por no ser verdaderas
las dos proposiciones A, B. Esta fila se interpreta ası́: si A es falsa y B es verdadera,
entonces A y B es falsa . En la última fila ponemos el caso A falsa y B falsa. En este
caso la proposición A y B es falsa. Se lee ası́ : si A es falsa y B es falsa, entonces la
proposición A y B es falsa.

3.2.1 LA PROPOSICIÓN: no A

Dada la proposición A, se puede construir la proposición: no A. La proposición no


A es verdadera cuando A es falsa. Veamos un ejemplo. Dada la proposición: 3 es un
número par. Construyo la proposición: 3 no es un número par. Esta última proposición
es verdadera, por ser falsa la proposición: 3 es un número par. En la siguiente tabla,
denominada tabla de verdad, se analizan todas las posibilidades. Si A es verdadera,
entonces no A es falsa. Si A es falsa, entonces no A es verdadera

80
A no A
V F
F V

3.2.2 LA PROPOSICIÓN: A y B

Dadas las proposiciones A, B, se puede construir la proposición A y B. Para que la


proposición A y B sea verdadera, tienen que serlo A y B. Ejemplo: 11 es un número
primo y 6 es un número par. Esta proposición es verdadera, por ser verdaderas las dos
proposiciones: 11 es un número primo y 6 es un número par. En la siguiente tabla,
se analizan todas las posibilidades. Si A es verdadera y B es verdadera, entonces la
proposición A y B es verdadera. Si A es verdadera y B es falsa, entonces la proposición
A y B es falsa. Si A es falsa y B verdadera, entonces la proposición A y B es falsa. Si
A es falsa y B falsa, entonces la proposición A y B es falsa

A B A y B
V V V
V F F
F V F
F F F

3.2.3 LA PROPOSICIÓN: A o B

Dadas dos proposiciones: A, B, se puede construir la proposición A o B. A o B


será verdadera cuando una de las dos proposiciones lo sea. Si son verdaderas las dos, A
y B, entonces también lo es la proposición A o B. La tabla de verdad de la proposición
A o B es la siguiente

81
A B A o B
V V V
V F V
F V V
F F F

3.2.4 LA PROPOSICIÓN: O BIEN A O BIEN B

Dadas dos proposiciones: A , B, se puede construir la proposición: o bien A o bien


B. Para que esta proposición sea verdadera tiene que serlo una y sólo una de las dos.
La tabla de verdad de la proposición A o B es la siguiente

A B o bien A o bien B
V V F
V F V
F V V
F F F

3.2.5 LA PROPOSICIÓN: A IMPLICA B

Dadas dos proposiciones: A, B, se puede construir la proposición: A implica B, que


se abrevia ası́: A ⇒ B. Intuitivamente significa: que si se verifica A, entonces se veri-
fica B. Es decir, si A es verdadera, entonces B es verdadera. La tabla de verdad es la
siguiente

82
A B A ⇒ B
V V V
V F F
F V V
F F V
Nota: Obsérvese que cuando la proposición A es falsa, la proposición A ⇒ B es ver-
dadera, independientemente de si B es verdadera o falsa. Solo hay un caso en el que
A ⇒ B es falsa: cuando A es verdadera y B falsa. Pueden llamar la atención los casos
siguientes: 1) cuando A es falsa y B verdadera, entonces la proposición A implica B
es verdadera. 2) Cuando A es falsa y B es falsa, entonces la proposición A implica
B es verdadera. El caso 1 con un ejemplo es ası́: si 1 es mayor que 2 entonces 3 es
número impar. El caso 2 con un ejemplo es asi: si 1 es mayor que 2 entonces 3 es
un número par. Los dos casos son verdaderos por un convenio que se ha adoptado.
Este convenio se ha adoptado para poder asignar un valor de verdadero o falso a la
proosición A implica B dependiendo de los valores de verdad o falsedad de A y B,
sin quedar ningún caso fuera. Además el convenio adoptado artificialmente es el único
convenio coherente posible, porque si hubiesemos asignados otros valores de verdad o
falsedad a los dos casos raros, la tabla de verdad coincidirı́a con la tabla de verdad de
otras proposiciones. Por suerte para los matemáticos, estas situaciones tan raras para
el sentido común, no se presentan casi nunca en la práctica. Este convenio lógico es
totalmente inofensivo para las matemáticas

En los textos especializados de lógica matemática a la proposicion A o B se la en-


tiende como la proposición: no A implica B (noA ⇒ B )

3.2.6 LA PROPOSICIÓN: DOBLE IMPLICACIÓN A, B

Dadas dos proposiciones: A, B, se puede construir la proposición: doble implicación


A, B, que se abrevia ası́: A ⇔ B. Esta proposición significa: A implica B y B implica
A, y será verdadera cuando lo sean las dos: A implica B y B implica A. Para compren-
derla mejor, vamos a ver una tabla de verdad con las tres proposiciones: 1) A implica
B 2) B implica A 3) doble implicación A, B

83
A B A⇒B B⇒A A⇔B
V V V V V
V F F V F
F V V F F
F F V V V

3.3 TAUTOLOGIAS Una tautologı́a es una proposición que siempre es verda-


dera, independientemente de los valores de verdad o falsedad de sus componenetes: A,
B, C....En el siguiente ejemplo veremos como la proposición (noB ⇒ noA) ⇒ (A ⇒ B)
es una tautologı́a. Hay que demostrar que cualesquiera que sean los valores de verdad
o falsedad de las componentes A, B, el resultado final de (noB ⇒ noA) ⇒ (A ⇒ B)
siempre es verdadero.

A B no A no B A ⇒ B noB ⇒ noA (noB ⇒ noA) ⇒ (A ⇒ B)


V V F F V V V
V F F V F F V
F V V F V V V
F F V V V V V
Las tautologı́as juegan un papel muy importante en lógica matemática, pues su uso
está permitido en las demostraciones matemáticas. Veamos con un ejemplo como se
puede usar la anterior tautologı́a: (noB ⇒ noA) ⇒ (A ⇒ B) para demostrar una
propiedad de los números enteros

Sea n un número entero. Demostrar que si n3 es par, entonces n es par. En lugar


de probar lo anterior directamente se puede probar lo siguiente: Si n es impar en-
tonces n3 es impar. Es decir, en lugar de demostrar que: A ⇒ B se demuestra que:
noB ⇒ noA. Como la tautologia dice que: (noB ⇒ noA) ⇒ (A ⇒ B), demostrando

84
que (noB ⇒ noA) habré demostrado también que (A ⇒ B). Veamos la demostración
de: si n es impar, entonces n3 es impar. Los números pares se pueden expresar como 2k
para algún entero k, y los impares se pueden expresar como 2k+1 para algún entero k.
Ejemplos: 8 = 2 x 4, 9 = 2 x 4 + 1. Con esta caracterización de los números impares
la demostración es muy fácil

Por ser n impar n = 2k+1. Elevando al cubo queda (2k + 1)3 = 8k 3 + 12k 2 + 6k + 1.
Sacando factor común al 2 queda 2(4k 3 + 6k 2 + 3k) + 1. Como hemos expresado n3
como el doble de un entero más 1, tenemos que n3 es impar

Muchas veces se pueden deducir las tautologı́as simplemente pensando, es decir sin
construir una tabla de verdad. Veamos una tautologı́a donde intervienen tres letras.
Si (A implica B y B implica C), entonces A implica C. Vamos a pensar lo anterior: Si
cuando se verifica A se verifica B; y cuando se verifica B se verifica C, entonces cuando
se verifica A se verifica C. Esto es totalmente lógico y se deduce que es una tautologı́a
sin necesidad de construir una tabla de verdad. Esta tautologı́a, formalmente se escribe
ası́: ((A ⇒ B) y (B ⇒ C)) ⇒ (A ⇒ C)

Si quisieramos construir la tabla de verdad de la tautologı́a: ((A ⇒ B) y (B ⇒


C)) ⇒ (A ⇒ C) tendrı́amos que considerar los ocho casos posibles de verdadero y fal-
so en las letras A, B, C. El resultado final de estos ocho casos posibles serı́a verdadero,
por tratarse de una tautologı́a. Para adquirir práctica con las tablas de verdad serı́a
interesante que la construyeses.

3.4 CUANTIFICADORES LOGICOS

Los cuantificadores lógicos surgen de forma natural en el lenguaje matemático. Es-


tos cuantificadores son:
1) para todo, cuyo sı́mbolo es ∀
2) existe, cuyo sı́mbolo es ∃

En los textos especializados en lógica matemática el cuantificador: existe x tal que


R, se define como una abreviatura de: no para todo x no R. Es decir, que en teorı́a, el
cuantificador: existe, no es imprescindible para escribir las matemáticas. Lo que ocurre
es que es muy cómodo usarlo, pues cada vez que se hable de existencia no hay que
usar la expresiń más larga: no para todo x no R; y aunque sea como una abreviatura
de: no para todo x no R, todos los textos lo usan. Ejemplo: existe un número primo
entre 25 y 30. Esto es equivalente a decir: no todos los números entre 25 y 30 son
compuestos. Donde la R anterior es la propiedad: ser primo y no R, lo contrario, es

85
decir, ser compuesto. A los no primos se les llama compuestos porque se pueden des-
componer en dos factores distintos del propio número y del 1. Ejemplo: 10= 2 x 5 .
Los cuantificadores están presentes en todas las partes de las matemáticas: axiomas,
teoremas, definiciones, etc... En algunos contextos se usa la expresión ”para cada”, en
lugar de ”para todo”

Supongamos que el axioma de las partes -axioma 5 de Zermelo- lo enunciamos


ası́: Para todo conjunto X, existe un conjunto Y formado por los subconjuntos de X.
También se puede escribir usando la expresión ”para cada”: Para cada conjunto X,
existe un conjunto Y formado por los subconjuntos de X. Estas pequeñas variacio-
nes se producen sólo en el lenguaje hablado o escrito. En el lenguaje formal de las
matemáticas hay que escribirlo usando el simbolo ∀, es decir, escribiendo: ∀X.... Las
matemáticas teóricamente se pueden escribir usando solamente sı́mbolos lógicos y le-
tras. Los conectores: y, o, no, también tienen sı́mbolos lógicos asociados, es decir, en el
lenguaje formal de las matemáticas no aparecerı́an: y, o, no; siendo sustituidos por sus
sı́mbolos lógicos. Para los objetivos de este libro no es necesario conocer estos sı́mbolos.

El problema que surge al querer escribir las matemáticas en su lenguaje formal es


que su lectura y su escritura es muy difı́cil y lenta . Los textos matemáticos se con-
sideran que están bien escritos cuando se consigue una lectura fluida combinando el
español -o el inglés si es un texto inglés- con el lenguaje formal de las matemáticas.
Aunque aparezcan sı́mbolos lógicos, la base del texto tiene que ser el lenguaje español
-o el inglés si es un texto inglés-. Veamos con un ejemplo como se complica la lectura
y escritura de las matemáticas usando solamente el lenguaje formal de las matemáticas.

El axioma de la unión, que ya hemos visto, dice: Para cada conjunto X, existe un
conjunto formado por los elementos de los elementos de X . Vamos a escribir este
axioma en lenguaje matemático para que se aprecie la complicación de su uso.

Axioma de la unión: ∀X∃Y ∀U (U ∈ Y ⇔ ∃V (U ∈ V V ∈ X))


V

El simbolo significa: y. Los primeros cuatro sı́mbolos: ∀X∃Y simplemente nos dicen
V

que para todo conjunto X existe un conjunto Y.....Obsérvese que en el lenguaje formal
de las matemáticas no es necesario escribir la palabra conjunto, porque en teorı́a de
conjuntos sólo se habla de conjuntos. Los siguientes sı́mbolos: ∀U (U ∈ Y ⇔ ∃V (U ∈
V V ∈ X)) nos describen qué elementos U van a pertenecer al conjunto Y. Si leemos la
V

implicacion de izquierda a derecha dice que los elementos U de Y cumplen la propiedad:


existe un elemento V de X tal que U pertenece a V; y leyéndola de derecha a izquierda
dice que si U pertenece a algún elemento V de X, entonces U es un elemento de Y.
Si no hubiesemos puesto la doble implicación en: ∀U (U ∈ Y ⇔ ∃V (U ∈ V V ∈ X))
V

no hubieramos determinado correctamente los elementos del conjunto Y. Veamos esto


último. Supongamos que sólo hubieramos puesto la implicación de derecha a izquierda.

86
Es decir: ∀U (U ∈ Y ⇐ ∃V (U ∈ V V ∈ X)). En este caso estamos obligando a que los
V

U que pertenezcan a algún elemento V de X, pertenezcan a Y, pero no estoy obligando


que los elementos de Y cumplan la propiedad: existe un elemento V de X tal que U
pertenece a V. No estarı́a bien descrito el conjunto Y, porque quiero que el conjunto Y
sólo conste de los elementos U que verifiquen la propiedad: existe un elemento V de X
tal que U pertenece a V. Ejemplo: supongamos que el conjunto X = { {1, 2}, {3,4}} si
sólo pongo la implicación de derecha a izquierda me puede valer como conjunto Y este:
{1 ,2, 3, 4 ,5, 6} porque los U que pertenecen a algún conjunto V de X, que son el 1, 2,
3, 4, pertenecen a Y. Pero también puede valer como conjunto Y este: {1, 2, 3 ,4, 5 ,6,
7} y muchos otros más. Esto ocurre porque no hemos exigido nada a los elementos de
Y. Sólo he exigido que los U que pertenecen a algún elemento V de X pertenezcan a Y,
y esto lo verifican el 1, 2, 3, 4, porque los he incluido en Y. Pero no he exigido que los
elementos de Y cumplan la propiedad: pertenecer a algún elemento V de X. Por eso
pueden pertenecer al conjunto Y: el 5, 6, 7, 8, 9,....sin problema. En cambio poniendo
también la implicación de izquierda a derecha: ∀U (U ∈ Y ⇒ ∃V (U ∈ V V ∈ X)),
V

exigo a los elementos de Y cumplir la propiedad: existe un elemento V de X tal que U


pertenece a V. De este modo el 5 ,6, 7, 8, 9... no pertenecen a Y porque no cumplen
la propiedad: existe un elemento V de X tal que U pertenece a V. De esta forma sı́ se
consigue que el conjunto Y sea el que quiero: la union de X, es decir Y ={1,2,3,4},
además este conjunto es único. Poner las dos implicaciones es lo mismo que decir: U
pertenece a Y si y solo si existe un elemento V de X tal que U pertenece a V. En
matemáticas, la doble implicación siempre es equivalente a decir: si y solo si
A ⇔ B es lo mismo que decir: se verifica A si y solo si se verifica B. Esto es fácil de
comprender. Si leemos la implicación de derecha a izquierda tenemos que: si se verifica
B, entonces se verifica A y si la leemos de izquierda a derecha, tenemos que: si se
verifica A entonces se verifica B. Pero esto último es lo mismo que decir que: si no se
verifica B, entonces tampoco se verifica A, pues (A ⇒ B) ⇔ (noB ⇒ noA) es una
tautologı́a. Compruébese como ejercicio que es una tautologı́a

3.5 LAS DEFINICIONES EN MATEMÁTICAS

Tećnicamente las definiciones no son otra cosa que abreviaturas de proposiciones


-de frases matemáticas- Veamos un ejemplo con la definición de inclusión entre con-
juntos. Decimos que un conjunto A está incluido en un conjunto B si todo elemento
de A es un elemento de B. A incluido en B es una abreviatura de la proposición:
todo elemento de A es un elemento de B; o en lenguaje formal una abreviatura de:
∀X((X ∈ A) ⇒ (X ∈ B))
A incluido en B se simboloza ası́: A ⊂ B. Ejemplo: Sea A el conjunto {1, 2, 3} y B el
conjunto {1, 2, 3, 4, 5}. Se verifica que todo elemento de A es un elemento de B, luego
A está incluido en B. Lo simbolizamos ası́: A ⊂ B. Cuando se habla sobre conjuntos
se hace a través de la abreviatura. Ejemplo: Sean A, B y C conjuntos tales que A

87
está incluido en B y B está incluido en C. Lo que no se hace es decir: Sean A, B y C
conjuntos, tales que todo elemento de A es un elemento de B; y todo elemento de B
es un elemento de C. Si se está escribiendo se escribe: Sean A, B y C conjuntos tales
que A ⊂ B y B ⊂ C

3.6 SISTEMAS FORMALES

Este apartado es el más abstracto de todo el libro, pero describe de una forma
precisa el funcionamiento de las matemáticas. Es la lógica que usan los matemáticos.
Para poder entenderlo hay que familiarizarse primeramente con algún texto riguroso
de lógica matemática. Quienes no deseen profundizar en lógica matemática, con una
lectura concentrada, pueden sacar una idea de lo básico del apartado. El apartado
está resumido. Al principiante, muchas cosas les parecerán superfluas o triviales, pero
nada más lejos de la realidad que considerar superfluos o triviales los contenidos que
siguen a continuación

Definición: Un sistema formal consta de lo siguiente:

1) Un lenguaje L (viene a ser como el conjunto de sı́mbolos que vamos a usar. Ejem-
plos: el sı́mbolo de igual =. Si estamos en la teorı́a de conjuntos, usaremos el sı́mbolo
de pertenece: ∈ , o letras para nombrar los conjuntos, etc...)

2) Axiomas lógicos (estos axiomas son los mismos para todas las teorı́as)

3) Axiomas no lógicos (estos axiomas son los axiomas especı́ficos de cada teorı́a,
por ejemplo, los axiomas que hemos visto de la teorı́a de conjuntos)

4) Reglas de deducción (permiten obtener proposiciones -tambien llamadas fórmulas-


a partir de otras. Estas reglas de deducción también son las mismas para todas las
teorı́as )

5) Teoremas ( se obtienen a partir de los axiomas utilizando las reglas de deducción )

Definición: Dado un lenguaje L, los axiomas lógicos de L son proposiciones o


fórmulas de L de los siguientes tipos:

1) Las tautologı́as de L (ya las hemos visto en un apartado anterior)

2) Los axiomas de cuantificadores

3) Los axiomas de igualdad

88
Los axiomas de cuantificadores son dos:

1) El axioma de ∀ (para todo): Para cada F y G, fórmulas de L, y para cada variable


x que no aparezca libre en G se tiene la fórmula: (∀x(G ⇒ F )) ⇒ (G ⇒ ∀xF )

2) El axioma de sustitución: Para cada F , fórmula de L, para cada variable x, y pa-


ra cada termino t, tal que x sea sustituible por t en F , se tiene la fórmula (∀xF ) ⇒ Fx [t]

Este axioma dice que si la fórmula F se verifica para todo x, entonces cuando
suistituyo en la fórmula la variable x por un término en concreto t, también se sigue
verificando la fórmula

Los axiomas de igualdad son tres:

1) x = x

2) Para cada f de Fn se tiene que (x1 = y1 , x2 = y2 , ..., xn = yn ) ⇒ (fx1 x2 ...xn =


fy1 y2 ...yn )

La f denota a un funtor de rango n. Los funtores, en lógica matemática, son co-


mo las funciones que manejamos a diario en matemáticas. Por ejemplo f (x) = x2 .
El anterior axioma para un funtor de longitud 1, traducido a funciones, dice: que si
x = y entonces f (x) = f (y). Es totalmente razonable, que si x = y, entonces, tengan
la misma imagen mediante la función f

3) Para cada R de Rm se tiene que (x1 = y1 , x2 = y2 , ..., xm = ym ) ⇒ (Rx1 x2 ...xm ⇒


Ry1 y2 ...ym )

La R aquı́ no denota al conjunto de números reales, sino a un relator de rango


m. Los relatores expresan hechos, equivalentes a los verbos en las lenguas naturales.
Por ejemplo, en la teorı́a de conjuntos está el relator pertenece, representado por ∈.
Este relator es de rango 2 pues relaciona a dos conjuntos. Cuando decimos x1 ∈ x2
estamos relacionando x1 y x2 mediante el relator ∈. Si lo escribimos con la notación del
axioma deberı́amos escribirlo ası́: ∈x1 x2 , primero el relator y a continuación las letras.
El axioma es totalmente razonable, pues dice que si x1 = y1 , x2 = y2 ⇒ si x1 ∈ x2
entonces y1 ∈ y2
Las reglas de deducción son dos:
1) Modus Ponens: para cada F y G, fórmulas del lenguaje L, de F , y, (F ⇒ G), se
deduce G
2) Generalización: para cada F , fórmula del lenguaje L, de F se deduce ∀xF

89
Definición: Dado un lenguaje L, una teorı́a T con lenguaje L y una fórmula F del
lenguaje L. Se dice que F es un teorema de la teorı́a T si existe una sucesion finita
F1 , F2 , ...., Fn de formulas de L tal que:
1) Fn es F (la última formula de la sucesión es el resultado al que queremos llegar, es
decir al teorema)
2) Todas las formulas: F1 , F2 , F3 ...., Fn son: o axiomas lógicos, o axiomas de la teorı́a T,
o se obtienen aplicando las dos reglas de deducción: modus ponens o generalización.
Por ejemplo, la fórmula F3 puede ser: ∀xF1 , es decir, la hemos obtenido aplicando
generalizacion a F1 , fórmula anterior en la sucesion. Otro ejemplo con modus ponens:
la formula F6 se puede obtener de dos fórmulas anteriores en la sucesión: F4 y F5 ,
aplicando modus ponens, donde F5 es: F4 ⇒ F6 . El esquema para obtener F3 y F6 es
el siguiente:
F1
F2
∀xF1
F4
F4 ⇒ F6
F6
F7
......
......
Fn

A la sucesión F1 , F2 , F3 , ...., Fn se le denomina demostración de F en la teorı́a T

90
TEMA 4: CÓMO ESTUDIAR Y COMPRENDER LAS MATEMÁTI-
CAS

La forma de estudiar las matemáticas es determinante para su comprensión. En


este capı́tulo responderemos a preguntas como: ¿Qué parte de las matemáticas hay
que trabajar para comprenderlas? ¿Existen técnicas concretas para las demostraciones
matemáticas? ¿Qué tipo de problemas es conveniente resolver? ¿Es conveniente rea-
lizar resúmenes de cada tema? ¿Existen estrategias para la resolución de problemas
matemáticoss?

Para la comprensión de las matemáticas es fundamental trabajar los siguientes tres


puntos básicos
1) Analizar detalladamente las definiciones
2) Demostrar los teoremas
3) Resolución de los problemas de cada capı́tulo

4.1 ANÁLISIS DE LAS DEFINICIONES

En una definición matemática hay muchos detalles que deben ser analizados para
una comprensión global de lo que estamos definiendo. Una buena familiarización con
los objetos matemáticos que se definen es fundamental para la posterior demostra-
ción de teoremas. Tambien es muy importante extraer una idea intuitiva de lo que se
está definiendo y no fijarse solamente en los numeros, letras y simbolos matematicos
que aparezcan en la definición. Veamos con un ejemplo: la definición de lı́mite de una
función en un punto, como hay que abordar este análisis

Dado un conjunto de números reales A se dice que el número real b es un punto


de acumulación de A si para todo número real r mayor que 0, en el intervalo (b-r,
b+r) hay al menos un punto de A distinto del b. Intuitivamente significa que podemos
encontrar puntos de A distintos de b tan próximos al b como queramos. El punto b
puede pertenecer o no pertenecer al conjunto A

En la siguiente definición, x es la variable independiente de una función f, y f(x) la


imagen de x, mediante la función f. Ejemplo: f (x) = x2 . A es un conjunto de números
reales, o lo que es lo mismo, un subconjunto del conjunto de los números reales.

DEFINICIÓN: Sea f una función de A en R -donde R es el conjunto de los números


reales y A es un subconjunto de R, es decir, un conjunto cuyos elementos son números
reales- y b un punto de acumulación de A, se dice que el número real c es lı́mite de f
cuando x tiende a b, si para todo número real s mayor que 0 existe un número real r
mayor que 0 tal que si x pertenece al intervalo (b-r, b+r) y x es distinto de b, entonces

91
f(x) pertenece al intervalo (c-s, c+s)
Ante la anterior definición lo primero que hay que preguntarse es por qué las funciones
se definen en un subconjunto A de R y no en todo R. La mayorı́a de las funciones que
se usan en matemáticas vienen definidas mediante una fórmula. Ejemplos: f (x) = x1
g(x) = x2 etc... Como no existe la división entre 0, la función f (x) = x1 debemos
definirla en el conjunto de los números reales excepto el 0, pues no existe f (0) = 10

La funcion f (x) = x1 también se puede definir en un conjunto más pequeño, por


ejemplo el intervalo (0, 1). Es decir, aún pudiéndose definir en un conjunto más grande
como es el conjunto de los números reales excepto el 0, nos limitamos a definirla en un
conjunto más pequeño como es el intervalo (0, 1). De lo único que hay que preocuparse
en este caso es de que para cada x del intervalo (0, 1) se pueda realizar la division
1
x . Como para todo x del intervalo (0, 1) se puede realizar dicha división ,la función
f (x) = x1 queda perfectamente definida en ese intervalo. Ası́ pues, el subconjunto A de
R donde definimos una función se puede elegir, pero siempre que no tengamos proble-
mas para calcular la imagen f(x) de cada elemento x de A

Formalmente, para definir una función se da una terna f = (A, B, G) donde A es


el conjunto donde está definida la función, llamado dominio de definición de f, B es el
conjunto de llegada y G es un subconjunto del producto cartesiano A × B que cumple
la siguiente propiedad: para cada x de A existe un único y de B tal que el par (x, y)
pertenece a G. A este único y se le denomina imagen de x mediante f y se le representa
mediante f(x). El ejemplo anterior -la función f (x) = x1 definida en el intervalo (0, 1)-
expresada formalmente serı́a ası́: f= ((0, 1), R, conjunto de los (x, y) de (0, 1) × R tales
que y = x1 )

En la práctica no se emplea el anterior formalismo, basta con dar el dominio de


definición: (0, 1) y la imagen de x mediante f: f (x) = x1 . El contexto nos debe aclarar
que el conjunto de llegada es el conjunto de los números reales: R. En cualquier tex-
to matemático nos podemos encontrar una expresión parecida a esta: Sea la función
f (x) = x1 definida en (0,1). Si no nos dicen donde está definida la función hay que so-
breentender que está definida en el mayor subconjunto de R donde es posible hacerlo,
es decir, en todo R excepto el 0

La siguiente pregunta que debemos hacernos sobre la definición de lı́mite es la si-


guiente: ¿por qué se exige que el punto b donde se toma el lı́mite tiene que ser un
punto de acumulación del conjunto A donde está definida la función? Veamos primero
qué es un punto de acumulación. Sea A un subconjunto de R y b un número real. Se
dice que b es un punto de acumulación del conjunto A si para todo r mayor que 0, en
el intervalo (b-r, b+r) hay algún punto de A distinto del b. Intuitivamente significa
que existen puntos de A distintos de b, tan próximos a b como queramos. El punto b

92
puede pertenecer al conjunto A o puede que no pertenezca. Ejemplos: 1) El 0 es un
punto de acumulación del intervalo abierto (0, 1) pero no pertenece al intervalo. 2) El
0 es un punto de acumulación del intervalo cerrado [0, 1] y pertenece al intervalo

Queremos que haya puntos infinitamente próximos a b y distintos de b donde la


función f esté definida, pues queremos analizar el comportamiento de f cuando se con-
sideran puntos infinitamente próximos al punto b. Si b no es de acumulación de A,
cualquier real c valdrı́a como lı́mte de f cuando x tiende a b y la definición dejarı́a de
tener sentido. Veamos esto. Supongamos que hemos definido una función f de A en R
y consideramos un punto b que no es un punto de acumulación de A. Por no ser b
un punto de acumulación, existe un número real s mayor que 0 tal que en el intervalo
(b-s, b+s) no hay puntos de A distintos de b. De este modo, para cualquier r mayor
que 0, considerando el s anterior, que es fijo, se cumple que para todo x de A distin-
to de b y perteneciente al intervalo (b-s, b+s) su imagen f(x) pertenece al intervalo
(c-r, c+r) al no haber ningún x de A distinto de b perteneciendo al intervalo (b-s,
b+s). Se puede demostrar, aunque no lo haremos aquı́, que si el punto b es un punto
de acumulación de A y existe lı́mite de f cuando x tiende a b, entonces el lı́mite es único

Otra pregunta que hay que hacerse en la definición de lı́mite es ¿por qué se pres-
cinde del valor de f en el punto b para analizar el lı́mite de f cuando x tiende a b?
Sólo se exige que los puntos x de A del intervalo (b-s, b+s) distintos de b tengan una
imagen f(x) muy próxima al lı́mite c, pero no se dice nada de f(b). Esto es porque nos
interesa el comportamiento de la función f en las proximidades del punto b, pero no
el valor f(b). En la definición de lı́mite ni siquiera se exige que la función esté definida
en el punto b, lo único que se exige es que el punto b sea un punto de acumulación del
conjunto A donde hemos definido la función.

Un ejemplo muy importante de esta última observación es la definición de derivada


en un punto. Una función f es derivable en el punto b si existe el lı́mite cuando x tiende
a b de la función auxuliar g definida mediante g(x) = f (x)−f
x−b
(b)

En esta definición se estudia si existe el lı́mite de una función g cuando x tien-


de a b, punto donde no está definida la función g, porque si sustituimos la x por la
b, en el denominador queda un 0 y no se puede dividir. Obsérvese que para poder
analizar si f es derivable en el punto b, la f si tiene que estar definida en el punto b
pues aparece el valor f(b) en el numerador. De la función g me interesa lo que ocurra
en las proximidades del punto b, pero no en b -ni siquiera se puede definir en el punto b-

Todas las definiciones matemáticas están llenas de detalles como los comentados an-
teriormente. Para conseguir comprender las matemáticas es necesario analizar a fondo
cada definición, tal y como hemos hecho en la definición de lı́mite y de derivada

93
4.2 TÉCNICAS DE DEMOSTRACIÓN EN MATEMÁTICAS

La demostración en matemáticas es la actividad más importante. Con ella se garan-


tiza que lo que afirmamos es cierto, es decir, que se puede deducir a partir de los axio-
mas de una teorı́a o de otros resultados demostrados anteriormente. Para desarrollar
cierta habilidad en el ejercicio de la demostración es necesario una práctica constante
y prolongada, enfrentándose a situaciones cada vez más complejas. A continuación se
proponen ciertas técnicas que son muy usuales en las demostraciones matemáticas. A
pesar de estas técnicas, la intuición y la imaginación de cada uno juegan un papel
decisivo a la hora de demostrar enunciados matemáticos.

4.2.1 DEMOSTRACIÓN DE A ⇒ B COMENZANDO A RAZONAR


DESDE A

Tratamos de demostrar que si se verifica A entonces también se verifica B. Se co-


mienza examinando todos los elementos que intervienen en la situación A, y compren-
diendo la situación que expresa B, se intenta conectar A con B. Veamos un ejemplo
práctico de una demostracion del tipo A implica B, que se escribe como A ⇒ B

Sea n un número entero, demostrar que si n es impar entonces n3 también es impar.


En este ejemplo la proposición A es: n es impar y la proposición B es: n3 es impar.
En un lenguaje matemático riguroso no se dice proposición sino fórmula. Los números
pares se pueden expresar como 2k para algún entero k, y los impares se pueden ex-
presar como 2k+1 para algún entero k. Ejemplos: 8 = 2 x 4, 9 = 2 x 4 + 1. Con esta
caracterización de los números impares la demostración es muy fácil

Por ser n impar n = 2k+1. Elevando al cubo queda (2k + 1)3 = 8k 3 + 12k 2 + 6k + 1.
Sacando factor común al 2 queda 2(4k 3 +6k 2 +3k)+1. Como hemos expresado n3 como
el doble de un entero más 1, tenemos que n3 es impar. A veces la demostración es más
compleja y necesitamos pasar por una proposición intermedia C. Es decir comenzando
a razonar desde A encontramos una proposición C de tal modo que A implique C y C
implique B. De esta manera A implicará B. Esta forma de proceder puede ser aún más
compleja necesitando encontrar varias proposiciones o fórmulas: C, D, E, etc... para
conectar A con B. Un esquema posible serı́a este: A implica C, C implica D, D implica
B. En esta situación A implicarı́a B

4.2.2 DEMOSTRACIÓN DE A ⇒ B COMENZANDO A RAZONAR


DESDE B

Queremos demostrar que si se verifica A entonces también se verifica B, pero ahora


comenzando a razonar desde B. Partiendo desde B, se intenta encontrar una proposi-

94
ción intermedia C, de tal modo que A implique C y C implique B, de esta forma A
implicará B. Veamos un ejemplo de esta situación
√ √ √
Veamos que si x e y son números reales mayores que 0, entonces x+y < x+ y

Ahora la proposición o fórmula A es: x e y son números reales mayores que 0. La


√ √ √
proposición o fórmula B es: x + y < x + y. Las raı́ces cuadradas en la fórmula
B dificultan la demostración, pero si elevo al cuadrado los dos miembros de B queda
√ √
x + y < x + y + 2 x y. A esta última fórmula la llamo C. Ahora conecto A con
C. Obviamente si x e y son números reales mayores que 0, entonces x + y < x + y
√ √
+ 2 x y. Ahora conecto C con B. Tomando raı́ces cuadradas en los dos miembros
√ √ √
queda: x + y < x + y. LLegando al resultado deseado.

Si en lugar de encontrar una fórmula o proposición C que me sirva para conectar


A con B hubiese necesitado encontrar más de una fórmula o proposición: C, D, E,
etc...para conectar A con B, el método hubiera resultado más complejo. Un esquema
posible serı́a este: A implica C, C implica D, D implica B. En esta situación A impli-
carı́a B

4.2.3 DEMOSTRACIÓN DE A ⇒ B USANDO LA TAUTOLOGIA (noB ⇒


noA) ⇒ (A ⇒ B)

Este método ya se explicó y se puso un ejemplo en el apartado de tautologı́as del


capı́tulo 3

4.2.4 DEMOSTRACIÓN POR REDUCCIÓN AL ABSURDO

En muchas situaciones para demostrar algo en matemáticas se comienza admitiendo


lo contrario de lo que se pretende demostrar. La demostración termina cuando se llega
a una contradicción. Veamos un ejemplo de esta técnica de demostración

Se quiere demostrar que existen infinitos números primos. Partimos de que sólo
existe un número finito de primos p1 , p2 , p3 ....pn . Consideramos el número que se ob-
tiene multiplicando los números primos anteriores y después sumando 1 al resultado:
k = p1 x p2 x p3 x....x pn + 1. k no es primo pues es mayor que todos los números
primos anteriores. p1 no es un divisor de k, pues k es el siguiente de un múltiplo de
p1 . Lo mismo ocurre para p2 , p3 , ...pn . Por no ser primo k, existe un número primo que
divide a k. Esto contradice la hipótesis de que p1 , p2 , p3 ....pn eran los únicos primos

En la anterior demostración la contradicción se ha producido en el contexto de ella.


Partı́amos de que los números primos eran p1 , p2 , p3 ....pn y llegabamos a que habı́a

95
algún primo más que éstos. Otras veces, en las demostraciones por reduccion al ab-
surdo, se llega a una contradicción con algo que ya conocı́amos antes. Por ejemplo,
cuando se demuestra que si existe el lı́mite de una función f cuando la variable x tien-
de a un punto b, entonces este lı́mite es único. En esta demostración por reducción al
absurdo se comienza suponiendo que una función f posee dos lı́mites distintos cuando
la variable x tiende al punto b. Se comienza a razonar sobre esta hipótesis y se llega a
que 1 < 23 y esto está en contradicción con las matemáticas que conocı́amos antes de
la demostración. Sabı́amos que 1 no es menor que 23

4.2.5 DEMOSTRACIÓN DE A ⇒ B POR REDUCCIÓN AL ABSURDO

Queremos demostrar que si se verifica A entonces también se verifica B, pero razo-


nando por reducción al absurdo. Partiendo de que se verifica A y no B hay que llegar a
una contradicción. En el siguiente ejemplo no importa no saber lo que es un conjunto
compacto de números reales, ni lo que son conjuntos abiertos de números reales, ni
lo que es un recubrimiento de un conjunto. Donde hay que fijar la atención es en la
estructura de la demostración. Queremos demostrar que si tenemos un conjunto K de
números reales de tal forma que de cualquier recubrimiento de K por conjuntos abier-
tos puede extraerse un recubrimiento finito, entonces K es compacto. La proposición
A va a ser: De cualquier recubrimiento de K por conjuntos abiertos puese extraerse
un recubrimiento finito. La proposición B va a ser: K es compacto. Partimos que de
cualquier recubrimiento de K por conjuntos abiertos puede extraerse un recubrimiento
finito, pero que K no es compacto. A partir de aquı́ se desarrolla una demostración
donde se utilizan las dos hipótesis anteriores: 1) De cualquier recubrimiento de K por
conjuntos abiertos puede extraerse un recubrimiento finito. 2) K no es compacto. Y
se llega a la construcción detallada de un recubrimiento de K por conjuntos abiertos
del cual no puede extraerse un recubrimiento finito. Esto está en contradicción con la
hipótesis 1): De cualquier recubrimiento de K por conjuntos abiertos puede extraerse
un recubrimiento finito

4.2.6 DEMOSTRACIÓN CON UN CONTRAEJEMPLO

Para demostrar la falsedad de un enunciado, a veces es muy práctico recurrir a


un contraejemplo que me demuestre que el enunciado no es verdadero. Si queremos
demostrar la falsedad del enunciado: todos los polı́ticos son honrados, basta mostrar
uno que no lo sea. En matemáticas se trabaja de forma análoga. Veamos un ejemplo
con funciones.

Queremos demostrar la falsedad del siguiente enunciado: Toda función derivable en


un punto a, tal que la derivada en el punto a sea igual a 0 tiene un máximo o un
mı́nimo en el punto a. Considerando la función f (x) = x3 la derivada en el 0 es igual

96
a 0 y la función no tiene ni máximo ni mı́nimo en el 0 pues es estrictamente creciente
en el 0. Se dice que la función f (x) = x3 en el punto 0 es un contraejemplo que ha
demostrado la falsedad del enunciado

4.2.7 DEMOSTRACIÓN POR ANÁLISIS DE CASOS POSIBLES

La naturaleza de algunos problemas en matemáticas hace que las demostraciones


se realicen separando los distintos casos de los que consta el enunciado. Veamos un
ejemplo de este método aplicado a la derivabilidad de una función

Vamos a analizar detalladamente si la función f (x) =| x | (valor absoluto de x) es


derivable en todo R. Si x ≤ 0 | x |= −x y si x > 0 | x |= x. En la demostración hay
que distinguir tres casos:
1) La derivabilidad en los puntos menores que 0
2) La derivabilidad en los puntos mayores que 0
3) La derivabilidad en el 0

Si a es menor que 0, f es derivable en a, pues definiendo la funcion g(x) = −x en


todo R, g es derivable en todo R y las funciones f y g coinciden en un entorno de a:
(a-r, a+r). Si a es mayor que 0, f es derivable en a, pues definiendo la función h(x) = x
en todo R, h es derivable en todo R y las funciones f y h coinciden en un entorno de
a: (a-r, a+r). Si a = 0 no podemos aplicar la técnica anterior de buscar una función
derivable que coincida con f en un entorno de 0: (-r, r) pues a la izquierda y a la derecha
del 0, f viene dada por dos fórmulas distintas. Este caso hay que analizarlo usando la
definición de derivada en un punto. Hay que ver si existe el lı́mite cuando x tiende a
0 de la función c(x) = f (x)−f
x−0
(0)

Si x es menor que 0 la función c vale c(x) = −x x = -1, luego el lı́mite de la función c


cuando x tiende a 0 desde la izquierda del 0 es -1. Si x es mayor que 0 la funcion c vale
c(x) = xx = 1, luego el lı́mite de la función c cuando x tiende a 0 desde la derecha del
0 es 1. Como los lı́mites laterales no coinciden, la función f (x) =| x | no es derivable
en el 0

4.2.8 DEMOSTRACIÓN USANDO EL PRINCIPIO DEL PALOMAR

Este principio es muy obvio y se utiliza en algunas demostraciones matemáticas.


El principio del palomar puede resumirse ası́: Tenemos un palomar con 8 agujeros
y 9 ó más palomas. Si todas las palomas han entrado por los agujeros del palomar,
entonces existe al menos un agujero por donde han entrado más de una paloma. Ob-
viamente cuando estamos ante un problema matemático los agujeros y las palomas
hay que sustituirlos por números, conjuntos, etc...Obviamente el número de agujeros

97
no tiene por qué ser 8 ni el número de palomas 9 ó más de 9. Estos números sólo se
han usado para exponer con mayor claridad el principio. Ver los acertijos 18 y 66 del
tema 1, donde ha resultado determinante para demostrar el enunciado

4.2.9 DEMOSTRACIÓN POR EL MÉTODO DE INDUCCIÓN

Este método de demostración es muy importante pues se usa con mucha frecuencia
en matemáticas. Cuando en un enunciado aparecen los números naturales: 1, 2, 3, 4...
es probable que el método de inducción sea el adecuado. Este método es una copia
de la estructura de los números naturales. Primero se demuestra que el enunciado es
verdadero para el 1. Después se demuestra que si el enunciado es verdadero para un
número natural arbitrario n, entonces también lo es para el siguiente natural n + 1.
De este modo el enunciado será cierto para todos los naturales

¿Por qué es verdadero para el 2? porque se ha demostrado que es verdadero para


el 1 y que si es verdadero para un natural también lo es para el siguiente, por lo tanto
también será verdadero para el 2 .¿Por qué es verdadero para el 3?. Por ser verdadero
para el 2 y por haber demostrado que si es verdadero para un natural también lo es
para el siguiente. Ası́ sucesivamente, el enunciado será verdadero para todos los núme-
ros naturales. Veamos un ejemplo de demostración por inducción

Vamos a demostrar la fórmula de las permutaciones de n elementos. Si tenemos


tres elementos a, b, c y los ordenamos de todas las formas posibles obtenemos: abc,
acb, bac, bca, cab, cba. Esto es, hay 6 formas posibles de ordenarlos. Este 6 no es
otra cosa que el factorial de 3, representado por 3! El factorial de 3 es el producto de
los números que van desde el 3 hasta el 1: 3! = 3 x 2 x1 = 6. El factorial de 4 es 4!
= 4 x 3 x 2 x 1 = 24. El factorial de 7 es 7! = 7 x 6 x 5 x 4 x 3 x 2 x 1 = 5040
etc.. Vamos a demostrar por el método de inducción que el número de permutacio-
nes u ordenaciones posibles de n elementos viene dado por la fórmula n! (factorial de n)

Comenzamos demostrándolo para el 1. Obviamente sólo hay una manera de ordenar


un elemento. Esto coincide con 1! = 1. Luego la fórmula de las permutaciones es ver-
dadera para el 1. Demostremos ahora que si la fórmula es verdadera para n elementos
entonces también lo es para n + 1 elementos.

Si tenemos n + 1 elementos, el último elemento de la ordenación puede ser cualquie-


ra de los n + 1. Si considero fijo el último elemento de la ordenación, los n primeros
elementos se pueden ordenar de n! maneras dsitintas -esto es por la hipótesis de in-
ducción, es decir por haber supuesto verdadera la fórmula para n elementos- .Tenemos
pues que por cada elemento que fije como último de la ordenación existen n! orde-
naciones distintas. De esto se sigue que el total de ordenaciones distintas serán n!

98
multiplicado por los n +1 elementos que pueden ocupar la última posición. Esto es, (n
+1) x n! Pero este producto es igual a (n + 1)! Ha quedado demostrada la fórmula de
las permutaciones de n elementos Pn = n!

4.2.10 DEMOSTRACIÓN POR EL MÉTODO DE INDUCCIÓN FUER-


TE

Este método es una variante del método anterior. Primero se demuestra que un
enunciado es verdadero para el 1. Después se demuestra que si el enunciado es verda-
dero para los n primeros números naturales: 1, 2, 3, ...n entonces también lo es para
el siguiente n + 1. De esta forma el enunciado será cierto para todos los números na-
turales. Veamos un ejemplo de este método de demostración: la demostración de que
todos los números naturales salvo el 1 se pueden expresar como un producto de núme-
ros primos. Nota: esta factorización es la que calculamos cuando queremos obtener
el mı́nimo común múltiplo o el máximo común divisor de varios números naturales.
Ejemplos: 18 = 2 x 3 x 3 = 2 x 32 , 30 = 2 x 3 x 5, 50 = 2 x 5 x 5 = 2 x 52 , 175 =
5 x 5 x 7 = 52 x 7. Comenzamos la demostración con el 2, pues para el 1 no hay que
demostrralo. El enunciado, obviamente, es cierto para el 2, pues 2 se expresa como un
producto de primos ası́: 2 = 2 (con un único factor). Supongamos que el enunciado es
verdadero para todos los números naturales menores o iguales que n, y probemos que
también lo es para el siguiente natural n + 1

Si n + 1 es primo, entonces no hay nada que demostrar. En caso contrario tendrá algún
divisor distinto de 1 y de n + 1. El menor de ellos tiene que ser necesariamente un
primo p1 . Entonces el cociente n+1
p1 por ser menor que n + 1 será producto de primos
y por lo tanto también lo habrá de ser n + 1. Con esto queda demostrado el enunciado.

No sólo se cumple que todo número natural distinto del 1 se puede factorizar como
producto de números primos, también se cumple que la factorización es única. Es decir
que un número no se puede expresar como producto de primos de dos formas distintas.
Ejemplo: si el 100 factoriza como 100 = 2 x 2 x 5 x 5 = 22 x 52 No hay que buscar
más factorizaciones posibles del 100 en números primos porque no existen

En algunas ocasiones se emplea el método de inducción o el de inducción fuerte


sin que aparezcan de una forma clara los números naturales. Por ejemplo, cuando in-
tentamos demostrar alguna propiedad de los polinomios, podemos sospechar que la
demostración por inducción sobre el grado del polinomio puede ser el camino correcto
. Comenzarı́amos demostrando la propiedad para los polinomios de grado 1. Después
demostrarı́amos que si la propiedad es cierta para los polinomios de grado n entonces
también es cierta para los de grado n+ 1. Usando la inducción completa comenzarı́amos
demostrando la propiedad para los polinomios de grado 1. Después demostrarı́amos

99
que si la propiedad es cierta para los polinomios de grado menor o igual que n, entonces
también es cierta para los de grado n+ 1

Otro campo donde se usa a veces la inducción es en el Álgebra Lineal. Para de-
mostrar un enunciado sobre matrices o determinnates se puede intentar el método de
inducción o el de inducción fuerte realizando una inducción sobre la dimensión de la
matriz o del determinante, pues esta dimensión es un número natural. En general, se
puede intentar una demostración por inducción o inducción fuerte en cualquier campo
de las matemáticas donde aparezcan de forma directa o indirecta los números naturales

4.2.11 DEMOSTRACIÓN POR EL MÉTODO DEL DESCENSO DE


FERMAT

Este método se basa en el hecho de que en cualquier conjunto de números naturales


hay uno de ellos que es menor que todos los demás. Cuando se usa este método se
procede por reducción al absurdo -recordemos que en la reducción al absurdo se supo-
ne que el enunciado es falso y se llega a una contradicción- Veamos un ejemplo de la
Teorı́a de Números donde se usa el método del descenso de Fermat

Vamos a ver un esquema de la demostración de que la ecuación: x4 + y 4 = z 2 no


tiene solución en los números naturales: 1, 2, 3, ....Es decir no existen tres números na-
turales x, y, z tales que cumplan la igualdad x4 + y 4 = z 2 . En cualquier texto avanzado
de Teorı́a de Números se puede ver la demostración completa. Lo importante ahora es
comprender el método de descenso de Fermat.

Procedamos por reducción al absurdo. Supongamos que la ecuación tiene solución.


De todas las soluciones que pueda tener elijamos el menor natural u tal que x4 +y 4 = u2
para algún x, y. A partir de esta terna x, y, u, se puede encontrar otra terna a, b, c que
cumpla la ecuación, y donde c es menor que u. Se llega ası́ a una contradicción pues
u era el más pequeńo natural para los que existı́an x e y tales que x, y, u era solución
de la ecuación

4.2.12 DEMOSTRACIÓN POR EL MÉTODO DE LA DIAGONAL DE


CANTOR

El método de la diagonal de Cantor se utiliza en algunas demostraciones de ma-


temáticas superiores. En esencia consiste en construir un cuadro infinito formado por
infinitas sucesiones y donde cada una de estas sucesiones también consta de un número
infinito de elementos. Se van ordenando las sucesiones una debajo de otra, como se
demuestra en el siguiente diagrama:

100
a11 , a12 , a13 , a14 , a15 , a16 .......

a21 , a22 , a23 , a24 , a25 , a26 .......

a31 , a32 , a33 , a34 , a35 , a36 .......

a41 , a42 , a43 , a44 , a45 , a46 .......

..............................................

..............................................

Simplemente razonando sobre la sucesión formada por los elementos de la diagonal:


a11 , a22 , a33 ..... se llega con éxito al final de la demostración. Veamos un ejemplo de
demostración usando el método de la diagonal de Cantor. Vamos a demostrar que es
imposible construir un cuadro infinito como el anterior donde estén las cifras decima-
les de todos los números reales mayores que 0 y menores que 1. Lo demostramos por
reducción al absurdo. Supongamos que tenemos construido un cuadro infinito como el
anterior, donde están las cifras decimales de todos los números reales mayores que 0 y
menores que 1. En la primera fila están las infinitas cifras decimales de un número real
mayor que 0 y menor que 1 donde se ha suprimido el 0 de la izquierda de la coma, en
la segunda fila están las infinitas cifras decimales de otro número real mayor que 0 y
menor que 1 donde se ha suprimido el 0 de la izquierda de la coma. Ası́ sucesivamente
con todas las filas. Considero el número real comprendido entre 0 y 1 cuya expresión
decimal es 0, b1 b2 b3 b4 b5 ... donde:
b1 = 1 si a11 = 0 y b1 = 0 si a11 es distinto de 0
b2 = 1 si a22 = 0 y b2 = 0 si a22 es distinto de 0
b3 = 1 si a33 = 0 y b3 = 0 si a33 es distinto de 0
Ası́ sucesivamente con todas las cifras decimales de 0, b1 b2 b3 b4 b5 ...

Este número, 0, b1 b2 b3 b4 b5 ..., está comprendido entre 0 y 1 pero no está incluido


en el cuadro anterior, ya que de la primera sucesión del cuadro - primera fila- difiere
por lo menos de la primera cifra decimal. De la segunda sucesión del cuadro difiere
por lo menos de la segunda cifra decimal. Ası́ sucesivamente de cualquier sucesión del
cuadro -cualquier fila- difiere por lo menos de una cifra decimal. Hemos llegado a una
contradicción pues hemos encontrado un número real mayor que 0 y menor que 1 cuyas
cifras decimales no están en el cuadro anterior. La demostración ha terminado

Si identificamos la primera fila del cuadro anterior con el número natural 1, la segun-
da fila con el número natural 2, la tercera fila con el número natural 3, etc...deducimos
que el conjunto N de los números naturales tiene menos elementos que el conjunto de

101
números reales comprendidos entre 0 y 1 pues no ha sido posible incluir todos los reales
comprendidos entre 0 y 1 en el cuadro anterior formado por infinitas filas. Aunque el
conjunto N de los números naturales y el conjunto de números reales comprendidos
entre 0 y 1 tienen infinitos elementos, el cardinal o número de elementos del intervalo
real (0, 1) es un infinito mayor que el cardinal o número de elementos del conjunto N
de los numeros naturales

4.2.13 DEMOSTRACIÓN POR CAMBIO DE INCÓGNITAS

Como dijimos al comienzo de la sección de las demostraciones, la intuición y la ima-


ginación pueden resultar decisivas a la hora de demostrar algún enunciado matemático.
El ejemplo que se expone a continuación para resolver las ecuaciones de tercer grado es
una muestra de ello. Los matemáticos del Renacimiento consigueron resolver mediante
cambios de incógnitas las ecuaciones de tercer y cuarto grado. Lo que sigue se conoce
como método de Cardano para la resolución de ecuaciones de tercer grado. La autorı́a
de Cardano es bastante dudosa.

Tenemos la ecuación: Ax3 + Bx2 + Cx + D = 0 donde A, B, C y D son numeros


reales y A es distinto de 0. Dividimos todos los miembros por A, para que el coeficiente
que multiplica a x3 sea un 1 y se trabaje de una manera más cómoda. Obviamente,
las soluciones de la nueva ecuación son las mismas que las de la ecuación original. La
ecuación que resulta la expresamos como: x3 + ax2 + bx + c = 0. Ahora realizamos
el primer cambio de incógnita, hacemos x = z − a3 . Sustituyendo x en la ecuación
anterior se produce el primer milagro, desaparece el término cuadrático -el exponente
2-, llegando a una ecuación de la forma: z 3 + pz + q = 0. Si has realizado los cálculos
2 3
anteriores tienes que haber obtenido que p = b − a3 y q = 2a ab
27 − 3 + c

La idea es encontrar las soluciones de la ecuación z 3 + pz + q = 0 con incógnita


z, y como x = z − a3 , a las soluciones que se obtengan les restamos a3 , obteniendo de
este modo los valores de x que son solución de x3 + ax2 + bx + c = 0. Para resolver la
ecuación z 3 + pz + q = 0 necesitamos realizar un nuevo cambio de incógnitas. Hacemos
z = u + v. Entonces z 3 = (u + v)3 = u3 + 3uv(u + v) + v 3 = 3uvz + u3 + v 3 . Juntando
esto último con z 3 queda z 3 − 3uvz − u3 − v 3 = 0. Igualando ahora los coeficientes de
esta ecuación con los coeficientes de la ecuación z 3 + pz + q = 0 nos queda un sistema
de ecuaciones con incógnitas u, v:
p = −3uv
q = −u3 − v 3

El anterior sistema es equivalente -tiene las mismas soluciones- a este otro sistema:
u + v 3 = −q
3

u3 v 3 = −( p3 )3

102
Ahora hay que darse cuenta que u3 y v 3 son las soluciones de la ecuación de segundo
p3
grado z 2 + qz − 27 = 0. Esto último es fácil de ver, pues si escribo la ecuación de
segundo grado: (z − u3 )(z − v 3 ) = 0 es obvio que u3 y v 3 son las soluciones de esta
ecuación. Multiplicando ahora queda z 2 + (−u3 − v 3 )z + u3 v 3 = 0, y teniendo en cuenta
p3
que u3 + v 3 = −q y que u3 v 3 = −( p3 )3 = − 27 se tiene que u3 y v 3 son las soluciones de
p3
la ecuación de segundo grado z 2 + qz − 27 = 0.

4 3
Ahora se calcula el discriminante de la ecuación de segundo grado 4 = q 2 + 27 p.
Dependiendo de si es positivo, negativo o cero, se obtendrán unas soluciones u otras
de la ecuación z 3 + pz + q = 0

Si 4 esr positivo, la ecuación posee una solución real y dos complejas. Si se establece
√ r √
que u = 3 −q+2 4 , y v = 3 −q−2 4 , entonces la única solucion real es z1 = u + v. Además
existen dos soluciones complejas conjugadas
z2 = ju + jv
z3 = j 2 u + j 2 v √
Donde j = − 2 + i 23 , j es el conjugado de j, y j 2 es el conjugado de j 2
1

Si 4 = 0, la ecuación posee dos soluciones reales, una simple y una doble


3 −q −p 3q
q q
z1 = 2 2 = −2 3 = p (Esta es la solución simple)
z2 = − 3 −q −p −3q
q q
2 = 3 = 2p (Esta es la solución doble)
Que z1 sea una solución simple y z2 sea una solución doble significa que el término
(z − z1 ) aparece con exponente 1 en la factorización del polinomio z 3 + pz + q, y el
término (z − z2 ) aparece con exponente 2 en la factorización del polinomio z 3 + pz + q.
Es decir, z 3 + pz + q = (z − z1 )(z − z2 )2

Si 4 es negativo, la ecuación tiene tres soluciones reales simples. Sin embargo, es


necesario hacer una incursión en rlos números complejos para encontrar todas las solu-
ciones. Si establecemos que u = 3 −q+i|4|
2 , las tres soluciones reales vienen dadas por
z1 = u + u
z2 = ju + ju
z3 = j 2 u + j 2 u
La barra horizontal
√ encima de los complejos significa conjugado y el complejo j es:
1 3
j = −2 + i 2

OBSERVACIONES IMPORTANTES
1) Una vez que hemos obtenido las soluciones de la ecuación z 3 + pz + q = 0, como
habı́amos partido del cambio x = z − a3 tenemos que restar a estas soluciones a3 y de
esta forma llegamos a las soluciones de la ecuación original x3 + ax2 + bx + c = 0
2) Ninguno de los tres casos descritos, dependiendo del valor del discriminabte 4,

103
recoge el caso de que la solución de la ecuación original x3 + ax2 + bx + c = 0 sea una
solución real triple. Es decir que el polinomio x3 + ax2 + bx + c = 0 factorice como
x3 + ax2 + bx + c = (x − r)3 donde r es la raı́z real triple. En este caso el proceso es más
corto, no es necesario llegar al cambio z = u + v . Cuando se hace el cambio x = z − a3
La ecuación z 3 + pz + q = 0 simplemente queda de la forma z 3 = 0 que tiene una raı́z
real triple z1 = 0. Si a esta raı́z triple le restamos a3 obtenemos − a3 que es la raı́z real
triple de la ecuación original x3 + ax2 + bx + c = 0

En la práctica, en muchas ocasiones, para no tener que aplicar este método de


resolución de ecuaciones de tercer grado, las ecuaciones se preparan de antemano
con coeficientes enteros, es decir Ax3 + Bx2 + Cx + D = 0 donde A, B, C y D son
números enteros y A es distinto de 0. Además también se prepara de antemano que
exista al menos una solución entera x1 . Como esta solución entera tiene que divi-
dir al término independiente D, con un simple tanteo con los divisores de D, obte-
nemos esta solución entera x1 . Una vez obtenida la solución entera x1 dividimos el
polinomio Ax3 + Bx2 + Cx + D entre x − x1 obteniendo una factorización del tipo
Ax3 + Bx2 + Cx + D = (x − x1 )(Ax2 + Ex + F ). Las soluciones de la ecuación de
segundo grado Ax2 + Ex + F = 0, junto a la solución que ya conocı́amos: x1 , serán
todas las soluciones de la ecuación original Ax3 + Bx2 + Cx + D = 0

Las ecuaciones de cuarto grado Ax4 + Bx3 + Cx2 + Dx + E = 0 también se resuel-


ven por cambio de incógnitas. En cambio, las ecuaciones de grado mayor que 4 no se
pueden resolver mediante fórmulas explı́citas como ocurre con las de grados 1, 2, 3 y 4.
En los cursos de Álgebra superior se demuestra la imposibilidad de encontrar fórmulas
explı́citas para las ecuaciones de grado 5, 6, etc....Se dice que estas ecuaciones no son
resolubles por radicales.

4.2.14 DEMOSTRACIÓN POR CAMBIO DE VARIABLE

En Cálculo Diferencial, muchas veces las demostraciones se consiguen efectuando


cambios de variables. Los capı́tulos donde más se utilizan estos cambios son el capı́tulo
de lı́mites, el de integrales y el de ecuaciones diferenciales. Vamos a comenzar viendo
una demostración de un lı́mite.

Queremos demostrar que el lı́mite de la función f (x) = ax cuando x tiende a b es


ab . En este tipo de funciones se considera que la base a es mayor que 0. Suponemos
que hemos demostrado con anterioridad que el lı́mite de la funcion f (x) = ax cuando
x tiende a 0 es 1. Usando las propiedades de las funciones exponenciales escribo ax =
ab ax−b . Ahora es cuando hay que realizar el cambio de variable. Para calcular el lı́mite
de ax−b cuando x tiende a b se hace el cambio z = x − b. De este modo, cuando x
tiende a b, z tiende a 0. Luego el lı́mite de ax−b cuando x tiende a b es igual al lı́mite

104
de az cuando z tiende a 0. Pero ya sabı́amos que el lı́mite de az cuando z tiende a 0
era 1. De esta forma queda que el limite de ab ax−b cuando x tiende a b es igual a ab
multiplicado por 1, es decir ab . Ya está demostrado
Nota: Para poder igualar el lı́mite de la nueva función az cuando z tiende a 0, que
surge tras el cambio de variable z = x − b, con el lı́mite de ax−b cuando x tiende a
b, existe un teorema en la teorı́a de lı́mites que me garantiza que estos dos lı́mites
coinciden.

El método del cambio de variables se puede usar en la demostración de enunciados


teóricos como el que acabamos de ver y en ejercicios prácticos de lı́mites. Una demos-
tración matemática, si está bien hecha, no distingue entre la teorı́a y los ejercicios.
Veamos ahora un ejemplo de cambio de variable en integrales. Los cambios de varia-
bles en integrales se realizan para llegar a una función que ya sabemos integrar

2
√ Queremos resolver la integral xsen(x )dx. Si hacemos el cambio de variable: x =
R

t tenemos que sustituir la variable x por la nueva variable t, de tal forma que en la
nueva integral no
√ aparezca la variable x. También tenemos que calcular la derivada de
la función x = t y el valor obtenido sustituirlo por dx en la integral de partida
√ 1
Tenemos que la derivada del cambio de variable x = t es dx = 2√ dt. Como
√ t
x√= t tenemos que x2 = t √ . Sustituimos ahora en le integral de partida, obteniendo
1
tsent 2√t dt. Cancelando t, por aparecer multiplicando y dividiendo, obtenemos
R

una función en la variable t que es muy fácil integrar: sent dt. Sacando el 21 fuera de
R
2
la integral, queda 12 sentdt = − 21 cost
R

Ahora hay que volver a la variable x. Como tenemos que x2 = t el resultado final
de la integral es xsen(x2 )dx = − 12 cos(x2 ) + C
R

Todos los cálculos que hemos hecho en el cambio de variables están avalados por
el teorema de cambio de variable en una integral. Este teorema, para poder aplicarse,
establece unas hipótesis que deben cumplirse. La función que elegimos para el cambio
de variable debe ser una función biyectiva, derivable, y con derivada continua. Veamos
como la función elegida para el cambio de variable en el ejemplo anterior cumple estas
(x) = xsen(x2 ) definida en el intervalo
hipótesis. Consideramos la función de partida f √
(0, ∞) y la función del cambio de variable x = t, con variable independiente t, defi-
nida también en el intervalo (0, ∞). La función del cambio de variable es una biyección
del intervalo (0, ∞) en el intervalo (0, ∞). También es derivable, y su derivada es una
1
función continua en (0, ∞) pues su derivada es 2√ t

La integral anterior xsen(x2 )dx se puede también resolver de una forma casi in-
R

mediata, sin necesidad de aplicar un cambio de variable. Si hubiese un 2 multiplicando

105
dentro de la integral serı́a totalmente inmediata pues tendrı́amos dentro de la integral
el seno de una función u(x) multiplicado por la derivada de u(x). La función u(x) = x2 .
Ajustando el 2 que falta con 12 queda: 12 2xsen(x2 )dx = − 12 cosx2 + C
R

Lo importante no era resolver la integral de una forma u otra, sino exponer el


método del cambio de variable. Para ver todos los detalles del teorema del cambio de
variable se recomienda consultar un libro riguroso de Análisis Matemático. Aunque en
este ejemplo se podrı́a haber prescindido de él, en otros ejercicios de integrales más
difı́ciles es absolutamente necesario. Al igual que en los lı́mites, el método del cambio
de variable se puede usar para demostrar enunciados teóricos o para resolver ejercicios
concretos de integrales, como hemos hecho en el ejemplo anterior

4.2.15 DEMOSTRACIÓN CAMBIANDO EL CONTEXTO

Las demostraciones por cambio de contexto son las más difı́ciles de lograr. Son más
que un método de demostración, constituyen una parte importante del corazón de las
matemáticas. En estas demostraciones se percibe claramente la creatividad y la genia-
lidad que hay que poseer a veces en matemáticas. Vamos a ver dos problemas clásicos
que se demostraron cambiando el contexto donde estaban enunciados

Queremos demostrar que no existen tres números enteros distintos de 0: x, y, z,


que verifiquen la ecuación x3 + y 3 = z 3 . Como el problema está planteado dentro de
los números enteros, en un principio es razonable pensar, que aplicando todo lo que
sabemos sobre números enteros, y sin salirnos del conjunto Z de los números enteros,
el problema tendrı́a que resolverse. La realidad es otra. Posteriormente a los números
enteros se inventaron los números racionales -las fracciones-, después los reales y luego
los complejos. Los números naturales están incluidos dentro de los números enteros.
Los enteros están incluidos dentro de los racionales, los racionales dentro de los reales,
y los reales dentro de los complejos. Para resolver este problema hay que ascender
hasta el conjunto de los números complejos. Se resuelve en un subconjunto muy parti-
cular de los complejos: en Z(ω). Este subconjunto de los complejos está formado por
los complejos que se pueden expresar como a + bω con a y b números enteros, y donde
ω es el número complejo ω = cos 2π 2π
3 + isen 3 . El número complejo ω es una de las tres
raı́ces cúbicas de 1, es decir, ω 3 = 1. Ejemplos de elementos de Z(ω): 2 + 3ω, −2 + 4ω,
5 − 3ω....Todos los números enteros pertenecen a Z(ω), pues eligiendo b = 0, todos
los enteros se pueden expresar como a + bω con a y b enteros. Ejemplos: 5 = 5 + 0ω,
−3 = −3 + 0ω etc...

Como los números enteros pertenecen al conjunto Z(ω), si se demuestra que la


ecuación x3 + y 3 = z 3 no tiene solución en Z(ω), entonces tampoco la tendrá en el
conjunto de los números enteros. Es decir, si no existen tres elementos de Z(ω) dis-

106
tintos de 0: x, y, z, que verifiquen la ecuación x3 + y 3 = z 3 , tampoco existirán tres
números enteros distintos de 0: x, y, z, que verifiquen la ecuación x3 + y 3 = z 3 . Lo
anterior es precisamente lo que se demuestra, que la ecuación no tiene solución en Z(ω).

Tanto Z como Z(ω) tienen estructura algebraica de anillo, pero la aritmética de


Z(ω) tiene gran interés ya que presisamente en Z(ω) es donde la ecuación x3 + y 3 = z 3
se factoriza: x3 + y 3 = (x + y)(xω + yω 2 )(xω 2 + yω) = z 3

Los interesados en la demostración completa pueden consultar cualquier texto avan-


zado de Teorı́a de Números. Veamos otro problema clásico cuya demostración se con-
sigue con un cambio de contexto
2
Para demostrar que la integral definida de la función f (x) = e−x en todo R es

π nos vamos a la teorı́a de integración en dos variables, es decir, a la integración
en R2 . Como tenemos que trabajar con funciones de dos variables, modificamos la
2 2
función anterior y consideramos la función g(x, y) = e−(x +y ) de R2 a R. Esta fun-
ción se puede poner como el producto de dos funciones, una función con variable x
2 2
y otra con variable y; g(x, y) = e−x e−y . Ahora aplicando el teorema de Fubinni en
2
R2 , queda que la integral de g(x,y) es igual a la integral de e−x en R multiplicado
2
por la integral de e−y en R. Obviamente estas dos integrales coinciden, pues son fun-
ciones iguales. En lo único que difieren es en la letra de la variable independiente. De
2
este modo, la integral de g(x, y) es igual a la integral de e−x en R, elevado al cuadrado
2 2
Por otra parte, realizando un cambio de variable en la función g(x, y) = e−(x +y ) , se
pasa de las variables (x, y) a las coordenadas polares (r, θ). Con este cambio se llega a
2 2
que la integral de g(x, y) = e−(x +y ) en R2 es igual a π. Como antes habı́amos llegado
2 2
a que la integral de g(x, y) = e−(x +y ) en R2 era igual al cuadrado de la integral de
2 2 √
e−x en R, obtenemos que la integral de e−x en R es igual a π. Para ver los detalles
de la demostración recomiendo leer algún libro riguroso de Análisis Matemático

Las demostraciones matemáticas no sólo son necesarias para comprender bien las
matemáticas, también lo son para resolver los ejercicios prácticos con gran soltura,
pues muchos detalles técnicos que se utilizan en las demostraciones se usan tambien
en la resolución de los problemas. Si no hemos visto estos detalles técnicos alguna vez,
por no haber realizado las demostraciones, es muy poco probable que se nos ocurran
cuando estemos resolviendo problemas en un examen

4.3 ESTRATEGIAS PARA LA RESOLUCIÓN DE PROBLEMAS EN


MATEMÁTICAS

La resolución de problemas es una labor imprescindible si se quiere adquirir des-

107
treza en cualquier campo de las Matemáticas: Cálculo Diferencial e Integral, Algebra
Abstracta, Algebra Lineal, Geometrı́a, Probabilidad, Estadı́stica, etc...Cuando se re-
suelven problemas, indirectamente se refuerzan los conocimientos teóricos, pues para
buscarlos solución tenemos que analizar la teorı́a para ver qué camino elegimos. Las
fórmulas se memorizan de una forma óptima cuando las hemos puesto en práctica va-
rias veces en los problemas. Si vamos a realizar un examen de la materia que estamos
estudiando, la resolución de problemas es totalmente necesaria, pues en el examen,
con toda probabilidad nos tendremos que enfrentar a resolver algunos problemas. Si
resolvemos antes una gran variedad de ellos, es muy probable que algunos problemas
de los que hemos resuelto sean similares a los del examen.

A continuación vamos a describir una serie de estrategias que nos ayudarán en la


resolución de problemas en matemáticas

4.3.1 Realiza un resumen de la teorı́a de cada capı́tulo

Antes de enfrentarnos a la resolución de problemas de un determinado capı́tulo es


muy conveniente realizar un resumen de los contenidos teóricos de éste. De esta forma,
tendremos a la vista todo el material teórico que podemos usar. A veces, los proble-
mas sencillos se resuelven directamente utilizando un único apartado de este resumen.
En problemas más complejos es probable que tengamos que usar varios apartados de
la teorı́a del capı́tulo. Si se observa que el resumen es insuficiente para resolver un
determinado problema, tal vez necesitemos algún contenido teórico de algún capı́tu-
lo estudiado con anterioridad. Como estos temas anteriores tienen que estar también
resumidos, será una tarea fácil encontrar lo que necesitamos. Muchos problemas ma-
temáticos se componen de una mezcla de varios capı́tulos y cuanto mejor tengamos
resumidos los capı́tulos anteriores más fácil será la localización de la teorı́a que nece-
sitemos

4.3.2 Dibuja un diagrama, un esquema, una gráfica...

Los enunciados de los problemas se comprenden mejor con un diagrama o una figura
que represente las partes más sustanciales del problema. Una vez que hemos entendido
mejor lo que nos piden que resolvamos, será mucho más fácil llegar a la solución. El
diagrama que construyas debe incorporar de forma sencilla los datos importantes y
suprimir los superfluos. De esta forma quedarán resaltadas visualmente las relaciones
entre los elementos importantes del problema y de ahı́ es más probable que se despren-
da alguna idea que clarifique la situación y haga más transparente el camino hacia la
solución. Estos dibujos adquieren una relevancia especial cuando estamos ante un pro-
blema de Geometrı́a, como puede ser el cálculo de una superficie o de un volumen, etc..

108
4.3.3 Intenta resolver el problema por reducción al absurdo

Como ya se explicó en la sección de técnicas de demostración, para demostrar algo


por reduccı́on al absurdo se parte justo de lo contrario de lo que se intenta
√ demostrar,
hasta llegar a una contradicción. Si por ejemplo quiero demostrar que 2 es un nu-
mero irracional -esto significa que no existe una fracción que al elevarla al cuadrado
se obtenga otra con un numerador doble que el denominador- se parte de que existe
una fracción cuyo cuadrado es una fracción con numerador doble que el denomina-
dor. Con esta hipótesis hay que conseguir llegar a una contradicción. La reducción
al absurdo se emplea con mucha frecuencia en matemáticas y siempre√hay que tener-
la en mente. Veamos como se demuestra por reduccion al absurdo que 2 es irracional.

Se parte de que es racional, es decir de que existe una fracción ab cuyo cuadrado es
igual a 2. Como toda fraccion admita una fracción equivalente irreducible, elegimos la
fraccion equivalente irreducible de ab . A esta fracción irreducible la representamos por
m 30 3
n . Por ejemplo, la fracción irreducible de 20 es 2 . Se comienza a razonar sobre la frac-
m2
ción irreducible m
n y se deduce que si n2 = 2, entonces m y n son números pares. Esto
m
es contradictorio con que la fracción n es irreducible, pues si m y n son pares la frac-
ción se puede reducir dividiendo numerador y denominador entre 2. Ver los detalles de
la demostración en algún libro riguroso de Análisis Matemático o de Teorı́a de Números

4.3.4 Intenta resolver el problema por algún método general de demos-


tración

Los métodos que se han visto en la sección de técnicas de demostración, son váli-
dos no sólo para demostraciones de resultados teóricos, sino para resolver cualquier
problema matemático. Si en el problema hay números naturales prueba con el método
de inducción o con el de inducción fuerte. En matrices y determinantes tambien se
puede probar la inducción o la inducción fuerte pues los números naturales aparecen
en la dimensión de la matriz y del determinante. En polinomios, los números naturales
aparecen en el grado del polinomio, ası́ pues, intentar una demostración por inducción
no es nada descabellado. Si estás en un problema de lı́mites, integrales o de ecuaciones
diferenciales y no encuentras un camino fácil prueba con un cambio de variable. El
método de reducción al absurdo puede tener éxito en cualquier campo, pues como he-
mos comentado en el apartado precedente, se usa con mucha frecuencia en matemáticas

4.3.5 Simplifica el problema dándote cuenta de las simetrı́as

Veamos ejemplos tı́picos de simetrı́as donde éstas simplifican notablemente los pro-
blemas. Estas simplificaciónes suponen normalmente un 50 por ciento del problema
2
Supongamos que me piden analizar y representar la función f (x) = e−x definida en

109
toda la recta real. Me doy cuenta que f (2) = f (−2); f (3) = f (−3); f (100) = f (−100);
etc... Es decir, me doy cuenta que f (x) = f (−x) para todo número real x. De esta
forma la parte de gráfica de la función a la izquierda del 0 se obtendrá mediante una
simetrı́a respecto al eje Y de la parte de gráfica que hay a la derecha del 0. No sólo
simplificamos el dibujo de la gŕafica en un 50 por ciento, sino que esta simplificación
también se produce en el análisis matemático de la función. Si por ejemplo hemos de-
mostrado que el lı́mite de la función cuando x tiende a infinito (∞) es cero, entonces,
por simetrı́a, el lı́mite de la función cuando x tiende a menos infinito (−∞) también
es cero. Si detecto un punto de inflexión en un número real positivo x = a, entonces
habrá otro punto de inflexión en el numero x = −a. Con el crecimiento y decrecimien-
to hay que tener cuidado, pues a la derecha del 0, es decir, en el intervalo (0, ∞) la
funcion es decreciente y a la izquierda del cero, es decir, en el intervalo (0, −∞) es
creciente. Lo mejor es observar con la gráfica las propiedades que se conservan y las
que cambian al hacer una simetrı́a respecto al eje Y

Tambien hay funciones que tienen simetrı́a respecto al origen (0, 0). La funcioń g(x) =
x3 −x
x2 es un ejemplo de este tipo de simetrı́a. Se cumple que f (2) = −f (−2); f (5) =
−f (−5) etc... es decir, f (x) = −f (−x). para todo numero real x. En este caso lo
que hayamos dibujado en el primer cuadrante, al aplicarle la simetrı́a respecto del
(0, 0), hay que dibujarlo en el tercer cuadrante y lo que hayamos dibujado en el cuarto
cuadrante hay que pasarlo al segundo cuadrante. Hay que tener cuidado con estas
simetrı́as pues invierten muchas cosas. Lo que es un máximo en un punto x = a, al
aplicarle la simetrı́a, se convierte en un mı́nimo en x = −a y viceversa. Si una función
es convexa en un punto x = a, en el dibujo se aprecia claramente que la función es
cóncava en el punto x = −a. En cambio, el crecimiento y el decrecimiento se conservan.
Lo mejor es ver la gráfica para ver qué se invierte y que no se invierte al aplicar la
simetrı́a respecto del (0, 0). De cualquier manera, apoyarse en la simetrı́a de la función
reduce el trabajo en un 50 por ciento

Otro ejemplo clásico de simetrı́as se produce en algunas funciones de varias varia-


bles, es decir, funciones definidas en R2 , R3 , etc...Supongamos que tenemos la función
f (x, y) = xyex+y . Si observamos como actúan las variables x e y en la fórmula xyex+y ,
debemos llegar a la conclusión de que las dos variables hacen exactamente lo mis-
mo. Esto se traduce diciendo que f (2, 3) = f (3, 2); f (5, −1) = f (−1, 5) etc...Es decir
f (a, b) = f (b, a) para todo (a, b) de R2 . Si considero g(x, y) = xye2x+y , entonces las
variables no actúan igual, pues a x se le multiplica por 2 en el exponenete, y a la
variable y no se le multiplica. Como en la función f las variables x e y actúan de igual
forma, todas las propiedades de la función f en la variable x son también propiedades
de f en la variable y, y viceversa. Si, por ejemplo, la función f tiene derivada parcial
con respecto a x continua, entonces f también tiene derivada parcial con respecto a
y continua. Todo lo que se diga para la variable x queda dicho para la variable y, y

110
viceversa. En el cálculo de las derivadas parciales también nos ahorramos trabajo. Si
hemos calculado primero la derivada parcial de f con repecto a la variable x, para
calcular la derivada parcial de f con respecto a la variable y simplemente hay que in-
tercambiar la x con la y en el resultado de la parcial con respecto a x. Veamos esto de
forma explı́cita. La derivada parcial de f con respecto a x es: yex+y + xyex+y Sin hacer
ningún cálculo se deduce que la derivada parcial de f con respecto a y es: xex+y +xyex+y

Veamos ahora un ejemplo de simetrı́a en el teorema fundamental del orden en R.


Este teorema afirma que todo conjunto no vacı́o de números reales acotado superior-
mente tiene un supremo. Acotado superiormente significa que existe un número real
K tal que todos los elementos del conjunto son menores o iguales que K. A cualquier
número que cumpla lo que cumple K se le dice que es una cota superior del conjunto. A
la menor de las cotas superiores se le denomina supremo del conjunto. A partir de este
teorema se deduce un resultado simétrico. Todo conjunto no vacı́o de números reales
acotado inferiormente tiene un ı́nfimo. Acotado inferiormente significa que existe un
número real H tal que todos los elementos del conjunto son mayores o iguales que H.
A cualquier número que cumpla lo que cumple H se le dice que es una cota inferior del
conjunto. A la mayor de las cotas inferiores se le denomina ı́nfimo del conjunto. Tan-
to el supremo como el ı́nfimo pueden pertenecer al conjunto o pueden no pertenecer.
Ejemplos: el supremo del intervalo abierto (0, 1) es el 1, y el 1 no pertenece al conjun-
to. El supremo del intervalo cerrado [0, 1] es el 1, y ahora el supremo sı́ pertenece al
conjunto

La demostración del teorema fundamental del orden en R no es fácil, pero el enun-


ciado del ı́nfimo sı́ lo es, simplemente hay que basarse en el teorema fundamental del
orden en R y aplicar una simetrı́a. Veamos como se aplica esta simetrı́a. Sea A un
conjunto no vacı́o de números reales acotado inferiomente. Considero el conjunto −A
formado por los opuestos de los elementos del conjunto A. Claramente −A está aco-
tado superiormente. Aplicando el teorema fundamental del orden en R, −A tiene un
supremo. Vamos a llamar s a este supremo. Es claro que el número real −s será el
ı́nfimo del conjunto A. Con un ejemplo se entenderá mejor los argumentos de la de-
mostración anterior. Supongamos que tengo el conjunto A que es el intervalo (−3, −2).
Al calcular el conjunto −A se obtiene el intervalo (2, 3). El supremo del conjunto −A
es el 3. Entonces,−3 es el ı́nfimo del intervalo A = (−3, −2). Geométricamente lo que
he hecho es una simetrı́a respecto del cero. El intervalo (2, 3) es el simétrico respecto
del 0 del intervalo (−3, −2)

4.3.6 Experimenta con casos concretos

Algunos resultados importantes de las matemáticas son consecuencia de muchos


experimentos. El matemático Gauus pasó gran tiempo elaborando listas enormes de

111
números primos para analizar como se distribuı́an éstos entre los números naturales. El
objetivo de Gauss no era contemplar las listas que elaboraba, sino intuir algún teorema
importante sobre los números primos. El teorema del mapa de los cuatro colores se
demostró por primera vez en 1967 con un ordenador. Esto, quiere decir, que no todos
los teoremas proceden de una idea genial, sino que muchas veces son las pruebas, los
experimentos, los que van conduciendo poco a poco a las conclusiones correctas. El
teorema del mapa de los cuatro colores es el acertijo 57 del capı́tulo 1 -vuelve a leerlo-.
Es un teorema aparentemente simple como para que la demostración se le hubiera ocu-
rrido a algún matemático, pero tuvieron que ser las pruebas con el ordenador las que
condujeran a la primera demostración. Esta demostración no fue admitida por algunos
matemáticos. Argumentaban que el ordenador no habı́a explicado por qué para colo-
rear un mapa con provincias se necesitaban a lo sumo cuatro colores. Posteriormente
se realizaron demostraciones más razonables. En la actualidad, la potencia de cálculo
de un ordenador es enorme y se pueden elaborar programas que vayan aportando luz
sobre alguna propiedad matemática desconocida. Al final, hay que realizar la demos-
tración matemática, pero los experimentos con el ordenador facilitan enormemente
la tarea de investigación. Este grado de experimentación con potentes ordenadores
corresponde sólo a matemáticos profesionales, pero hay propiedades sencillas que se
pueden experimentar sin ordenador. Si, por ejemplo, estoy dudando de una propiedad
de las matrices, puedo poner varios ejemplos para ver si es verdadera o no. Si en algún
caso no se ha verificado la propiedad significa que no es verdadera, y ese caso me sirve
de demostración de la falsedad de la propiedad. Este método lo vimos en la técnica de
demostración denominada: demostración con un contraejemplo. Pero, ¿qué conclusión
hay que extraer si en todos los ejemplos que he puesto se ha verificado la propiedad?
No se puede concluir que es verdadera, sólo se puede sospechar que es verdadera, pero
no afirmar. Para afirmar que es verdadera hay que encontrar una demostración gene-
ral. Los ejemplos con los que he experimentado, en este caso, me han servido de guı́a
para saber lo que tengo que hacer a continuación: encontrar la demostración

4.3.7 Busca analogı́as con otros problemas

En muchas ocasiones nos encontramos con problemas que tienen un gran parecido
a otros que hemos resuelto antes. En estos casos los detalles de las demostraciones
suelen ser muy parecidos. Ası́ pues, si recordamos como demostramos esos problemas
anteriores, dispondremos de una excelente guı́a. Veamos un ejemplo de álgebra donde
la demostración de las propiedades de un anillo me sirven de guı́a para demostrar las
propiedades de un espacio vectorial sobre un cuerpo

El problema consiste en demostrar las siguientes propiedades de un espacio vectorial


V sobre un cuerpo K
−(λ→−
v ) = λ(−→
−v ) = (−λ)→−
v , para todo λ de K y todo → −
v de V

112
Comparo las propiedades anteriores con las de un anillo A

−(ab) = a(−b) = (−a)b, para todo a y b de A

Para demostrar la primera igualdad −(ab) = a(−b) hay que fijarse en lo que di-
ce: el opuesto de ab es: a multiplicado por el opuesto de b. Aplicando lo que quiere
decir ser opuesto, si demuestro que ab + a(−b) = 0, donde 0 es el elemento neu-
tro de la suma, quedarı́a demostrado. Aplico la propiedad distributiva de los anillos:
ab + a(−b) = a(b + (−b)) = a0 = 0

Para demostrar la primera igualdad de los espacios vectoriales −(λ→ −v ) = λ(−→−


v)
simplemente me fijo en la anterior demostración para anillos. Tengo que demostrar que
el opuesto de (λ→ −
v ) es: λ multiplicado por el opuesto de →−v . Nuevamente aplicando

− →

lo que quiere decir ser opuesto, si demuestro que λ v + λ(−→

− −
v ) = 0 , donde 0 es
el neutro de la suma en V, quedarı́a demostrado. Al igual que en los anillos aplico
la distributiva, pero ahora es la distributiva respecto de la suma de vectores: λ→−
v +

− →
− →
− →
− →

λ(− v ) = λ( v + (− v )) = λ. 0 = 0 .

Vuelvo otra vez al anillo. En lugar de demostrar a(−b) = (−a)b, lo que hago es
demostrar que −(ab) = (−a)b. Hay que ver que ab + (−a)b = 0. Se demuestra apli-
cando otra vez la distributiva. En el espacio vectorial V en lugar de demostrar que
λ(−→−v ) = (−λ)→−
v se demuestra que −(λ→ −
v ) = (−λ)→−v y se hace de forma análoga que
en los anillos pero ahora aplicando la distributiva respecto de la suma de escalares
(se denomina escalares a los elementos de K). Para ver todos los detalles se puede
consultar un libro riguroso de Álgebra

4.3.8 Comienza resolviendo los casos más sencillos

Cuando estamos ante un problema muy general lo más conveniente es comenzar


resolviendo los casos más sencillos. ¿A qué llamamos problema muy general? A pro-
blemas que vienen sin datos numéricos. El enunciado puede comenzar ası́: Sea f una
función de Rm en Rn .....demostrar que.....En este tipo de problemsa hay que demos-
trar que lo que nos piden es verdadero para todo m y n naturales. Si nos hubieran
puesto esto: Sea f una función de R2 en R3 ..... demostrar que....Este problema no es
tan general como el anterior, pues nos están especificando quienes son los conjuntos de
partida y de llegada de la función y lo que nos piden solo tendrı́amos que demostrarlo
para esos conjuntos (R2 , R3 )

Supongamos que un problema de matrices comienza ası́ . Sea A una matriz cuadra-
da de orden n....demostrar que....Supongamos que intuimos que la demostración va a

113
consistir en actuar sobre todos los elementos de la matriz. No en tomar la matriz como
un paquete entero sino en hacer cálculos sobre todos los elementos de la matriz. Para
resolver este problema lo mejor es comenzar a resolverlo para algunos casos concretos:
n =1, n= 2, n=3...Es mucho más fácil razonar sobre una matriz cuadrada de orden
2, que sabemos que tiene exactamente 4 elementos, que sobre una matriz cuadrada
general de orden n. Solamente con escribir una matriz cuadrada de orden n nos damos
cuenta de que la notación se complica enormemente. La matriz estarı́a llena de puntos
suspensivos. Las filas de esta matriz serı́an de la siguiente forma: a11 a12 a13 ..... a1n .
La segunda fila serı́a: a21 a22 a23 ..... a2n . Ası́ sucesivamente con todas las filas. Pero
surge otro problema con la notación. Como no sabemos cuántas filas tiene la matriz,
debemos poner también puntos suspensivos entre las filas hasta llegar a la ultima fila
que serı́a: an1 an2 an3 .... ann . La matriz completa tendrı́a la siguiente forma:
a11 a12 a13 ..... a1n
a21 a22 a23 ..... a2n
............................
............................
an1 an2 an3 ..... ann

En la anterior matriz es mucho más dificil razonar que en una cuadrada de dimen-
sión 2 que tiene una forma mucho más simple, al no tener puntos suspensivos. Tiene
esta forma:
a11 a12
a21 a22

Una vez que hayamos resuelto el problema en la matriz cuadrada de dimensión 2,


tenemos que trasladar las ideas que hayamos usado, a la matriz cuadrada de orden n,
pero ahora la tarea es mucho más fácil pues ya sabemos qué ideas son. También se
puede probar antes en una matriz cuadrada de dimensión 3. Siempre será más sencillo
que en la matriz cuadrada de dimensión n

Veamos ahora un problema de cálculo diferencial en varias variables que al resolver-


lo en el caso sencillo n =1 se obtiene la idea para la demostración general en dimensión
n. Obtener la idea directamente en el caso general de dimensión n hubiera sido mucho
más difı́cil

Quienes hayan estudiado alguna vez funciones en varias variables comprenderán el


problema. Los que no hayan estudiado nunca esto, quédense sólo con la estrategia de
resolver el problema en una variable y después aplicar las ideas al caso general de
varias variables. El enunciado del problema es el que sigue:
Sean A y B dos conjuntos abiertos de Rn y f un C 1 - difeomorfismo de A en B. De-
mostrar que para todo a de A el Jacobiano de f en a es distinto de 0

114
Un C 1 - difeomorfismo de A enB es una funcion biyectiva f de A en B, tal que f
y su inversa f −1 tienen derivadas parciales continuas. El Jacobiano de f en a es el
determinante de la matriz formada por las derivadas parciales de f en a

Lo primero que hay que hacer es interpretar el problema en el caso de que la función
f sea de una variable, es decir, en el caso de que n = 1. La interpretación es la que
sigue:
Sean A y B dos conjuntos abiertos de R y f una función biyectiva de A en B tal que f
y f −1 tienen derivadas continuas. Demostrar que para todo punto a de A la derivada
de f en a es distinta de 0

El problema, planteado en R, se ha hecho mucho más asequible. Hay que demos-


trarlo en R y luego trasladar las ideas de la demostración a Rn , para demostrarlo en
Rn . En R no es demasiado difı́cil darse cuenta de que el problema se resuelve aplicando
la regla de la cadena para funciones derivables, pues al ser f derivable en el punto a y
f −1 derivable en el punto f (a), la función compuesta f −1 ◦ f es derivable en el punto
a, y ademas la derivada de la función compuestaf −1 ◦ f en el punto a es la derivada
de f −1 en el punto f (a), multiplicada por la derivada de f en el punto a. Por otra
parte, como la función compuesta f −1 ◦ f es igual a la función identidad i(x) = x y la
derivada de la funcion identidad es constantemente igual a 1, tenemos que la derivada
de la composición f −1 ◦ f en el punto a es igual a 1. De este modo, tenemos que la
derivada de f −1 en el punto f (a), multiplicada por la derivada de f en el punto a es
igual a 1. Por consiguiente la derivada de f en el punto a es distinta de 0. Esta es la
demostración para R (caso n =1)

Ahora se piensa si la regla de la cadena en Rn me resolverá el problema general


para Rn . La respuesta es afirmativa.

Al razonar en R, fue más fácil darse cuenta, de que aplicar la regla de la cadena
era la solución de mi problema. Para ver que dice la regla de la cadena en R y en Rn
consultar un libro riguroso de Análisis Matemático

115

Você também pode gostar